Communities Final Exam Practice Questions

Ace your homework & exams now with Quizwiz!

A nurse is reviewing data on the rates of varicella zoster (chicken pox) for a county. If there were 416 cases of varicella in one year among a population of 32,000 people, what should the nurse record as the incidence rate per 1,000 people?

13 cases per 1,000 people

A nurse is determining the attack rate following an e.coli outbreak at a restaurant. If 84 people ate contaminated lettuce, and 13 people developed e.coli infection, what should the nurse conclude as the attack rate?

25%

Which is a characteristic of nursing practice in rural environments? a. Anonymity b. Less autonomy c. Professional isolation d. Faster pace

ANS: C Concerns with professional isolation are often noted in rural practice environments. However, technology continues to improve the problems with isolation. A nurse working in a rural environment has no anonymity, has greater autonomy, and works at a slower pace.

A nurse is caring for a client who is from different culture than the nurse. When beginning the cultural assessment, which of the following actions should the nurse take? A. determine the clients perception of their current health status B. gather data about the clients cultural beliefs C. determine how the clients culture can affect the effectiveness of nursing actions D. gather information about previous client interactions with the health care system

ANS: B

A newly hired occupational health nurse at an industrial facility is performing an initial workplace assessment. Which of the following should the nurse determine when conducting a work site survey? A. Work practices of employees B. Past exposure to specific agents C. Past jobs of individual employees D. Length of time working in current role

ANS: A

A nurse at an urban community health agency is developing an education program for city leaders about homelessness. Which of the following groups should the nurse include as the fastest-growing segment of the homeless population? A. families with children B. adolescent runaways C. individuals who have experienced spouse or partner violence D. older adults

ANS: A

A nurse is planning interventions for migrant farm workers in a rural area. Which of the following should the nurse include as part of the primary prevention? A. establish food banks at locations throughout the community B. provide employers handouts on recognizing pesticide toxicity C. partner with clinics to provide on-site care for acute injuries D. create handouts on identifying skin cancer in multiple languages

ANS: A

A nurse is using the I PREPARE mnemonic to assess a clients potential environment exposures. Which of the following questions should the nurse ask when assessing for "A" in the mnemonic? A. "what do you like to do for fun?" B. "what year was your residence built?" C. "what jobs have you had in your past?" D. "what industries are near where you live?"

ANS: A

A nurse is discussing the role of a vector in the spread of disease. Which of the following is the nurse most likely referring to? a. Contaminated water b. A tick c. A dirty needle d. An infected person

ANS: B Vectors are arthropods such as ticks and mosquitoes that transmit the infectious agent by biting or depositing the infective material near the host. Contaminated water, a dirty needle, and an infected person would be common vehicles.

A study that uses information on current health status, personal characteristics, and potential risk factors or exposures all at once is called: a. cross-sectional. b. ecological. c. case-control. d. cohort.

ANS: A A cross-sectional study collects information on current health status, personal characteristics, and potential risk factors or exposures all at once. A cohort study is the type of epidemiologic study that is used to describe a group of persons enrolled in a study who share some characteristic of interest and who are followed over a period of time to observe some health outcome. An ecological model considers the multiple factors that contribute to disease development.

Which statement about disasters is true? a. They can be natural or human-made. b. They can be relieved without assistance. c. There is always injury and death when a disaster occurs. d. The timing of a disaster does not influence the types of injuries that will occur.

ANS: A A disaster is any natural or human-made incident that causes disruption, destruction, and/or devastation requiring external assistance. Although natural incidents like earthquakes of hurricanes trigger many disasters, predictable and preventable human-made factors can further affect the disaster. In the disaster response phase, the incident type and time predict subsequent injuries and illnesses.

A nurse considers an audience's ability to read, comprehend, and act on medical instructions while preparing health education materials. Which of the following factors is the nurse considering? a. Health literacy b. Resilience c. Social justice d. Health disparity

ANS: A A measure of a patient's ability to read, comprehend, and act on medical instructions is health literacy. Resilience has to do with the factors that allow people to resist the effects of vulnerability. Social justice describes justice with respect to the concepts of egalitarianism and equality. Health disparity refers to the wide variations in health services and health status among certain population groups.

A nurse plans to contact a multilateral organization. Which of the following would the nurse most likely contact? a. United Nations (UN) b. United States Agency for International Development (USAID) c. Project Hope d. Catholic Relief Services

ANS: A A multilateral organization is one that receives funding from multiple governments and nongovernmental sources, which describes the UN. Project Hope and Catholic Relief Services are private voluntary organizations. USAID is funded by one country, the United States.

A visitor from Japan comes to the United States for a 2-week vacation. Which of the following best describes this person? a. Nonimmigrant b. Refugee c. Legal immigrant d. Lawful permanent resident

ANS: A A nonimmigrant is admitted for a limited duration for a specific purpose. Refugees are people who seek protection in the United States because of fear of persecution in their homeland. Legal immigrants are people who are not citizens but are legally allowed to live and work in United States, usually because they fulfill labor demands or have family ties. Lawful permanent resident is another name for legal immigrants.

A nurse is completing a disease investigation. Based on the clinical symptoms presented, it is assumed that the individual has measles. However, there has been no laboratory evidence documenting that this individual has the disease. Which of the following types of cases is the nurse investigating? a. A suspected case b. A confirmed case c. A prolonged case d. An identified case

ANS: A A suspected case is a clinically compatible case of illness without isolation. A confirmed case is a clinically compatible case that is laboratory confirmed by isolation. A prolonged case and identified case are not types of cases discussed in the text.

A nurse is working in the community with an aggregate/population. Who is the nurse most likely to interact with? a. Students in a county school system b. Christians around the world c. A patient in the intensive care unit at the local hospital d. People who drink coffee

ANS: A A population or aggregate is a collection of individuals who have one or more personal or environmental characteristics in common. Members of a community who can be defined in terms of geography or a special interest can be seen as constituting a population. The clients in the first option share a geographical and special circumstance (school) characteristic. Christians around the world are too large of a geographical space. A patient cannot be an aggregate/population as there is no one to interact with. People who drink coffee share a common interest, but may not share a common geographical location to interact.

A nurse responds to a human-made disaster. Which of the following describes what the nurse is responding to? a. Structural collapse b. Communicable disease epidemics c. Mud slides d. Floods

ANS: A A structural collapse would be considered a human-made disaster. Communicable disease epidemics, mud slides, and floods are considered to be natural disasters.

A nurse working at a battered woman's shelter is counseling a pregnant woman who has just entered the shelter. When assessing the client's history, which information would the client most likely report? a. A history of abuse before pregnancy b. A history of child abuse c. A history of multiple pregnancies d. A history of substance abuse

ANS: A Abuse before pregnancy is the most important risk factor for physical battery during pregnancy. A history of child abuse, multiple pregnancies, and substance abuse are not as closely associated with the risk of physical battery during pregnancy.

A nurse plans to collect data about morbidity and mortality among migrant workers in a community. Which of the following barriers is the nurse most likely to encounter? a. Difficulty securing a stable population b. Lack of resources c. Unwilling participants d. Fear of deportation

ANS: A Accurate morbidity and mortality data are difficult to obtain due to farm workers leaving when no longer working, going back to their home to receive medical services, and decreases in infectious diseases that are easier to record.

A nurse is providing education to a mother about the importance of having her infant immunized for measles, mumps, and rubella. Which of the following best describes the type of immunity that will be provided? a. Active b. Passive c. Natural d. Acquired

ANS: A Active immunity refers to the immunization of an individual by administration of an antigen (infectious agent or vaccine) and is usually characterized by the presence of an antibody produced by the individual host. Passive immunity refers to immunization through the transfer of a specific antibody from an immunized individual to a non-immunized individual. Natural immunity refers to species-determined, innate resistance to an infectious agent. Acquired immunity is the resistance acquired by a host as a result of previous natural exposure to an infectious agent.

A nurse schedules an appointment with a physician who has a practice in the community to learn more about the community's beliefs regarding childhood immunizations. Which of the following is being demonstrated? a. Informant interview b. Participant observation c. Active participation d. Windshield survey

ANS: A An informant interview is a method of community data collection that involves directed conversation with selected community members. Participant observation refers to the deliberate sharing in the life of a community, such as participating in a local fair or festival, or attending a political or social event. Active partnerships are those in which all participants share leadership and decision making to some degree. Windshield surveys are a method of simple observation, providing a quick overview of a community.

Which of these statements is true about the DALYs in lesser-developed countries? a. Children are at great risk for an early death. b. Diarrheal disease and respiratory infections are rampant despite technology. c. Injuries account for most disability. d. Birth control methods are often lacking.

ANS: A Approximately 2.5 million neonatal deaths occurred in 2011 and over 20 million children under age 5 died during the same year in lesser-developed countries. Children are at great risk for an early death in these lesser-developed nations. If these children could face the same risks as those in developed nations, the deaths would decrease by 90%. Noncommunicable diseases such as respiratory, cardiac, musculoskeletal, and other conditions are problems in all countries.

The purpose of public health core functions is to: a. clarify the role of the government in fulfilling the mission of public health. b. ensure the safety of populations in receiving quality health care. c. provide community-based individualized care to every person in the United States. d. unite public and private providers of care in a comprehensive approach to providing health care.

ANS: A As defined by the Institute of Medicine in its 1988 report The Future of Public Health, assessment, policy development, and assurance are core functions at all levels of government for the purpose of clarifying the government's role.

A PHN collects data and monitors the health status of the population. Which of the following core public health functions is being implemented? a. Assessment b. Prevention c. Assurance d. Policy development

ANS: A Assessment consists of systematic data collection and monitoring health status. Prevention is not a core function, assurance is making sure essential services are available, and policy development is needed to provide leadership in developing policies.

The nurse manager makes sure that the staff members who work in a local clinic are competent in their job responsibilities. Which of the public health core functions is being demonstrated? a. Assurance b. Assessment c. Prevention d. Policy development

ANS: A Assurance refers to making sure a competent health care workforce is available. Prevention is not a core function, assessment refers to systematic data collection, and policy development refers to the need to provide leadership in developing health policies.

A nurse is concerned about an event of bioterrorism. Which of the following would the nurse most likely be referring to? a. Outbreak of smallpox b. Bombed historic buildings c. Explosion of land mines d. Threats of war

ANS: A Bioterrorism is a term used to describe the deliberate release of viruses, bacteria, or other germs (agents) used to cause illness or death in people, animals, or plants. Bombing historical buildings, explosion of land mines, and threats of war may be part of terrorist attacks, but do not include the component of bioterrorism.

An example of a successful campaign against which of the following communicable diseases was carried out during the 1960s and 1970s? a. Tetanus b. Measles c. Pertussis d. Smallpox

ANS: D Smallpox was virtually eliminated throughout the world as a result of immunization. Tetanus, measles, and pertussis all continue to exist throughout the world.

Change for the community as a client must often occur at several levels because: a. health problems caused by lifestyle are multidimensional. b. most individuals can change their habits alone. c. aggregates are responsible for social change. d. geographic areas often have health risks that the nurse must identify.

ANS: A Because health problems caused by lifestyle cannot be solved simply by asking individuals to choose healthy habits, change for the community client must often take place at several levels. Society must also be involved in the change by supporting individual choices. There may be certain geographic areas that have higher health risks than others, but this does not explain why care must occur at several levels.

Research has shown that Asian men tend to have a greater sensitivity than white Europeans to codeine, and they experience significantly weaker effects from the drug. Which of the following types of cultural variations is being demonstrated? a. Biological variations b. Personal space c. Social organization d. Perception of time

ANS: A Biological variations are the physical, biological, and physiological differences that exist between racial groups and distinguish one group from another. Personal space is the physical distance between two individuals during an interaction. Social organization refers to the way in which a cultural group structures itself around the family to carry out role functions. Perception of time is the duration or period between successive events, where some cultures assign greater or lesser emphasis to events that occur in the past, present, or future.

Which historical event caused dramatic changes in home care nursing? a. Passage of Medicare legislation b. Establishment of Visiting Nurse Associations c. Creation of the Montefiore Hospital Home Care Program d. Provision of funds by the Civil Works Administration

ANS: A Medicare introduced regulations for home care practice as well as for reimbursement mechanisms.

A nurse examines birth and death certificates during an epidemiologic investigation. Which of the following data categories is being used? a. Routinely collected data b. Data collected for other purposes but useful for epidemiologic research c. Original data collected for specific epidemiologic studies d. Surveillance data

ANS: A Birth and death certificates are considered to be vital records and are examples of data collected routinely. Data collected for other purposes would be hospital, physician, health department, laboratory, and insurance records. Original data is that which is collected by the National Center for Health Statistics for specific health surveys. Surveillance data is used to assess and prioritize the health needs of populations, design public health and clinical services to address those needs, and evaluate the effectiveness of public health programs.

Which statement about community health nursing practice is correct? a. It focuses on the delivery of personal health services to individuals and families. b. It provides care to protect the health of the community as a whole. c. It emphasizes the setting where care is provided for clients and families. d. It requires a baccalaureate preparation for practice.

ANS: A By definition, the first option is correct. Public health nursing provides care to protect the health of the community as a whole. Community-based nursing practice emphasizes the setting where care is provided for clients and families. It is generally assumed that a graduate of any baccalaureate program has the necessary basic preparation to function as a beginning staff PHN; however, this is not a requirement for public health nursing or community health nursing practices.

What are the six "rights" of case management? a. Care, time, provider, setting, price, and outcomes b. Patient, medication, route, time, documentation, and evaluation c. Place, setting, patient, plan, outcomes, and documentation d. Disease process, time, place, beneficence, advocate, and care provider

ANS: A Care, time, provider, setting, price, and outcomes are used to judge the effectiveness of case management.

A nurse is working in a temporary shelter for victims following a natural disaster. Which condition is the nurse most likely to encounter? a. Stress b. Communicable disease c. Depression d. Injuries requiring first aid

ANS: A Causes of stress can be the shock of the disaster itself, loss of personal possessions, fear of the unknown, living in close proximity to total strangers, and boredom. Illnesses requiring first aid and communicable diseases are not the primary concern of the nurse working in the shelter. Immediately following the disaster, it is unlikely that the victims will immediately encounter depression because of the shock of the disaster.

A nurse is caring for a migrant farm worker who has been working in the agricultural industry for the past 10 years. When questioned about environmental hazards, the client reports regular exposure to pesticides. Which of the following is the client at increased risk of developing? a. Cancer b. Memory loss c. Skin rashes d. Headaches

ANS: A Chronic exposure to pesticides may lead to cancer. Acute exposure to pesticides may cause memory loss, skin rashes, and headaches.

A nurse is assessing a client who has had an acute exposure to a pesticide. Which of the following symptoms is the client most likely to display? a. Memory loss b. Musculoskeletal problems c. Cancer d. Infertility

ANS: A Memory loss is a symptom that is displayed following an acute exposure to a pesticide. Musculoskeletal problems, cancer, and infertility are from chronic exposure.

What information is shared among agencies when they collaborate to implement a quality surveillance system? a. How to use algorithms to identify which events should be investigated b. Who is to blame for a disease outbreak c. What shelters will be used and by whom d. How political action will be necessary to ensure public health

ANS: A Collaboration promotes the development of plans and a directory of emergency responses. How to use algorithms is a key type of information that is shared. Blaming others is not part of collaboration. Determining what shelters will be used and by whom is not the priority for collaboration. How political action will be necessary to ensure public health is not a priority for collaboration.

A public health worker had displayed a competency in disaster medicine and public health. Which of the following describes what the worker has done? a. Communicate effectively with others b. Get involved in the chain of command c. Use a set of preplanned activities for every disaster d. Implement a community assessment

ANS: A Communicating effectively with others in a disaster or public health emergency is one of the competencies demonstrated in disaster medicine and public health. Involvement in the chain of command, using a set of preplanned activities for every disaster, and implementing a community assessment are not part of these competencies.

A nurse is working in a community health nursing practice setting. Which of the following is the nurse most likely to implement? a. Administrating a flu shot to a client in a physician's office b. Conducting a flu shot clinic at a community center c. Performing a client assessment in a hospital d. Providing supervision of staff in a rehabilitation center

ANS: A Community health nursing practice focuses on the health of individuals, families, and groups and the effect of their health status on the health of the community as a whole. Administrating a flu shot to an individual is the only example that meets this criterion. Performing a client assessment focuses only on individual care and not the community. Providing supervision of staff does not focus on the community. Conducting a flu shot clinic at a community center focuses on protecting the community as a whole and would be considered public health nursing practice.

The nurse is investigating environmental health problems caused by contaminated ground water. Which of the following types of nursing practice is being used? a. Community-oriented b. Community-based c. Policy development d. Tertiary care

ANS: A Community-oriented nursing emphasizes the prevention of disease and disability. Community-based nursing practice is a setting-specific practice whereby care is provided for clients and families where they live, work, and attend school. Policy development seeks to build constituencies that can help bring about change in public policy. Tertiary care focuses on highly specialized medical care.

A nurse is working with incarcerated adults who are being released from prison. Which of the following nursing interventions would be most appropriate to implement? a. Connect offenders with community-based mental health programs b. Provide community supervision for mentally ill offenders c. Advocate for increased prison time to decrease recidivism rates d. Educate about available state resources

ANS: A Connecting offenders with community-based mental health programs at the time of release from prison can decrease recidivism rates, as many incarcerated adults experience major psychiatric disorders. It would be impossible for the nurse to supervise all mentally ill offenders in the community. Increased prison time is not shown to decrease recidivism rates and education about available state resources is not the most important nursing intervention to provide for this population.

To meet a client's needs, it is sometimes necessary to integrate into the client's care a culturally relevant practice that lacks scientific utility. Which of the following best describes this action? a. Cultural accommodation b. Cultural awareness c. Cultural preservation d. Cultural repatterning

ANS: A Cultural accommodation refers to assistive, supportive, facilitative, or enabling nurse actions and decisions that help people of a particular culture to accept nursing strategies, or to negotiate with nurses to achieve satisfying health care outcomes. Cultural awareness refers to the self-examination and in-depth exploration of one's own biases, stereotypes, and prejudices as they influence behavior toward other cultural groups. Cultural preservation refers to assistive, supportive, facilitative, or enabling nurse actions and decisions that help the clients of a particular culture to retain and preserve traditional values so they can maintain, promote, and restore health. Cultural repatterning refers to assistive, supportive, facilitative, or enabling nurse actions and decisions that help clients of a particular culture to change or modify a cultural practice for new or different health care patterns that are meaningful, satisfying, and beneficial.

A nurse is working with an immigrant population. Which of the following should be the first action taken by the nurse? a. Be aware of one's own culture b. Become familiar with traditional practices of the immigrants c. Try to see things from the immigrant's viewpoint d. Learn to speak the language of the immigrant population

ANS: A Cultural competence is one of the core attributes of public health nurses. Nurses come from a variety of cultural backgrounds and have their own cultural traditions. Nurses also bring their biomedical beliefs and values to the practice environment that may differ from the client's own beliefs and values. Because nurses recognize their own culture, they are better able to understand that there are differences among cultures. Being aware of one's own culture should be done before completing the other tasks of increasing familiarity with traditional practices, trying to see things from the other's point of view, or learning to speak the language.

Which health problem is in need of control in developed countries? a. Hepatitis b. Malaria c. Polio d. Smallpox

ANS: A Current health concerns in more developed countries are hepatitis, infectious diseases, and new viral strains such as hantavirus, SARS (severe acute respiratory syndrome), H1N1, and avian flu. Malaria, polio, and smallpox are more commonly found in lesser-developed countries.

A nurse plans to address the increasing prevalence of depression in the community. Which of the following would be the best strategy to implement? a. Educate the community about how depression affects men b. Recognize available community resources c. Advocate for health policy changes d. Understand how free clinics are able to provide mental health services to those without health insurance

ANS: A Depression between men and women is recognized differently in the community. More women than men are classified as having depression. Men with depression often go unrecognized and underreported. Men tend to be stoic and do not verbalize how they feel, are reluctant to talk about health issues, and often do not have positive relationship with their health care provider. Recognition is key in order to get those with depression the necessary services that they need. Once depression has been detected, these individuals can be referred to the appropriate resources. Health policy changes and the understanding of free clinics would not provide the most immediate solution to the community.

A nurse is assisting with recruitment of health care providers to a primary care facility. Which of the following facts about the rural population and their patterns of seeking medical care should be considered? a. Fewer rural adults seek medical care than urban adults. b. There is a greater population to serve in rural areas. c. Poor adults in urban areas have less access to care. d. Rural adults do not want to seek care.

ANS: A Despite their overall poorer health status and higher incidence of chronic health conditions, rural adults seek medical care less often than urban adults. There are resources for care in rural areas, but in some cases this may be scarce. The lack of providers or other resources in some rural areas may make it difficult for residents to obtain care. In most rural areas, the population is smaller than in an urban area.

A patient with a long history of asthma with many hospital admissions is referred to a case manager for disease management. Which of the following best describes the purpose of this referral? a. Chronic and costly disease conditions that require long-term care interventions b. Patients who cannot handle their disease c. Those who seek to control use by providing clients with correct information d. Patients who will need an advanced practice nurse instead of physician for monitoring

ANS: A Disease management activities target chronic and costly disease conditions that require long-term care interventions. Demand management seeks to control use by providing clients with correct information. Patients who cannot handle their disease or who need an advanced practice nurse instead of a physician do not explain the referral to disease management.

A nurse is applying the principle of distributive justice. Which of the following describes the benefits that will occur through application of this principle? a. Basic needs, material and social goods, liberties, rights, and entitlements b. Taxes, military service, location of incinerators or power plants c. Entitlement to equal rights and equal treatment d. The right to private property and personal assets

ANS: A Distributive justice requires that the distribution of benefits and burdens on a society be fair or equal. Entitlement to equal rights and equal treatment refers to egalitarianism. The right to private property and personal assets refers to libertarianism. Taxes, military service, and location of incinerators or power plants are not benefits associated with justice.

A nurse has identified a point source of air pollution. Which of the following has the nurse identified? a. A smoke stack b. The number of cars and trucks c. The amount of fossil fuel consumed in a community d. Ground ozone levels

ANS: A Point sources of pollution are identifiable sources of air pollution, such as a smoke stack. Nonpoint sources come from more diffuse exposures, such as from cars and trucks. The amount of fossil fuel that is consumed and ground ozone levels do not identify sources of air pollution.

A community health nurse is caring for an individual who is experiencing crisis poverty. Which of the following characteristics would this individual most likely display? a. Episodic homelessness b. Physical disability c. Mental disability d. Drug abuse

ANS: A Episodic homelessness is a characteristic of crisis poverty. Physical disability, mental disability, and drug abuse would be considered in persistent poverty.

A nurse believes that the best treatment for illness is the use of Western medicine and alternative therapies should not be used for healing. Which of the following best describes what has happened? a. Ethnocentrism b. Cultural imposition c. Racism d. Stereotyping

ANS: A Ethnocentrism is the belief that one's own cultural group determines the standards by which another group's behavior is judged. Cultural imposition is the act of imposing one's cultural beliefs, values, and practices on individuals from another culture. Racism is a form of prejudice that occurs through the exercise of power by individuals and institutions against people who are judged to be inferior on the basis of intelligence, morals, beauty, inheritance, and self-worth. Stereotyping is ascribing certain beliefs and behaviors about a given racial and ethnic group to an individual without assessing for individual differences.

Rurality is a subjective concept because: a. everyone has an idea as to what constitutes a rural setting. b. the differences between rural and urban areas are very distinct. c. rural health has not been very well researched. d. the demographic and social characteristics of urban people are similar.

ANS: A Everyone has an idea as to what constitutes rural as opposed to urban residence. With the increased degree of urban influence on rural communities, the differences are no longer as distinct as they may have been in the past. There has been research done on rural health.

A nurse states that the client has exhibited an explicit cultural behavior. Which of the following has the nurse most likely observed? a. Verbal communication b. Body language c. Use of titles d. Perception of health and illness

ANS: A Explicit behaviors are straightforward and do not leave room for misinterpretation of what the person wants to communicate, such as when using verbal communication. Implicit behaviors are less exact and may be difficult to interpret, including body language, use of titles, and perception of health and illness.

Which of the following health problems ranks as one of the top five problems for farm workers aged 5 to -19 years? a. Dental problems b. Communicable diseases c. Gastrointestinal problems d. Stress

ANS: A Farm workers of all ages consistently have more dental disease than the general population.

A nurse is providing care to populations who are from different socioeconomic and cultural backgrounds. Which of the following barriers should the nurse be aware of that may prevent full engagement in a relationship? a. Fears and misconceptions related to poverty b. Lack of tangible and emotional resources c. Lack of knowledge about people who live in poverty d. Perception that the poor don't need to be poor

ANS: A Fears and misconceptions of nurses related to poverty is a barrier that may prevent nurses from fully engaging in relationships with people who come from different socioeconomic and cultural backgrounds. It is important for the nurse to be a good listener, individualize care, and avoid making inappropriate assumptions about their needs. It is important to listen to the stories of their lives as well as examine social and cultural definitions and considerations related to poverty.

A nurse is referring to the federal income guidelines. Which of the following best describes why the nurse is completing this action? a. To determine financial eligibility for government assistance programs b. To calculate statistical occurrences in the community c. To identify those with inadequate resources for basic needs d. To provide guidelines for reform of social programs

ANS: A Federal income guidelines are used primarily to determine financial eligibility for government assistance programs. The Poverty Threshold Guidelines are used primarily for statistical purposes. The federal income guidelines do not identify those with inadequate resources for basic needs or provide guidelines for reform of social programs.

A public health nurse (PHN) is working with a community during the recovery phase of a disaster. Which of the following attributes would be most important for the nurse to possess? a. Flexibility b. Organization skills c. Personable d. Sense of humor

ANS: A Flexibility is key to a successful recovery operation. The role of the PHN in the recovery phase of a disaster is as varied as in the preparedness and response phases, but the PHN's connection to the community puts the PHN in an incredible position of knowledge and awareness on the interprofessional recovery team. The attributes of being organized, personable, and having a sense of humor are not as important as flexibility when dealing with this phase of the disaster.

Which statement regarding Florence Nightingale's ideas about ethics is correct? a. Nursing is a call to service, and the moral character of persons entering nursing is important. b. Ethical principles are based on the values of the individual nurse. c. Society will dictate the ethical principles to which nurses must adhere. d. Ethics are very important in times of war, such as in the Crimean War, when she set up public health centers.

ANS: A Florence Nightingale saw nursing as a call to service and viewed the moral character of persons entering nursing as important. Florence Nightingale did not set up public health centers. Florence Nightingale did not believe that nurses must adhere to society's view of ethical principles. Ethical beliefs are based on the values of the individual nurse, not ethical principles.

Food intoxication is caused by: a. toxins produced by bacterial growth and chemical contaminants. b. bacterial, viral, or parasitic invasion of food. c. overcooking of meat and produce. d. adding too many spices or ingredients to food.

ANS: A Food intoxication is caused by toxins produced by bacterial growth, chemical containments, and a variety of disease-producing substances found naturally in certain foods such as mushrooms and some seafood. Bacterial, viral, or parasitic invasion of food is not a cause of food intoxication. Food intoxication is not caused by overcooking meat or adding too many ingredients to food.

A leader of a support group introduces a nurse to its members and discusses the professional relationship he has with the nurse for the past several years. Which of the functions is the leader performing? a. Gatekeeper b. Insider c. Community health workers d. Advocate

ANS: A Gatekeepers refer to formal or informal community leaders who create opportunities for nurses to meet diverse members of the community. Insiders are those who grew up in the community, have personal ties to the people there or comes from a similar cultural or ethnic background. Community health workers are not professional or licensed health care providers but are community members from diverse backgrounds who receive training to do health outreach work. An advocate is someone who speaks up for and supports the needs of the community.

A nurse is partnering with an organization to improve affordable housing in rural areas of the country. Which of the following best describes who the nurse is collaborating with? a. Housing Assistance Council (HAC) b. Local housing authority c. Temporary Assistance for Needy Families (TANF) d. Public Housing Development for Families

ANS: A HAC is a nonprofit organization whose mission is to improve affordable housing in rural areas. TANF is a program that assists families in need. The local housing authority provides public housing assistance. Public Housing Development for Families promotes the development of local strategies for affordable public housing.

Current research on hormone replacement therapy (HRT) has found that it: a. does not prevent heart disease. b. should be used to prevent osteoporosis. c. is useful with complementary therapies. d. causes breast cancer.

ANS: A HRT does not prevent heart disease. To prevent heart disease, women should avoid smoking, reduce fat and cholesterol intake, limit salt and alcohol, maintain a healthy weight, and be physically active. HRT should be used to prevent osteoporosis only among women who are unable to take nonestrogen medications. HRT is not useful with complementary therapies and does not cause breast cancer.

What is the best method for preventing health care-associated infections? a. Perform good hand washing before and after approaching every patient. b. Prevention is almost impossible due to the high infection rates in hospitals. c. Isolate every patient having surgery. d. Use contact isolation for every patient at risk.

ANS: A Hand washing is the best way to prevent infection. If good hand washing is in place, it is not necessary to isolate patients or implement contact isolation. Prevention is possible with the use of good hand washing.

Homicide is the second leading cause of death among which population? a. Individuals between 15 and 24 years of age b. Native American infants c. Caucasian women between 25 and 34 years of age d. African American children between 15 and 19 years of age

ANS: A Homicide is the second leading cause of death for individuals between 15 and 24 years of age. Rates of homicide for children under the age of 1 were highest in Non-Hispanic blacks and American Indian/Alaska Native infants.

Which group has the highest rate of poverty? a. Children b. Teenagers c. Women d. Older adults

ANS: A In 2012, 21.6% of those 18 years or less were living in poverty. Poverty is highest among children, and in 2012, it was 21.8% higher than any other age group.

A nurse is completing a community assessment. Which of the following actions would be most likely for the nurse to complete? a. Identify community needs and clarify problems b. Determine the weaknesses of a community c. Perform the core functions of public health nursing d. Assess individual needs within a community

ANS: A In a community assessment, one of the core functions is a logical, systematic approach to identifying community needs, clarifying problems, and identifying community strengths and resources. The focus of the community assessment is on the needs of the community, not on individuals. The core functions of public health nursing are not all used during the community assessment process. The community assessment should identify both the strengths and weaknesses of the community.

A public health nurse (PHN) is investigating an outbreak of salmonellosis in a community. Which type of surveillance system is being used? a. Active b. Passive c. Sentinel d. Special

ANS: A In the active system, a PHN begins to search for cases through contacts in the community. The nurse names the disease and gathers data about existing cases to try to determine the magnitude of the problem. In the passive system, case reports are sent to local health departments by health care providers. In the sentinel system, trends in commonly occurring diseases or key health indicators are monitored. Special systems are developed for collecting particular types of data and may be a combination of active, passive, and/or sentinel systems.

A nurse is providing education to a parent-teacher organization about substance use and abuse among children. Which of the following information would the nurse most likely include in this presentation? a. Inhalants are among the first drugs that young children use. b. Implementing the Drug Abuse Resistance Education (DARE) program is the most effective way to prevent drug use. c. Underage drinking is not a problem in most communities. d. Early experimentation with substances decreases the likelihood of future abuse.

ANS: A Inhalants are among the first drugs that young children use. The primary abusers of most inhalants are adolescents who are 12-17 years of age. Most recent studies have found that the DARE program is less effective than other interactive prevention programs and may even result in increased drug use. Underage drinking is seen as the most serious drug problem for youth in the United States. The younger a person is when beginning intensive experimentation with drugs, the more likely dependence will develop.

A nurse is caring for a client who has been diagnosed with tuberculosis (TB). Which of the following medications would the nurse most likely administer? a. Isoniazid b. Nevirapine c. Amodiaquine d. Bacille Calmette-Guérin (BCG)

ANS: A Isoniazid is one of several chemotherapeutic medications used to treat TB. Nevirapine is used to decrease the likelihood of maternal transmission of HIV-AIDS. Amodiaquine is one of several medications used to treat malaria. BCG is a vaccine that has been promoted as effective in inducing active immunity against TB.

John Snow is called the "father of epidemiology" because of his work with: a. cholera. b. malaria. c. polio. d. germ theory.

ANS: A John Snow investigated the spread of cholera in the mid-nineteenth century. John Snow did not investigate the other examples.

A nurse is investigating the overall health status of a population. Which of the following indicators would most likely be used by the nurse? a. Life expectancy b. Health status c. Morbidity rate d. Prevalence

ANS: A Life expectancy is a measure that is often used to gauge the overall health of a population. Health status indicators are the quantitative or qualitative measures used to describe the level of well-being or illness present in a defined population or to describe related attributes or risk factors. Morbidity measures rate of disability or illness. Prevalence is the proportion of the population that has a specific disease or illness.

A nurse is caring for a population that has experienced a health disparity. Which of the following best describes a health disparity? a. Low childhood immunization rates b. High dropout rates c. Unemployment d. Income below poverty level

ANS: A Low childhood immunization rates signify a health disparity. High dropout rates, unemployment, and income below poverty level are social conditions that may lead to health disparities.

A nurse who is using population management needs to be able to work with integrated care delivery systems. Which of the following describes the rationale for this competency? a. Management has shifted from inpatient care to primary care providers as points of entry. b. Emphasis is on episodic illness care for individuals rather than on population management. c. Care management services and programs do not provide access and accountability, as provided by case management services. d. Assessment of the needs of the population is no longer necessary.

ANS: A Management has shifted from inpatient care as a point of entry to primary care providers as points of entry. The other statements are false.

The nurse is caring for a migrant family. During the interview, the mother tells the nurse that the family follows an Eastern migratory stream originating in southern Florida. Which of the following statements is accurate? a. This is one of the migratory streams that migrant farm workers traditionally follow. b. This is a unique migratory pattern. c. This is a traditional family-generated migratory pattern. d. This is an unpredictable pattern that may lead the family through southern California.

ANS: A Migrant workers traditionally have followed one of three migratory streams: (1) Eastern, originating in Florida; (2) Midwestern, originating in Texas; and (3) Western, originating in California.

A riverfront community builds a retaining wall to divert flood water away from the town. Which term describes what has happened? a. Mitigation b. A natural disaster c. Community resilience d. Rapid needs assessment

ANS: A Mitigation is defined as actions or measures that can either prevent the occurrence of a disaster or reduce the severity of the effects. A natural disaster, such as an earthquake or hurricane, causes disruption, destruction, and/or devastation requiring external assistance. Community resilience is defined as the sustained ability of a community to withstand and recover from adversity. A rapid needs assessment is based on the traditional model of community assessment where there is a rapid appraisal of a sector or region's population, social systems, and geophysical features.

A nurse is implementing a secondary prevention strategy. Which of the following activities would the nurse most likely implement? a. Monitoring for prenatal care b. Treatment for anemia c. HIV prevention d. Tetanus immunization

ANS: A Monitoring for prenatal care is an example of a secondary prevention strategy. Treatment for anemia is tertiary prevention, and HIV prevention and tetanus immunization are primary prevention strategies.

Which statement is true about mortality rates? Mortality rates: a. are informative only for fatal diseases. b. provide information about existing disease in the population. c. are calculated using a population estimate at year-end. d. reveal the risk of getting a particular disease.

ANS: A Mortality rates are informative only for fatal diseases and do not provide direct information about the level of existing disease or the risk of getting a particular disease. Because the population changes during the course of a year, typically an estimate of the population at midyear is taken as the denominator for annual rates, because the midyear population approximates the amount of person-time contributed by the population during a given year.

Multilateral organizations: a. receive funding from multiple sources. b. control the spread of disease. c. feed the people of the world. d. use nurses as their main source of information.

ANS: A Multilateral organizations are those that receive funding from multiple government and nongovernment sources. The other items are specific to the agencies listed, but do not include all of the organizations.

A nurse is providing care to a Native Alaskan client. Which of the following exposures should the nurse anticipate may require specialized community nursing care? a. Exposure to toxic substances b. Farmer's lung c. Field sanitation d. Black lung

ANS: A Native Alaskans have special care needs related to exposure to toxic substances/contaminants, hypothermia, interpersonal/domestic violence, infectious diseases, dental caries/loss, and diabetes. Farmer's lung is associated with farmers/ranchers. Field sanitation is associated with migrant farmworkers. Black lung is associated with coal miners.

A nurse is applying the ethical principle of non-maleficence. Which of the following describes the action that the nurse is taking? a. Administering medications using the "five rights" b. Allowing clients to be active participants in their care c. Providing patient privacy when delivering care d. Referring a client to a physical therapist

ANS: A Non-maleficence requires that one do no harm. It requires that health care professionals act according to the standards of due care, always seeking to produce the least amount of harm possible. Providing privacy when delivering care demonstrates the client's right to privacy. Allowing clients to be active participants in their care refers to the ethical principle of the right to autonomy. Referring a client to a physical therapist demonstrates the nursing role of referral agent.

A nurse is working with a recent rape survivor. Which of the following actions would be most appropriate for the nurse to implement? a. Provide continuous care once the victim enters the health care system b. Examine evidence for its authenticity c. Work with the criminal justice system to find the rapist d. Provide long-term therapy for psychological trauma

ANS: A Not only do nurses often provide continuous care, in some states nurses are trained as sexual assault examiners who gather forensic evidence. Nurses provide victims with privacy, respect, and assurance of confidentiality. The nurse would not be providing long-term therapy for the survivor or have the responsibility to find the rapist.

As a result of an outbreak of influenza in a community, a nurse encourages members of the community to receive the influenza vaccine. Which of the following levels of prevention is being used? a. Primary prevention b. Secondary prevention c. Tertiary prevention d. Multifactorial prevention

ANS: A Nurses are involved in epidemiologic surveillance by monitoring the potential for disease outbreaks. Primary prevention refers to interventions aimed at preventing the occurrence of disease, injury, or disability. Immunizations are an example of primary prevention. Secondary prevention interventions are designed to increase the probability that a person with a disease will have that condition diagnosed at a stage when treatment is likely to result in cure. Tertiary prevention includes interventions aimed at disability limitation and rehabilitation from disease, injury, or disability. Multifactorial prevention is not possible with a disease like influenza, which has one cause.

A nurse is striving to be culturally competent. Which of the following actions would most likely be taken by the nurse? a. Respect individuals from different cultures and value diversity b. Immerse themselves in different cultures c. Design care for special ethnic groups d. Give explicit instructions to avoid client decision making

ANS: A Nurses who strive to be culturally competent respect individuals from different cultures and value diversity. Immersing oneself in a different culture, designing care for special ethnic groups, and giving explicit instructions so the client does not have to make a decision does not demonstrate cultural competence.

A nurse is making a home visit to a low-income, frail, older person's home. Which of the following would most likely cause the nurse to suspect adult maltreatment? a. The older person reports giving money to family members on a regular basis. b. The windows of the house are in need of repair. c. The older person spends a great deal of time watching television. d. There are dogs running loose in the yard of the home.

ANS: A Older adults are at risk for financial abuse through fraud, coercion to relinquish property rights, and money mismanagement. A low-income, frail, older person would most likely not be giving money to family members on a regular basis, because the client would need the money to meet basic needs.

Which is a feature of public health surveillance? a. Sharing of the results with others b. Defining public health policy c. Evaluating interventions d. Planning national programs

ANS: A One of the features of public health surveillance is sharing of the results with others. Defining public health policy, evaluating interventions, and planning national programs are all purposes of surveillance, not features.

A nurse is implementing a population-focused model of home care delivery. Which of the following actions would most likely be taken by the nurse? a. Use an assessment protocol when conducting visits b. Collaborate with other disciplines c. Provide care for a specific population d. Apply high-intensity interventions

ANS: A Population-focused home care models of care usually include structured approaches to regular visits with assessment protocols, focused health education, counseling, and health-related support and coaching.

Which of the following is most likely to live in poverty? a. Those who work in high-risk jobs b. Those who have adequate nutrition c. Those who effectively manage stress d. Those who live in single family homes

ANS: A People who are poor are more likely to live in hazardous environments that are overcrowded and have inadequate sanitation, work in high-risk jobs, have less nutritious diets, and have multiple stressors.

Why is it important for nurses to understand the premises of environmental health? a. Nurses should be able to assess risks and advocate for policies that support healthy environments. b. Toxicologists often consult nurses about environmental pollutants. c. Pollutant exposures such as lead are reported by nurses to the Environmental Protection Agency (EPA). d. Many Americans live in areas that do not meet current national air quality standards.

ANS: A Potential risks to health are concerns for professional nurses. It is the responsibility of the nurse to understand as much as possible about these risks: how to assess them, how to eliminate/reduce them, how to communicate and educate about them, and how to advocate for policies that support healthy environments. Toxicologists do not often consult nurses about environmental pollutants. Pollutant exposures are not routinely reported by nurses. Although there may be problems with air quality standards in the United States, this would not be the primary reason why nurses should understand environmental health.

A nurse is completing an exposure history using the mnemonic I PREPARE. What data would a nurse collect when asking questions about the first "P"? a. Present work b. Potential exposures c. Personal protective equipment use d. Past work

ANS: A Present work is the first "P". Potential exposures are part of the "I"—investigate potential exposures. Personal protective equipment use is not part of the mnemonic. Past work is the second "P."

A screening for diabetes revealed 20 previously diagnosed diabetics and 10 probable new cases, which were later confirmed, for a total of 30 cases. Which of the following best describes what is being measured? a. Prevalence b. Incidence c. Attack rate d. Morbidity rate

ANS: A Prevalence is the measure of existing disease in a population at a particular time. Incidence quantifies the rate of development of new cases in a population at risk, whereas an incidence proportion indicates the proportion of the population at risk who experience the event over some period of time. Attack rate is defined as the proportion of persons who are exposed to an agent and develop the disease. Morbidity rate is the incidence of disease.

A school nurse is coordinating the implementation of a drug education program in the school. Which of the following levels of prevention is being implemented? a. Primary level of prevention b. Secondary level of prevention c. Tertiary level of prevention d. Primary health care prevention

ANS: A Primary prevention includes education about drugs and guidelines for their use, preventing the problem before it occurs. Secondary prevention aims for early detection and screening. Tertiary prevention addresses treatment and rehabilitation. Primary health care prevention is not a level of prevention.

A nurse helps the community to take a stand against violence and advocates for elected officials and local media to make nonviolence a priority. Which of the following levels of prevention is being implemented? a. Primary b. Secondary c. Tertiary d. Primary care

ANS: A Primary prevention refers to those interventions aimed at preventing the occurrence of disease, injury, or disability. Being an advocate for nonviolence demonstrates primary prevention. Secondary prevention refers to interventions aimed at early detection and screening. Tertiary prevention refers to interventions aimed at treatment and rehabilitation. Primary care is not a level of prevention.

A PHN conducts an immunization clinic for measles. Which of the following is being implemented? a. Primary prevention b. Secondary prevention c. Tertiary prevention d. Health promotion

ANS: A Primary prevention refers to those interventions aimed at preventing the occurrence of disease, injury, or disability. Immunizations are an example of primary prevention. Secondary prevention interventions are designed to increase the probability that a person with a disease will have that condition diagnosed at a stage when treatment is likely to result in cure. Tertiary prevention includes interventions aimed at disability limitation and rehabilitation from disease, injury, or disability. Health promotion is a specific primary prevention strategy.

A nurse promotes the use of universal precautions by all health care workers. Which of the following best describes the action that was taken by the nurse? a. Primary prevention b. Secondary prevention c. Tertiary prevention d. Health care-associated infection

ANS: A Primary prevention refers to those interventions aimed at preventing the occurrence of disease, injury, or disability. Secondary prevention seeks to prevent the spread of infection and/or disease once it occurs. Tertiary prevention reduces complications through treatment and rehabilitation. Health care-associated infections are prevented through the use of good hand washing.

A local senator has proposed changes to the health care delivery system in the United States. Based on current trends, which of the following is most likely to occur? a. There will be new opportunities provided for public health specialists. b. It will result in isolated care being provided to individuals. c. There will be an increased emphasis in specialty care. d. It will increase the utilization of acute care services.

ANS: A Proposed changes in the health care delivery system will provide new opportunities for public health specialists and result in the creation of new roles within the system. The proposed changes will build an integrated system (not isolated), decrease emphasis on acute care services, and focus on health promotion and disease prevention.

A nurse is employed in home health nursing practice. Which of the following terms best describes the care that is being provided? a. Intermittent nursing care b. Custodial care c. Family caregiving d. Palliative care

ANS: A Provision of intermittent skilled visits is a component of home health care nursing. Custodial care refers to the care given in long-term care facilities. Family caregiving may be an important role of keeping the client at home, but it is not a necessary component of home care. Palliative care is an extended continuum of chronic serious illness to acute serious illness during which stabilization and exacerbations may occur.

Public health nursing is a specialty because: a. it has a distinct focus and scope of practice. b. it must be done by a registered nurse with a master's degree. c. it is focused on disadvantaged citizens. d. it performs interventions at the acute care level.

ANS: A Public health nursing has a distinct focus and requires a special knowledge base. The other answers are not characteristics of a specialty.

Resilience refers to the: a. resistance of certain groups to risk factors. b. increased susceptibility to cumulative risk factors among vulnerable groups. c. variability in the effects of stressors according to socioeconomic status. d. increased sensitivity of the very young and the very old to risk factors.

ANS: A Resilience refers to how vulnerable populations are able to resist or overcome the effects of the vulnerability. These populations do not succumb to the health risks that impinge on them.

A nurse who had been working in an urban area is now employed in a rural area. Which of the following differences between the populations is the nurse most likely to notice? a. Increased number of residents under age 18 b. Increased number of residents over age 50 c. Increased number of residents who are married d. Increased number of residents who have more years of formal education

ANS: A Rural communities have a higher-than-average number of residents under 18 years of age and over 65 years of age compared with an urban setting. Rural residents 18 years of age and older are more likely to be, or have been, married than their urban counterparts. As a group, rural adults are more likely to be widowed and have fewer years of formal education than urban adults.

A nurse is practicing in a rural environment. Which of the following opportunities exist for in the nurse's practice? a. Variety/diversity in clinical experiences b. Slower pace c. Direct collaborative practice opportunities d. Abundant resources

ANS: A Rural nursing offers a great variety and diversity of clinical experiences because care is often given across the life span. Typically in nursing practice in rural areas, it is a faster pace, has greater independence/autonomy, and sparse resources. In rural areas there are typically less providers, so there may be more indirect collaborative practice opportunities but more limited direct collaborative practice.

A nurse analyzes data from minutes at a community meeting when completing a community assessment. Which of the following methods of data collection is being used? a. Secondary data b. Informant interviews c. Primary data d. Windshield survey

ANS: A Secondary sources include published data about the community. The data is secondary because it is collected by someone else. An informant interview is a method of community data collection that involves directed conversation with selected community members. Primary data includes using informant interviews, focus groups, and participant observation in order to collect information about a community. Windshield surveys are a method of simple observation, providing a quick overview of a

A nurse is polite and non-confrontational when working with the family of a migrant worker. Which of the following concepts is the nurse demonstrating? a. Simpatía b. Respeto c. Dignidad d. Personalismo

ANS: A Simpatía means having polite, non-confrontational relationships with others. Respeto means respect, dignidad means dignity, and personalismo is relating to the individual.

A nurse is providing a skilled nursing service. Which of the following best describes the intervention the nurse is performing? a. Teaching the client and family about medication administration b. Consulting with other nurses about the care of a client in the home c. Coordinating services for maximum benefit at minimal cost d. Reviewing nursing assistant charting at the end of each day

ANS: A Skilled nursing service is the Medicare term that describes the duties of the registered nurse, and refers to the requirement of nursing judgment. Those services involve assessment, teaching, and selected procedures. Teaching the client and family to implement a therapeutic plan, such as taking medications, is an example of a skilled nursing service. Consultation, coordination, and review of documentation do not fit the definition of a "skilled nursing service."

Which is a disease that was once isolated and rare but is now widespread throughout the world? a. Acquired immunodeficiency syndrome (AIDS) b. Smallpox c. Malaria d. Measles

ANS: A Smallpox has been eradicated; malaria and measles were not isolated and rare throughout the world. AIDS was once isolated and rare, but is now worldwide.

The nurse labels a patient an alcoholic because of his ethnicity. Which of the following best describes this action by the nurse? a. Stereotyping b. Prejudice c. Racism d. Ethnocentrism

ANS: A Stereotyping is ascribing certain beliefs and behaviors about a given racial and ethnic group to an individual without assessing for individual differences. Prejudice is the emotional manifestation of deeply held beliefs about a group. Racism is a form of prejudice that occurs through the exercise of power by individuals and institutions against people who are judged to be inferior in, for example, intelligence, morals, beauty, and self-worth. Ethnocentrism is the belief that one's own cultural group determines the standards by which another group's behavior is judged.

A nurse is caring for adolescents who have experienced substance abuse. Which of the following should the nurse recognize as a major influence in this population? a. Family-related factors b. Positive media messages c. Socioeconomic level d. Peer pressure

ANS: A Substance abuse among adolescents is influenced most by family-related factors, such as genetics, family stress, and parenting styles. Research suggests that successful social influence-based prevention programs may be driven by their ability to foster social norms that reduce an adolescent's social motivation to begin using ATODs. Peer pressure and socioeconomic level may play a role in the abuse of substances, but this has not been the major factor identified. Media messages promoting the use of substances may influence this population, but is not the major factor that has been identified.

A nurse is presenting information to the county health department about potential bioterrorism threats. Which of the following agents would the nurse discuss in this presentation? a. Smallpox b. West Nile Virus c. Severe acute respiratory syndrome (SARS) d. Novel influenza A (H1N1)

ANS: A Susceptibility to smallpox is 100% in the unvaccinated and fatality rate is estimated at 20-40% or higher. The agents of highest concern are anthrax, plague, smallpox, botulism, tularemia, and selected hemorrhagic viruses. West Nile Virus, SARS, and H1N1 are not viruses that would be used for bioterrorism.

An intervention that focuses on the tertiary level of prevention is implemented by the nurse? Which of the following did the nurse most likely complete? a. Rehabilitative job training b. Parenting education c. Testicular self-examination d. Family counseling

ANS: A Tertiary prevention includes those interventions aimed at disability limitation and rehabilitation from disease, injury, or disability. Rehabilitative job training would be an example of tertiary prevention. Parenting education is an example of primary prevention. Testicular self-examination and family counseling are examples of secondary prevention.

A public health department is using the mission of public health as described by the Institute of Medicine when planning its health programming. Which of the following activities will most likely be implemented? a. Tracking avian flu outbreaks and doing surveillance in the United States b. Providing a flu shot for an elderly person at the health department c. Keeping track of alternative therapies in use in the United States d. Keeping snake antivenom at the Centers for Disease Control and Prevention in Atlanta

ANS: A The Institute of Medicine's stated mission on public health is "to generate organized community and technical knowledge to prevent disease and promote health." Tracking avian flu outbreaks and doing surveillance applies this concept at a population level. Providing a flu shot for an elderly person only addresses individual care. Keeping track of the use of alternative therapies does nothing to prevent disease or promote health of the population. Keeping snake antivenom is aimed at disease care for an individual, not health promotion or disease prevention.

The intergovernmental organization that focuses its efforts on assisting countries in Latin America is the: a. Pan American Health Organization. b. World Bank. c. World Health Organization (WHO). d. United Nations Children's Fund.

ANS: A The Pan American Health Organization serves as a regional field office in Latin America for the WHO. The World Bank's major aim is to lend money to the lesser-developed countries so that they might use it to improve the health status of their people. The WHO addresses the wide scope and nature of the world's health problems. The United Nations Children's Fund is an advocate for the health needs of women and children under the age of 5.

A nurse is working with a family who is unable to find adequate health care coverage for the children in the family. Which of the following pieces of legislation will assist in provision of health care coverage for these children? a. Social Security Act b. Balanced Budget Act c. HIPAA d. ACA

ANS: A The Social Security Act provides funds to insure currently uninsured children. The Balanced Budget Act shifted payment in home health care. The HIPAA was intended to help people keep their health insurance when moving from one place to another. The ACA of 2010 provides the opportunity for all to purchase health insurance.

Which federal program created support for older and poor Americans? a. Social Security Act b. Medicare Amendment c. Medicaid Amendment d. Hill-Burton Act

ANS: A The Social Security Act sought direct payments to eligible individuals to ensure a minimum level of support for people at risk of problems from inadequate financial resources. The Social Security Act encompasses the Medicare and Medicaid Amendment. The Hill-Burton Act provided funding to assist with building hospitals.

A government agency is planning a project in hopes of receiving funding from the World Bank. Which of the following projects would most likely receive this funding? a. Development of a sanitation system b. Improvement of roads and bridges c. Creation of a national health insurance plan d. Recovery efforts following a hurricane

ANS: A The World Bank lends money to lesser-developed countries so that they might use it to improve the health status of their people. Projects include control and eradication of disease, providing safe drinking water and affordable housing, developing sanitation systems, and encouraging family planning and childhood immunizations. Improvement of roads and bridges, creation of a national health insurance plan, and recovery efforts following a hurricane would not be projects where World Bank funds would be used.

A nurse is planning care using the case management model. Which of the following interventions would be most appropriate when implementing this model of care? a. Providing a client-professional partnership b. Delivering accessible care to medically underserved areas c. Defining the characteristics of a community d. Evaluating the outcomes of rural programs

ANS: A The case management model provides a client-professional partnership that can be used to arrange a continuum of care for rural clients, with the case manager tailoring and blending formal and informal resources. Collaborative efforts between a client and the case manager allow clients to participate in their plan of care in an acceptable and appropriate way, especially when local resources are few and far between. The case management model does not define the characteristics of the community or evaluate the outcomes of rural programs.

A public health nurse (PHN) is working with the chronically ill and families with young children needing age-specific health maintenance. Which of the following models of case management is being used? a. Client-focused b. System-focused c. Social service d. Long-term care

ANS: A The concern in client-focused models is with the relationship between case manager and client to support continuity of care and to access providers of care. System focused models address the structure and processes of using the population-based tools of disease management and case management plans to offer care for client populations. The social service models provide services to clients to assist them in living independently in the community and in maintaining their health by eliminating or reducing the need for hospital admissions or long-term care. Long-term care is not a model of case management.

A nurse is using the first stage of the Outcomes and Assessment Information Set (OASIS) outcomes analysis. Which of the following best describes what the nurse is doing? a. Comparing an agency's performance to a national sample b. Reporting data on patient care to a national registry c. Stabilizing a client's condition to continue to receive funding d. Documenting client assessment data and findings

ANS: A The first stage, outcomes analysis, enables an agency to compare its performance to a national sample, identify factors that may affect outcomes, and identify final outcomes that show improvement in or stabilization of a client's condition. The second stage, outcomes enhancement, enables the agency to select specific client outcomes and determine strategies to improve care.

A nurse has identified the boundaries of the community. Which of the following steps should the nurse take next when completing a community assessment? a. Gather relevant existing data and generating missing data b. Plan interventions that benefit the entire community c. Formulate nursing diagnoses d. Evaluate the interventions that were used

ANS: A The first step of the community assessment is to define the community. In order to do this, geographic boundaries, the population within the boundaries, the purpose of the assessment, and a data collection plan will be identified. Those inhabiting the community are a primary focus of the assessment. A variety of strategies are used to identify this central core of the community. The next step involves gathering and generating data to learn more about the community. After the community has been completely assessed then nursing diagnoses can be formulated, interventions planned, and finally interventions evaluated.

A nurse has referred a home care client to hospice care. Which of the following best explains the rationale for this referral? a. To provide comfort and peace at a difficult time b. To reduce the cost of care for the terminally ill c. To keep clients at home until the end of life d. To provide care for terminally ill clients expected to live longer than 6 months

ANS: A The focus of hospice care is comfort, peace, and a sense of dignity at a very difficult time. Comprehensive services emphasize continuity of care. Hospice care is provided for those with a life expectancy of less than 6 months. Hospice is the only Medicare benefit that includes medications, medical equipment, 24 hours/7 days a week access to care, and support for family members after death. Hospice care does not reduce the cost of care.

A nurse is implementing the harm reduction model. Which of the following will be the focus of nurse when using this approach? a. Education b. Law enforcement c. Scare tactics d. Rehabilitation

ANS: A The harm reduction model accepts the reality that psychoactive drug use is endemic, and it focuses on pragmatic interventions, especially education, to reduce the adverse consequences of drug abuse and get treatment for addicts. The harm reduction model encourages nurses to seek the underlying roots of various health problems and plan action that is realistic, nonjudgmental, holistic, and positive. Law enforcement, scare tactics, and rehabilitation are not part of the harm reduction model

What was the purpose of the CDC creating the first list of standard case definitions for notifiable diseases? a. Some diseases were underreported and others were overreported. b. There was no central office to collect data. c. Some people refused to notify officials when diseases were seen. d. No one knew where to report the information that was collected.

ANS: A The initial work was done by the CDC because diseases were overreported or underreported. Before this time, state and local health departments used many different criteria for identifying cases of reportable diseases. There was a central office to collect the data and providers knew that this information needed to be reported, but what was being reported by departments was different.

The main characteristics of partnership are: a. awareness, flexibility, and distribution of power. b. rights, responsibilities, and consensus. c. commitment, participation, and articulation. d. collaboration, advocacy, and utility.

ANS: A The main characteristics of partnership are awareness, flexibility, and negotiated distribution of power. Rights, responsibilities, and consensus are not the main characteristics of partnership. Commitment, participation, and articulation are not the main characteristics of partnership. Collaboration, advocacy, and utility are not the main characteristics of partnership.

A nurse is providing care for a Mexican client who has sustained a farm injury. If a major decision about the client's treatment needs to be made, who would most likely make that decision? a. The male head of the family b. The wife/mother c. The person needing services d. The crew leader

ANS: A The male is considered to be the major decision maker in the family, whereas the female is considered the caretaker.

A community health nurse collects data about the number and proportion of persons aged 25 or older with less than a high school education. Which of the following best describes this data? a. Sociodemographic characteristics b. Health status data c. Health risk factors d. Health care resource consumption information

ANS: A The number and proportion of people is a sociodemographic characteristic. Health status includes birth and death rates. Health risk factors discuss proportions of populations who have particular health conditions or health risks. Health care resource consumption refers to spending on health care per capita.

A nurse has collected a urine specimen from a client for drug testing. Which of the following best describes the purpose of a urinalysis? a. To assess whether a certain drug has been used in the recent past b. To determine the degree of intoxication c. To assess the extent of performance impairment d. To determine how much alcohol was consumed

ANS: A The nurse collects a urine specimen with the rationale that urinalysis for drug testing can be used to determine whether a certain drug has been used in the recent past. Urine testing indicates only past use of certain drugs, not intoxication. The extent of performance impairment cannot be determined with urine testing. Alcohol is not always included in a urine drug screen.

Which statement about the nursing workforce is true? a. The nursing workforce is overwhelmingly white. b. The number of minority nurses meets the needs of the country's demographics. c. The nursing workforce is becoming increasingly diverse with increasing minorities. d. Refugees make up a large part of the nursing workforce in some areas.

ANS: A The nursing workforce is 83% white. Because of the large percentage of white nurses this does not meet the needs of the country's demographics. Unauthorized immigrants and refugees do not make up a part of the nursing workforce.

What is implied by the web of causation model? a. Variables interact resulting in higher probability of illness. b. One disease causes another, especially in vulnerable populations. c. The greater the poverty, the more likely people are to have diseases. d. Immunizations are necessary because vulnerable populations spread disease.

ANS: A The web of causation model implies that not only are there more variables, but the variables interact, resulting in higher probability of illness. The relative risk for poor health is greater for vulnerable populations.

The process of moving conflicting parties toward an outcome is called: a. negotiation. b. conflict management. c. problem-purpose expansion method. d. brainstorming.

ANS: A The process of moving conflicting parties toward an outcome is called negotiation. Conflict management has the goal of mutual benefit with limited loss, which is directed toward getting all parties to work together. The problem-purpose-expansion method is a way to broaden limited thinking by restating the problem and expanding the problem statement so that different solutions can be generated. In brainstorming, as many alternatives as possible are generated without placing a value on them.

A nurse focuses intervention strategies on the structural dimension of community health. Which of the following best describes the focus of these strategies? a. Health services b. Primary prevention c. Health promotion d. Secondary prevention

ANS: A The structure of the community is defined in terms of services and resources. The subsystems of community structure consist of physical environment, health and social services, economy, transportation and safety, politics and government, communication, education, and recreation. Secondary prevention focuses on early detection and prompt treatment of disease, injury, or disability. Primary prevention attempts to promote health and provide specific protection from disease. Health promotion activities are interventions focused on improving one's health.

A nurse is triaging victims following a disaster. Which of the following describes the action the nurse would take? a. Allocation of treatment based on the victim's potential for survival b. Assignment of tasks to the appropriate disaster response personnel c. Determination of the level of disaster and agency involvement d. Participation in community preparedness for the disaster response

ANS: A The triage process in disaster involves separating the casualties and allocating treatment based on the victim's potential for survival. Assignment of tasks to the appropriate disaster response personnel, determination of the level of disaster and agency involvement, and participation in community preparedness for the disaster response are not part of the triage process.

A nurse is working from a client who is part of the Mexican culture and is experiencing a folk illness. Which of the following actions by the client should the nurse anticipate? a. Prefer to seek care with a folk healer b. Consult the female leader of the family c. Rely on Western medicine to cure the ailment d. Become isolated and not seek care

ANS: A There are four common folk illnesses that a nurse may encounter with the Mexican client: (1) mal de ojo (evil eye), (2) susto (fright), (3) empacho (indigestion), and (4) caida de mollera (fallen fontanel). When faced with one of these illnesses, the traditional Mexican individual would prefer to seek care with a folk healer. Healers known as curanderos, herbalistas, and espiritualistas are often sources of health care for folk illnesses.

One of the primary focuses of improving the health of the American people in the twenty-first century is to address: a. bioterrorism and global health threats. b. delivery of individual care and hygiene. c. the need for increased hospital and acute care. d. chronic disease and disability management.

ANS: A There are new concerns, and of the most serious are bioterrorism and globally induced infections, such as the avian flu. These threats will divert health care funds and resources from other health care programs to be spent for public safety. The others are not related to public health or are concerns that have been present for many years.

A nurse is providing care using the idea of "servicing citizens, not customers." Which of the following ideas is being applied by the nurse? a. Ethical tenets of policy development b. Basic concepts of the feminist theory c. Underlying premise of virtue ethics d. Components of distributive justice

ANS: A There are three tenets of both policy and ethics. The approach is based on the voice of the community as the foundation on which policy is developed. The basic concept of feminist theory allows us to think critically about connections among gender, disadvantage, and health as well as the distribution of power in public health processes. The goal of virtue ethics is to enable persons to flourish as human beings. Distributive justice requires that there be a fair distribution of the benefits and burdens in society based on the needs and contributions of its members.

A nurse is implementing quality performance standards in a public health department. Which of the following best describes the importance of this action? a. Quality performance standards are used to guide improvement in the public health system. b. Quality performance standards rigidly control public health. c. Quality performance standards guide administrators to monitor public health at the national level. d. Quality performance standards can be used as hiring guidelines for nurses.

ANS: A These standards were developed by the Centers for Disease Control and Prevention in 1998. They set the bar for the level of performance that is necessary to deliver essential public health services (PHSs). These standards were developed around the 10 essential PHSs. They focus on the overall public health system rather than on single organizations and describe an optimal level of performance. They are intended to support a process of quality improvement. State and local communities can assess their performance using certain available tools. They are not meant to control public health or be used for hiring guidelines.

According to Leininger and Watson, the moral ideal of nursing is: a. caring. b. advocacy. c. responsibility. d. accountability.

ANS: A This conceptualization occurred as a response to the technological advances in health care science and the desire of nurses to differentiate nursing practice from medical practice. Advocacy, responsibility, and accountability are not part of the moral idea of nursing proposed by Leininger and Watson.

A Mexican American mother insists on using special candles to help her daughter's ear infection. The nurse asks the mother if she would also give her daughter antibiotics. Which of the following best describes the action of the nurse? a. Cultural accommodation b. Cultural repatterning c. Culture brokering d. Cultural awareness

ANS: A This means that the nurse supports and facilitates the use of cultural practices with interventions from the biomedical health care system. Cultural accommodation refers to assistive, supportive, facilitative, or enabling nurse actions and decisions that help clients of a particular culture accept nursing strategies or negotiate with nurses to achieve satisfying health care outcomes. Cultural repatterning refers to assistive, supportive, facilitative, or enabling nurse actions and decisions that help clients of a particular culture to change or modify a cultural practice for new or different health care patterns that are meaningful, satisfying, and beneficial. Culture brokering is advocating, mediating, negotiating, and intervening between the client's culture and the biomedical health care culture on behalf of clients. Cultural awareness refers to the self-examination and in-depth exploration of one's own biases, stereotypes, and prejudices as they influence behavior toward other cultural groups.

The community health nurse is caring for a client who is living in persistent poverty. Which of the following best describes this client? a. Severe mental illness b. Transient homelessness c. Victim of domestic violence d. Temporary housing

ANS: A Those who are in persistent poverty typically are older, with physical and mental disabilities, alcohol and other drug abuse, severe mental illness, chronic health problems, and significant family difficulties. Transient homeless persons, victims of domestic violence, and persons with temporary housing would be considered in crisis poverty.

The basic science applied to understanding the health effects associated with chemical exposures is: a. toxicology. b. pharmacology. c. chemistry. d. environmental epidemiology.

ANS: A Toxicology is the study of the health effects associated with chemical exposures. Pharmacology is the branch of medicine concerned with the multiple aspects of drugs. Chemistry is the branch of science with deals with the way that matter is composed. Environmental epidemiology is concerned with the discovery of environmental exposures that cause or protect against illness or disease.

The nurse is caring for an elderly client who has written advanced medical directives. Which of the following best describes what this means? a. The client has made decisions regarding treatment if he becomes incapacitated. b. The client has a living will. c. The client has a Do Not Resuscitate order. d. The client's family has decided on a medical treatment regimen.

ANS: A Two parts of advance directives are the living will and a durable power of attorney. An advance directive indicates treatment choices if clients become incapacitated. A living will allows the client to express the wishes regarding the use of medical treatments in the event of a terminal illness. A Do Not Resuscitate order is a specific order from a physician not to use cardiopulmonary resuscitation.

A large portion of foreign-born residents of the United States: a. work in service-producing and blue-collar sectors. b. reside in rural areas. c. have a high school education. d. are refugees and nonimmigrants.

ANS: A Two-thirds of the foreign-born population lives in or around major metropolitan areas and works in mainly service-producing and blue-collar sectors. They are also more likely to be poorer. The majority of foreign born are legal immigrants (85%). More than 54% of the foreign-born adults in the labor force have completed high school, which would not be considered a large portion.

A nurse is implementing Wagner's Chronic Care Model (CCM). Which of the following actions would most likely be taken by the nurse? a. Educate a community group about hypertension control b. Create a budget for chronic disease management c. Administer immunizations to community members d. Conduct depression screenings in the community

ANS: A Use of electronic health records, provider reminders for key evidence-based care components, interprofessional teams communicating regularly, and community health classes to educate people with chronic diseases are various ways the CCM is being implemented. Creating a budget is not a way to use CCM. Administration of immunizations and conducting depression screenings do not address the management of the most common and costly chronic diseases: heart disease, diabetes, stroke, cancer, and arthritis.

A nurse is teaching members of the community about vertical transmission of a disease. Which of the following would the nurse most likely discuss? a. Transmission through breast milk b. Transmission through sexual contact c. Transmission from mosquito bites d. Transmission through contaminated food

ANS: A Vertical transmission is the passing of infection from parent to offspring via sperm, placenta, milk, or contact in the vaginal canal at birth. Transmission through sexual contact is horizontal transmission. Transmission from mosquito bites is vector transmission. Transmission from contaminated food is common vehicle transmission.

Which of the following is a characteristic of a vulnerable population? a. Have worse health outcomes and an increased sensitivity to risk factors than the general population b. Have a single risk factor but experience worse health outcomes than the general population c. Have multiple risk factors but equal health outcomes to the general population d. Have worse outcomes with better access to health care than the general population

ANS: A Vulnerable population groups are more sensitive to risk factors and have worse health outcomes. Vulnerable populations experience multiple risk factors. Vulnerable populations have worse health outcomes than the general population. Vulnerable populations have more problems accessing health care than the general population.

A nurse lobbies with legislators for the passage of legislation to prohibit smoking of tobacco products in public places. Which of the following levels of prevention is being implemented? a. Primary b. Secondary c. Tertiary d. Health promotion

ANS: A When using primary prevention, the focus is on health promotion and disease prevention. When using tertiary prevention, the nurse focuses on treatment and rehabilitation. Secondary prevention focuses on early detection and screening. Health promotion is not a level of prevention.

The public health workforce should demonstrate competency in which of the following competency categories? a. Financial planning and management b. Workforce needs assessment c. Acute care services d. Curriculum development

ANS: A Workforce needs assessment, acute care services, and curriculum developments are not considered categories of core competencies of public health. Rather there are eight categories of competency, which include: (1) analytic/assessment, (2) policy development/program planning, (3) communication, (4) cultural competence, (5) community dimensions of practice, (6) basic public health sciences, (7) financial planning and management, and (8) leadership and systems thinking.

Which organizations offer volunteer opportunities in disaster work for nurses? (Select all that apply.) a. ARC b. Citizens Corps c. American Nurses Association d. National Public Health Training Centers e. Sigma Theta Tau International

ANS: A, B ARC and Citizens Corps offer volunteer opportunities. The American Nurses Association and National Public Health Training Centers offer education and training opportunities. Sigma Theta Tau International is an honor society for nurses.

A nurse is caring for an elderly female population. Which of the following considerations should be made? (Select all that apply.) a. Postmenopausal women are more likely than men to experience hypertension. b. Women use more health services than men. c. Women report lower rates of disability than men. d. Women and men have similar socialization patterns. e. Postmenopausal women are less likely than men to experience depression.

ANS: A, B Postmenopausal women are more likely than men to experience hypertension; prior to menopause, the rate is higher among men. Women use more health services than men. Women report higher rates of disability than men. Women and men have different socialization, expectations, and lifestyles.

A nurse would like to take action to reduce the risk of liability. Which of the following interventions should be used by the nurse? (Select all that apply.) a. Provide accurate documentation of client visits b. Inform clients of their rights of appeal c. Refer clients to the providers of their choice d. Utilize care planning whenever possible e. Promote strong community partnerships

ANS: A, B To reduce risk exposure, it is important to provide clear documentation of client encounters and inform clients of their rights of appeal. Client choice is important when choosing a provider; however, the nurse is responsible for using reasonable care when selecting referral choices for a client. Although care planning may be helpful for establishing a plan with the client, it is not necessary in reducing the risk of liability.

Which of the following are considered barriers to public health nursing? (Select all that apply.) a. The mindset that the only role for the nurse is at the bedside b. The structures within which nurses work and the process of role socialization within those structures c. Few nurses receive graduate-level preparation in the concepts and strategies of the disciplines basic to public health d. The steady decline in the number of job opportunities in the area e. The inability to equally distribute the increasing funds

ANS: A, B, C Barriers exist in thinking that nurses only work in a hospital at the bedside, the process of role socialization in public health which can be a very autonomous profession, and that few nurses have had advanced education in public health. The number of job opportunities continues to increase, but the amount of funding allotted to public health continues to decline.

A nurse is planning interventions that will be used when providing care for a rural client. Which of the following characteristics of rural life should the nurse anticipate? (Select all that apply.) a. Lack of anonymity b. Higher prevalence of high-risk occupations c. Churches and schools as socialization centers d. Preference for interacting with outsiders e. Increased mobility

ANS: A, B, C Characteristics of rural life include lack of anonymity, higher prevalence of high-risk occupations, churches and schools as socialization centers, and preference for interacting with members of own community. Those living in a rural community typically have decreased mobility.

A nurse is defining the community as part of the community assessment process. Which of the following best describes how the community may be defined? (Select all that apply.) a. Social group determined by geographic boundaries b. Group of people who share common values and interests c. Group of people defined by their interactions d. Individual with a specific health concern e. Individuals with certain interests

ANS: A, B, C Community can be defined as many things, including a social group determined by geographical boundaries, a group of people who share common values and interests, and a group of people defined by their interactions. Individuals comprise a community, but would not be an accurate definition of a community.

A nurse is using public surveillance. Which of the following describes the actions involved? (Select all that apply.) a. Estimating the magnitude of an influenza outbreak b. Determining the geographical distribution of gonorrhea cases c. Detecting an epidemic of whooping cough d. Reducing the prevalence of obesity e. Determining the incidence of cardiovascular disease

ANS: A, B, C Estimating the magnitude of the problem, determining geographic distribution of an illness, and detecting epidemics are all uses of public surveillance. Reduction of the prevalence of obesity and determining the incidence of cardiovascular disease are not how public health surveillance is used.

A nurse "sets the stage" when assessing members of vulnerable population groups. Which of the following interventions would be completed? (Select all that apply.) a. Creating a comfortable, non-threatening environment b. Providing culturally and linguistically competent assessment c. Collaborating with others as appropriate d. Providing financial and legal advice e. Developing a free clinic in a low-income neighborhood

ANS: A, B, C Nurses set the stage by creating a comfortable, non-threatening environment, providing culturally and linguistically competent assessment, and collaborating with others as appropriate. A nurse should not provide financial or legal advice. Developing a free clinic does not "set the stage."

Which of the following factors are cited as obstacles to improving men's health? (Select all that apply.) a. Reluctance to consult their primary care provider b. Emphasis of physical strength and competitiveness on masculinity c. Lack of a primary care provider d. Lack of access to health care by African American and Hispanic men e. Emphasis on the male role as the head of the family

ANS: A, B, C Obstacles to improving men's health include reluctance to consult their primary care provider, the concept that masculinity emphasizes physical strength and competitiveness, and the fact that many men do not have a primary care provider.

A nurse is assisting with community planning. Which of the following facilities should the nurse encourage be developed to reduce the potential for crime and violence in the community? (Select all that apply.) a. Playgrounds b. Movie theaters c. Swimming pools d. Shopping malls e. Restaurants

ANS: A, B, C Recreational facilities including playgrounds, movie theaters, and swimming pools all provide socially acceptable outlets for a variety of feelings, including aggression. Shopping malls and restaurants would not be considered recreational facilities.

Which populations are at greatest risk for disruption after a disaster? (Select all that apply.) a. Single-parent families b. Children c. Substance abusers d. Middle-class families e. Young adults

ANS: A, B, C Single-parent families, children, and substance abusers are all at greater risk for disruption after disaster than a middle-class family or young adults.

A nurse is coordinating care through a high-intensity transitional care program. Which of the following best describes the populations that the nurse is working with? (Select all that apply.) a. Adults with cognitive impairments b. Women with high-risk pregnancies c. Older adults with heart failure d. People who are moving from one state to another e. Men who have just experienced divorce

ANS: A, B, C Transitional care ensures the coordination and continuity of health care as clients transfer between different locations and different levels of care in the same location. High-intensity transitional care programs are designed for populations who have complex or high-risk health problems.

Which illegal and unethical activities have occurred in health care systems in the past? (Select all that apply.) a. Inappropriate use of home health services b. Inaccurate billing for services c. Excessive administrative staff d. "Kickbacks" for referrals e. Primary care services provided in the home

ANS: A, B, C, D Examples of Medicare fraud and abuse in home health and hospice include inappropriate use of services, excessive payments to administrative staff or owners, "kickbacks" for referrals, and billing for visits and/or medical supplies that are not authorized or provided. The home care nurse must abide by established federal regulations when delivering care to clients.

A nurse is providing care for at-risk minority groups in rural America. Which of the following health care needs are appropriate for the nurse to address? (Select all that apply.) a. Field sanitation b. Infectious diseases c. Maternal care services d. Oral/dental care e. Chronic diseases

ANS: A, B, C, D Field sanitation, infectious diseases, maternal care, and oral/dental concerns are major health care needs for at-risk minority groups in rural America. Chronic diseases are a problem for all Americans, not just the at-risk minority groups in the rural areas.

A nurse is caring for a client who has a history of chronic alcohol abuse. Which of the following medical problems should the nurse anticipate? (Select all that apply.) a. Gastrointestinal disturbances b. Cardiac dysrhythmias c. Depression d. Organic brain syndrome e. Hepatitis B

ANS: A, B, C, D Gastrointestinal disturbances, cardiac dysrhythmias, depression, and organic brain syndrome are some effects of chronic alcohol abuse.

A community health nurse is applying the Ethical Principles for Effective Advocacy. Which of the following best describes the action(s) of the nurse? (Select all that apply.) a. Act in the health care provider's best interest b. Keep the client (group, community) properly informed c. Maintain client confidentiality d. Carry out instructions with diligence and competence e. Provide advice to all members of the community

ANS: A, B, C, D Keep the client (group, community) properly informed, maintain client confidentiality, and carry out instructions with diligence and competence are ethical principles for effective advocacy. The nurse should act impartially and offer frank, independent advice, which does not necessarily mean that advice should be provided to all members of the community.

Which factors are challenges of the migrant lifestyle? (Select all that apply.) a. Leaving one's home every year b. Traveling c. Experiencing uncertainty regarding work and housing d. Lack of resources e. Middle income wages

ANS: A, B, C, D Leaving one's home every year, traveling, experiencing uncertainty regarding work and housing, and lack of resources are some of the challenges of the migrant lifestyle. The specifics for payment differ depending on the location and type of work. Reports of average income for farmworkers have varied.

A nurse is providing care to a child who has been abused. Which characteristics are most likely present in this child's family? (Select all that apply.) a. The parents are unemployed. b. There is no support network willing to offer assistance. c. The parents are not knowledgeable about child development. d. The mother seems frightened of her partner. e. There is no primary caregiver in the home.

ANS: A, B, C, D Risk factors for child abuse include unemployed parents, no support network, parents not knowledgeable about child development, and a mother who is scared of her partner.

What knowledge and skills are required in order to become a competent case manager? (Select all that apply.) a. Knowledge of community resources and financing mechanisms b. Written and oral communication skills c. Proficient negotiation and conflict-resolving practices d. Application of evidence-based practices and outcomes measurements e. Experience with ethical decision

ANS: A, B, C, D The knowledge and skills required to achieve competency include knowledge of community resources and financing mechanisms, written and oral communication and documentation, proficient negotiating and conflict-resolving practices, and the ability to apply evidence-based practices and outcomes measures. Experience with ethical decision making is not part of becoming a competent case manager.

A nurse is analyzing the media images that influence society's beliefs about poor persons. Which of the following is the nurse most likely to see? (Select all that apply.) a. Poor persons are often cast as lazy, shiftless folk. b. Poor people are shown receiving food at soup kitchens. c. Criminals are portrayed as poor, desperate persons. d. Homeless people are seen sleeping on park benches. e. Criminals are seen as individuals who have been discriminated against.

ANS: A, B, C, D The media is a powerful influence on how people view poverty. Media images of persons who are poor influence and are influenced by cultural attitudes and values. Poor persons may be cast in negative ways by the media, which influences what we believe to be true about poor persons.

A staff nurse has demonstrated the ability to be competent as a participant in surveillance and investigation activities. Which of the following skills would you expect that nurse to possess? (Select all that apply.) a. The ability to identify appropriate data sources b. The ability to communicate effectively using written reports c. The ability to collaborate with community agencies d. The ability to create a budget e. The ability to interpret laboratory findings

ANS: A, B, C, D The staff nurse must have analytic assessment skills, communication skills, and community dimensions of practice to be a participant in surveillance and investigation. The ability to interpret laboratory findings is not a core competency.

Which elements of surveillance does a nurse use? (Select all that apply.) a. Mortality registration b. Epidemic field investigation c. Laboratory reporting d. Individual case investigation e. Application of research

ANS: A, B, C, D There are 10 basic elements of surveillance. Mortality registration, epidemic field investigation, laboratory reporting, and individual case investigation are among them. Application of research is not part of the 10 basic elements of surveillance.

Which trends are occurring when providing care for vulnerable populations? (Select all that apply.) a. Community-based care and interorganizational partnerships b. Outreach and case finding c. Elimination of disparities d. Culturally and linguistically appropriate care e. Increased incidence of acute illnesses

ANS: A, B, D Community-based care and interorganizational partnerships, outreach and case finding, and provision of culturally and linguistically appropriate care are all trends to improve care among vulnerable populations. Elimination of disparities is not a trend related to caring for vulnerable populations. Increased incidence of acute illness is not a trend of care provision for vulnerable populations.

A nurse is planning a refugee outreach clinic at the neighborhood health center. Which of the following considerations should be made by the nurse in order to provide culturally competent care? (Select all that apply.) a. Their own background, beliefs, and knowledge may be significantly different from those of the people receiving care. b. Language barriers may interfere with efforts to provide assistance. c. Certain risk factors may be present for a given population. d. Certain groups may use nontraditional healing practices. e. Introduction of the American culture will be an important part of the encounter.

ANS: A, B, C, D When working with immigrants, it is important to be aware of one's own beliefs while realizing that language barriers may exist, that different populations experience different risk factors and illnesses, and that nontraditional healing practices may be used. It would not be appropriate to impose the American culture into the encounter.

A nurse is preparing an educational program on cultural perspectives in nursing. The nurse should include that which of the following factors are influenced by an individual's culture? (select all that apply) A. nutritional practicies B. family structure C. Health care interactions D. biological variations E. views about illness

ANS: A, B, C, E

A PHN working with a family living in poverty is concerned about their exposure to environmental hazards. Which of the following factors should the nurse consider that may put the family at risk? (Select all that apply.) a. Limited funds to pay for health care b. Poor nutrition c. Homes located closer to hazardous waste sites d. Less education e. Unsafe working conditions

ANS: A, B, C, E Families living in poverty are more likely to experience environmental justice issues such as disproportionate environmental exposures. Substandard housing, living closer to hazardous waste sites, working in more hazardous jobs, poorer nutrition, and less access to quality health care all contribute to this issue. Although limited education is related to poverty, it is not discussed as causing an increase in environmental exposure.

A nurse is working to manage the public health aspects of a bioterrorist event. Which of the following does the nurse need to understand? (Select all that apply.) a. Medical management and public health measures b. The names and descriptions of the agent/disease c. The chemical features of the pathogens d. How to address mental health concerns e. The amount of risk that each type of event will have for the public

ANS: A, B, D Nurses need to understand the names of the pathogens and the disease they cause, the medical management and public health measures (so that accurate information can be disseminated), and how to help people cope with their feelings. The chemical features of the pathogens and the amount of risk that each type of event will have for the public would not be part of the nurses' role when managing the public health aspects of a bioterrorist event.

Which of the following are ethical tenets that underlie the core function of assessment? (Select all that apply.) a. Competency: the persons assigned to develop community knowledge are prepared to collect data on groups and populations b. Moral character: the persons selected to develop, assess, and disseminate community knowledge possess integrity c. Service to others over self: a necessary condition of what is "good" or "right" policy d. "Do no harm": disseminating appropriate information about groups and populations is morally necessary and sufficient e. Providers of public health services should be competent and available

ANS: A, B, D Service to others over self is an ethical tenet of policy development. Providers of public health services should be competent and available is an ethical tenet of assurance. Competency, moral character, and "do no harm" are the ethical tenets of assessment.

An occupational health nurse is consulting with senior management of a local industrial facility. When discussing work-related illness and injury, the nurse should include which of the following factors as physical agents? (select all that apply) A. noise B. age C. lighting D. viruses E. stress

ANS: A, C

Which of the following do the Millennium Development Goals address? (Select all that apply.) a. Reduction of child mortality b. Eradication of communicable diseases c. Promotion of gender equality and empowerment of women d. Achievement of universal health care access e. Promotion of specialty practice by physicians

ANS: A, C The Millennium Development Goals address reduction of child mortality and promotion of gender equality and empower women. Other goals include eradicating extreme poverty and hunger; achieving universal primary education; improving maternal health; combating HIV/AIDS, malaria, and other diseases; ensuring environmental sustainability; and developing a global partnership for development. Eradication of communicable diseases, achievement of universal health care access, and promotion of specialty practice by physicians are not part of the Millennium Development Goals.

A nurse is planning to implement Wagner's CCM. Which of the following essential elements should be considered by the nurse? (Select all that apply.) a. Clinical information systems b. Evidence-based change c. Self-management support d. Community e. Clinical practice guidelines

ANS: A, C, D The essential elements of CCM are: the community, the health system, self-management support, delivery system design, decision support, and clinical information systems. Evidence-based change concepts are found under each of the essential elements and help to foster productive interactions between clients and providers.

A nurse at a community clinic is conducting a well-child visit with a preschool-age child. The nurse should identify which of the following manifestations as possible indication of child neglect? (select all that apply) A. underweight B. healing spiral fracture of the arm C. genital irritation D. burns on the palms of the hands E. poor hygiene

ANS: A, E

A nurse is conducting health screenings at a statewide health fair and identifies several clients who require referral to a provider. Which of the following statements by a client indicates a barrier to accessing health care? A. "i dont dive, and my son is only available to take me places in the morning" B. "i cant take off during the day, and the local after-hours clinic is no longer in operation" C. "only one doctor in my town is a designated provider by my health maintenance organization" D. "i would like to schedule an appointment with the local doctor in my town who speaks Spanish and English"

ANS: B

A school nurse is planning health promotion and disease prevention activities for the upcoming school year. In which of the following situations is the nurse planning a secondary prevention strategy? A. placing posters with images of appropriate hand hygiene near restrooms B. routinely checking students for pediculosis throughout the year C. implementing age-appropriate injury prevention programs for each grade level D. working with a dietitian to determine carbohydrate counts for students who have diabetes mellitus

ANS: B

A nurse is unable to provide culturally competent care to a population within the community. Which of the following is an effect of this type of care? a. Enhanced communication b. Increased cost of health care c. Achievement of health indicators d. Improvement in client compliance

ANS: B Care that is not culturally competent may increase health care costs and decrease positive client outcomes. Communication is inhibited through care that is not culturally competent. It is not possible to achieve health indicators if culturally competent care is not given. Client compliance decreases if culturally competent care is not provided.

The necessary basic preparation for public health nursing is a(n) _____ in nursing. a. associate's degree b. baccalaureate degree c. master's degree d. Doctor of Nursing Practice degree

ANS: B A bachelor of science in nursing (BSN) is the necessary basic preparation to function as a beginning staff PHN. An associate's degree is not enough for basic preparation. A master's degree or Doctor of Nursing Practice degree would be part of preparation for advanced practice.

The nurse is interviewing a client who reports that he drinks alcohol when snorting cocaine. Which of the following best describes what is happening with this client? a. Drug addiction b. Polysubstance use c. Substance abuse d. Indiscriminate drug use

ANS: B A client who drinks alcohol when snorting cocaine is suffering from polysubstance abuse. Drug addiction is a pattern of abuse characterized by an overwhelming preoccupation with the use of a drug, securing its supply, and a high tendency to relapse if the drug is removed. Substance abuse is the use of any substance that threatens a person's health or impairs his or her social or economic functioning. Indiscriminate drug use refers to careless use of drugs.

A nurse is working with an older person following a disaster. Which of the following emotions of the client should be anticipated by the nurse? a. Anger b. Fear of loss of independence c. Violence d. Regression

ANS: B A common reaction of older persons experiencing disaster may be fear of loss of independence. Young children may respond with regression. The community in general may respond will feelings of anger, sorrow, guilt, and perceived blame for the disaster or the outcomes of the disaster. Violence should not be anticipated with any of the populations discussed in the text.

A nurse is developing a community program to prevent violence. Which of the following should be the first action taken by the nurse? a. Talk to people who are victims of violence for their opinion b. Identify factors that lead to violence c. Develop a plan of action to combat violence d. Evaluate current community programs

ANS: B A community's population can influence the potential for violence. The starting point is the identification of factors that lead to violence, such as poverty, urban crowding, and racial inequality. In addition, one's perceptions of the safety in a community can be influenced by racism and perceptions of criminality.

A nurse is working with a client whose principal employment is in agriculture on a seasonal basis and has established temporary housing in the area while employed. Which of the following terms best describes this laborer? a. Seasonal farm worker b. Migrant farm worker c. Transient worker d. Share cropper

ANS: B A migrant farm worker is a laborer whose principal employment is in agriculture on a seasonal basis and, for the purposes of such employment, establishes a temporary abode. A seasonal farmworker returns to his permanent residence, works in agriculture for at least 25 days or parts of days, and does not work year round only in agriculture. A transient worker moves from place to place to find work. A share cropper is a tenant farmer who uses part of the crop to pay for rent.

The SARS (severe acute respiratory syndrome) outbreak was an epidemic that spread over several countries. Which of the following terms describes what happened? a. Hyperendemic b. Pandemic c. Mixed outbreak d. Holoendemic

ANS: B A pandemic refers to the epidemic spread of the problem over several countries or continents. If a problem is considered hyperendemic, there is a persistently (usually) high number of cases. A mixed outbreak is a common source followed by secondary exposures related to person-to-person contact. Holoendemic implies a highly prevalent problem found in a population commonly acquired early in life.

A nurse is working with multiple vulnerable groups. Which of the following would be most sensitive to adverse effects? a. Pregnant teenager living with her parents for financial support b. Poor, older woman with no means of transportation c. 2-year-old boy of underinsured parents d. Recently unemployed father of five

ANS: B A poor, older woman with no means of transportation has a combination of risk factors. A pregnant teenager, 2-year-old boy, and recently unemployed father are only displaying one risk factor, not multiple risk factors as the older woman displays.

A set of actions one undertakes on behalf of another is: a. social justice. b. advocacy. c. resilience. d. risk.

ANS: B A set of actions one undertakes on behalf of another is advocacy. Social justice describes justice with respect to the concepts of egalitarianism and equality. Resilience refers to how vulnerable populations are able to resist or overcome the effects of the vulnerability. Risk describes that some people have a higher probability of illness than others.

A PHN has been prepared at the graduate level. Which of the following activities should the practitioner be able to complete? a. Teach public and community health nursing b. Assess and intervene successfully at the aggregate level c. Diagnose and treat disease and have prescriptive authority d. Run for political office as experts in public health policy

ANS: B According to the Consensus Conference, specialists should have assessment skills that allow them to intervene at the aggregate level. Public health practitioners have a broad range of practice areas within public health and are not limited to only nursing and medicine.

The community leaders in a lesser-developed country decide not to tell the citizens of a small village about a chemical spill at a major industrial facility that could produce harmful effects. Which of the following principles is being violated? a. Morality b. Advocacy c. Caring d. Virtue

ANS: B Advocacy requires that the community be properly informed, and this was violated in the above scenario. Morality is shared and generational societal norms about what constitutes right or wrong conduct. Caring represents the essence of nursing. Virtue demonstrates behavior showing high moral standards.

Population-focused practice focuses on defining the problems or needs of and implementing solutions for: a. individuals. b. aggregates. c. communities. d. geographical regions.

ANS: B Aggregates are a defined population made up of individuals in communities of a specific geographical region. Population-focused practice is implemented with defined populations or subpopulations. This is larger than the scope of individual care, but may not address the larger needs of the entire community or geographical region.

A nurse is studying the characteristics of an agent as part of the epidemiologic triangle. Which of the following is the nurse most likely studying? a. Human population distribution b. Salmonella c. Genetic susceptibility d. Climate

ANS: B An agent includes infectious organisms, such as Salmonella, chemical agents, and physical agents. Genetic susceptibility is considered to be a characteristic of a host. Human population distribution and climate are considered to be characteristics of the environment.

Which factor influences the growing number of poor persons in the United States? a. Increase in company earnings b. Changes in the labor force c. Better access to quality education d. Adequate welfare benefits

ANS: B Changes in the labor force affect the growing number of poor persons in the United States. Decreased company earnings, decreased access to education and job skills, and inadequate welfare benefits also influence the growing number of poor persons.

A nurse is investigating the role of the agent in the cause of an illness. Which of the following best describes what the nurse is examining? a. Host resilience b. Virus c. Infectiousness d. Bug bite

ANS: B An agent is described by its ability to cause disease and the nature and the severity of the disease. The four major categories of agents are: (1) bacteria, (2) parasites, (3) fungi, and (4) viruses. Host resilience is a host factor. Infectiousness is the measure of the potential ability of an infected host to transmit the infection to other hosts. Environmental factors facilitate the transmission of an infectious agent from an infected host to other susceptible hosts, such as a bug bite.

An epidemiologist wants to know what caused severe diarrhea and vomiting in several people at a local banquet. Which of the following principles is being applied in this situation? a. Descriptive epidemiology b. Analytic epidemiology c. Distribution d. Determinants

ANS: B Analytic epidemiology is directed toward understanding the etiology of the disease. Descriptive epidemiology seeks to describe the occurrence of a disease in terms of person, place, and time. Distribution describes who has the disease and where and when the disease occurs. Determinants are the factors, exposures, characteristics, and behaviors that determine patterns of disease, which may be individual, relational, social, communal, or environmental.

A nurse is volunteering on a disaster medical assistant team as a first responder. Which of the following tasks would the nurse be prepared to perform? a. Set up immunization clinics b. Assist in triaging disaster victims c. Provide all the medical care for disaster victims d. Complete a needs assessment of the community

ANS: B As a first responder arriving on the scene, the plans for triaging of disaster victims should begin immediately. Setting up immunization clinics would not be a priority following a disaster. In responding to a disaster, it may be impossible to provide all of the medical care for disaster victims. It would not be appropriate to start with completing a needs assessment of the community following a disaster; this should be done before the disaster occurs.

The PHN compares the rate of teenage pregnancy in various areas of the city. Which of the core functions of public health is being implemented? a. Assurance b. Assessment c. Prevention d. Policy development

ANS: B Assessment refers to systematic data collection, which this nurse is doing for teenage pregnancy. Prevention is not a core function, assurance is making sure essential services are available, and policy development is needed to provide leadership in developing policies.

A nurse believes that all Americans should receive basic health care services. Which of the following core functions supports this belief? a. Assessment b. Assurance c. Policy development d. Advocacy

ANS: B Assurance purports that all persons should receive essential personal health services. Assessment refers to systematically collecting data on the population, monitoring the population's health status, and making information available about the health of the community. Policy development refers to the need to provide leadership in developing policies that support the health of the population, including the use of the scientific knowledge base in making decisions about policy. Advocacy embodies an ethical focus grounded in quality of life.

Which factor may limit access to care for migrant farm workers? a. Unwillingness to seek affordable health care b. Lack of knowledge regarding services c. Availability of 24-hour health care services d. Adequate transportation

ANS: B Because of isolation, migrant farm workers lack usual sources of information regarding available services. Migrant workers typically are unable to afford health care, have limited services available, and may not have adequate transportation to get to health care services.

A nurse is educating migrant women to serve as links between the existing health care agencies and the migrant farm worker family. Which of the following best describes the role of these women? a. Migrant head start workers b. Outreach care workers c. Physician assistants d. Nurse aides

ANS: B Because these outreach workers are members of the migrant community, they are trusted and know the culture and the language. Physician assistants and nurse aides do not serve in this lay person role in the community.

A nurse's excessive attention to cost containment impairs the nurse's duty to provide measures to improve health. Which of the following ethical principles is being influenced? a. Autonomy b. Beneficence c. Veracity d. Nonmaleficence

ANS: B Beneficence is influenced when excessive attention to cost containment supersedes or impairs the nurse's duty to provide measures to improve health or relieve suffering. Autonomy is the individual's right to choose a provider. Veracity is truth telling which is important to building trusting relationships with clients. Nonmaleficence is doing no harm, which is addressed when incorporating outcomes measures, evidence-based practice, and monitoring processes in plans of care.

A nurse is trying to break down the barriers between the health care professionals and people who live in poverty. Which of the following actions should be taken by the nurse? a. Maintain a strict professional demeanor b. Call the client by name c. Keep client appointments as scheduled d. Smile and be friendly to clients

ANS: B By calling the client by name, nurses can then begin the process of breaking down barriers of fear, isolation, uncertainty, and the unknown. It is important for the nurse to be a good listener, individualize care, and avoid making inappropriate assumptions about their needs. It is important to listen to the stories of their lives as well as examine social and cultural definitions and considerations related to poverty.

An enduring process in which a manager establishes systems and monitors the health status, resources, and outcomes for a targeted aggregate of the population is called: a. case management. b. care management. c. disease management. d. demand management.

ANS: B Care management is an enduring process in which a manager establishes systems and monitors the health status, resources, and outcomes for a targeted aggregate of the population. Case management is defined as a collaborative process of assessment, planning, facilitation, care coordination, evaluation, and advocacy for options and services to facilitate an individual's and family's comprehensive health needs through communication and available resources to promote quality cost-effective outcomes. Disease management constitutes systematic activities to coordinate health care interventions and communications for populations with disease conditions in which client self-care efforts are significant. Demand management seeks to control use by providing clients with correct information and education strategies to make healthy choices, to use healthy and health-seeking behaviors to improve their health status, and to make fewer demands on the health care system.

Environmental health is important to nurses because chemical, biological, and radiological materials are: a. a major cause of global warming. b. often found in the air, water, and products we use. c. frequently linked to the development of chronic illnesses. d. products that nurses work with on a daily basis.

ANS: B Chemical, biological, and radiological pollutants are often found in the air we breathe, the water we drink, and the products we use. These are not a major cause of global warming. They are not frequently linked to the development of chronic illnesses. These are not products that nurses work with on a daily basis.

A public health nurse (PHN) is working with a migrant farm worker who has experienced an exposure to a pesticide. When researching pesticides, the nurse looks at the "family" of the chemical. What similarities are found among chemicals that have been placed in the same family? a. Route of entry into the body b. Actions and associated risks c. Effects that they have on the body d. Potency and toxicity

ANS: B Chemicals are grouped so it's possible to understand the actions and risks associated with each group. Although some common health risks exist within these families of chemicals, the possible health risks for each chemical should be evaluated individually when a potential human exposure exists.

The nurse and client, a 20-year-old expectant mother, are discussing the advantages and disadvantages of breastfeeding. To enhance understanding of the implications of breastfeeding, the nurse says, "Tell me more about how you will work full time and breastfeed." Which of the following aspects of the advocacy process is being used? a. Verification b. Clarification c. Amplification d. Affirmation

ANS: B Clarification is a process in which the nurse and client strive to understand meanings in a common way. Verification is the process used by the nurse advocate to establish accuracy and reality in the informing process. Affirmation is based on an advocate's belief that a client's decision is consistent with the client's values and goals. Amplifying occurs between the nurse and the client to assess the needs and demands that will eventually frame the client's decision.

Which statement is true about health care services for migrant workers? a. Clinics are usually found within walking distance from work. b. Undocumented workers fear deportation when using clinics. c. Health care records are kept by workers to allow for continuation of services. d. Clinics are most often open on weekends and keep evening hours.

ANS: B Clinics are usually a great distance from work and offer health services only during work hours. Health records do not typically follow the client.

A community has residents who provide information to the city council so that decisions can be made about the health of the community. Which of the following characteristics is being displayed? a. Active partnerships b. Passive partnerships c. Gatekeeping d. Focus groups

ANS: B Coalitions are active partnerships in which all participants share leadership and decision making to some degree. Unfortunately, some community health efforts view community residents only as sources of information and receivers of interventions; this limits residents to passive participation. Gatekeepers refer to formal or informal community leaders who create opportunities for nurses to meet diverse members of the community. A focus group is similar to an interview in that it collects data mainly through asking open-ended questions to participants but to a small group rather than an individual.

A nurse is providing care for a family in which a nonaddict is experiencing a stress-induced preoccupation with an addict's life, leading to extreme dependence on the addict. Which of the following best describes what is happening? a. Obsession b. Codependency c. Enabling d. Addiction

ANS: B Codependency occurs when a person in a close relationship with an addict develops unhealthy coping mechanisms to continue the relationship. Obsession is a thought that continually preoccupies ones' thoughts. Enabling is the act of shielding or preventing the addict from experiencing the consequences of addiction. Drug addiction is a pattern of abuse characterized by an overwhelming preoccupation with the use of a drug, securing its supply, and a high tendency to relapse if the drug is removed.

A nurse is implementing community-based nursing care. Which of the following nursing interventions is the nurse most likely to complete? a. Assessing the health needs of a defined community b. Providing care to families in a community c. Promoting the health of an entire community d. Investigating environmental health problems in a community

ANS: B Community-based nursing practice is a setting-specific practice whereby care is provided for families where they live, work, and attend school. The focus is on acute and chronic care and the provision of services. Providing care to families is the only intervention that accurately describes this definition. Assessing the health needs of a community, promoting the health needs of an entire community, and investigating environmental health problems in a community would be considered public health nursing practice.

Dependence is a term that refers to the: a. presence and severity of withdrawal symptoms. b. necessary use of drugs to prevent withdrawal symptoms. c. level of intoxication associated with addiction. d. amount of substance needed to satisfy cravings.

ANS: B Dependence refers to the necessary use of drugs to prevent withdrawal symptoms. This occurs because of a state of neuroadaptation (a physiological change in the central nervous system).

A nurse visited a client living on a farm residency. Which of the following best describes the area the nurse visited? a. A region that has fewer than six persons per square mile b. A home outside areas zoned as "city limits" c. A county in a metro area with fewer than 1 million people d. An area adjacent to a highly populated city

ANS: B Farm residency usually refers to residency outside areas zoned as "city limits," usually implying involvement in agriculture. A frontier is a region having fewer than six persons per square mile. The term small describes counties in a metro area with fewer than 1 million people. Suburban is an area adjacent to a highly populated city.

A nurse advocates and intervenes between the health care system and the client's cultural beliefs on behalf of the client. Which of the following best describes the nurse's action? a. Cultural accommodation b. Culture brokering c. Cultural preservation d. Cultural repatterning

ANS: B Culture brokering is advocating, mediating, negotiating, and intervening between the client's culture and the biomedical health care culture on behalf of clients. Cultural accommodation refers to assistive, supportive, facilitative, or enabling nurse actions and decisions that help clients of a particular culture accept nursing strategies or negotiate with nurses to achieve satisfying health care outcomes. Cultural preservation refers to assistive, supportive, facilitative, or enabling nurse actions and decisions that help the clients of a particular culture to retain and preserve traditional values, so they can maintain, promote, and restore health. Cultural repatterning refers to assistive, supportive, facilitative, or enabling nurse actions and decisions that help clients of a particular culture to change or modify a cultural practice for new or different health care patterns that are meaningful, satisfying, and beneficial.

A nurse is observing behaviors that may be defined by culture. Which of the following is the nurse most likely to observe? a. Speaking a dialect of a language in a local region b. Standing when an older adult gets on the bus to give him a seat c. Immigrating to the United States and seeking work d. An organizational structure of a cultural group

ANS: B Culture is a set of beliefs, values, and assumptions about life that are widely held among a group of people and is transmitted intergenerationally. Behaviors may be culturally determined, as illustrated in the correct response. This behavior is based on the value of respect of elderly people. Speaking a particular dialect in a local area would not fit the definition of culture. An organizational structure of a group is not an observable behavior. Immigrating to the United States does not demonstrate culture.

A nurse is coordinating care to ensure a comfortable and peaceful death. Which of the following factors should be an important consideration for the nurse? a. The unique stress dying patients often experience b. The cultural values, expectations, and preferences of the family c. The communication style of each family member d. The abilities of the nurse and health care team

ANS: B Cultures vary in their beliefs and responses to death. Nurses should know the differences in cultural responses so that they can effectively help people in their time of need. It will be important for the nurse to take cues from the client and the loved ones regarding their needs. The stress experienced by dying patients is different based on the client's culture.

A nurse is working as a case manager and is in the process of performing interdisciplinary, family and client conferences. Which of the following phases of the nursing process is being implemented? a. Assessment b. Diagnosis c. Planning for outcomes d. Implementation

ANS: B Diagnosis includes the identification of a problem/opportunity. Examples of activities used during the diagnosis phase include holding conferences, determining conclusions on the basis of assessment, and using interprofessional teams. Examples of assessment include developing networks with target populations and dissemination of written materials. Examples of planning for outcomes include validating and prioritizing problems and selecting evidence-based interventions. Examples of implementation include contacting providers and coordinating care activities.

A nurse is conducting disease surveillance. Which of the following describes the rationale for this action? a. To eradicate a disease before it starts b. To establish a baseline rate of disease occurrence and patterns of spread c. To target populations for triage d. To reduce the incidence of heart disease in a community

ANS: B Disease surveillance helps establish baseline rates of disease occurrence and patterns of spread to make it possible to initiate a rapid response to an outbreak. Disease surveillance is unable to eradicate a disease before it starts, to target populations for triage, or to reduce the incidence of heart disease in a community. However, disease surveillance does make it possible to initiate a rapid response to an outbreak of a disease or an event that can cause a health problem.

A client has not yet made a commitment to take action to change his health behaviors. Which stage of change is the client experiencing? a. Precontemplation b. Contemplation c. Preparation d. Action

ANS: B During contemplation, the individual is aware that a problem exists and is seriously thinking about overcoming it but has not yet made a commitment to take action. The nurse can encourage the individual to weigh the pros and cons of the problem and the solution to the problem. In precontemplation, the person does not intend to change in the foreseeable future. In the preparation phase, the individual is prepared for action and may reduce the problem behavior but has not yet taken effective action. In the action stage, the individual modifies the behavior, experiences, or environment to overcome the problem.

A nurse is working in a community that is experiencing the Honeymoon Phase after a disaster. Which of the following is characteristic of this phase? a. First responders work tirelessly to save others. b. Survivors share their stories. c. Medical personnel experience exhaustion. d. Community organizations rebuild the community.

ANS: B During the Honeymoon Phase, survivors rejoice for their survival and may share their experiences and stories. First responders work tirelessly to save others during the Heroic Phase. Medical personnel experience exhaustion during the Disillusionment Phase. The community begins to rebuild during the Reconstruction Phase.

A nurse is completing a community assessment. Which of the following community characteristics would be identified as a factor for violence? a. Lack of family cohesion b. High crime rate c. Social isolation d. Substance abuse

ANS: B High crime rate is the only "community characteristic." Lack of family cohesion is a familial factor. Social isolation and substance abuse are individual factors.

A nurse believes everyone is entitled to equal rights and equal treatment in society. Which of the following principles is being applied? a. Distributive or social justice b. Egalitarianism c. Libertarian view of justice d. Communitarianism

ANS: B Egalitarianism is defined as the view that everyone is entitled to equal rights and equal treatment in society. Distributive justice requires that the distribution of benefits and burdens on a society be fair or equal. The libertarian view of justice holds that the right to private property is the most important right. Communitarianism views that individual rights need to be balanced with social responsibilities; individuals do not live in isolation but are shaped by the values and culture of their communities.

What type of abuse is occurring when a child does not receive the basic nurturing and caring needed for healthy development? a. Sexual abuse b. Emotional neglect c. Physical neglect d. Child abuse

ANS: B Emotional neglect is the omission of basic nurturing, acceptance, and caring of a child. Physical neglect is failure to provide adequate food, proper clothing, shelter, hygiene, or necessary medical care and is most often associated with extreme poverty. Child abuse ranges from violent physical attacks to passive neglect. Sexual abuse is a form of child abuse when a child has had nonconsensual sexual contact of any kind.

An orderly process that considers ethical principles, client values, and professional obligations is: a. moral distress. b. ethical decision making. c. a value. d. a code of ethics.

ANS: B Ethical decision making is defined as an orderly process that considers ethical principles, client values, and professional obligations. Moral distress is an uncomfortable state of self when one is unable to act ethically. Values are beliefs about the worth or importance of what is right or esteemed. A code of ethics is a moral standard that delineates a profession's values, goals, and obligations.

A client shares with the nurse that her grandparents immigrated to the United States from Germany. Which of the following best describes what she has disclosed? a. Multiculturalism b. Ethnicity c. Race d. Culture

ANS: B Ethnicity represents the identifying characteristics of culture, such as race, religion, or natural origin. Multiculturalism is the blending of diverse cultures. Race is primarily a social classification that relies on physical markers. Culture is a set of beliefs, values, and assumptions about life that are widely held among a group of people and transmitted intergenerationally.

A nurse is engaging in the evaluation phase of community health program planning. Which of the following would be part of effective completion of this phase? a. Demonstrate the ability to improve the health of the participants b. Develop measureable objectives and goals before implementation c. Encourage full participation by community members d. Improve the health of the population through the program

ANS: B Evaluation begins in the planning phase, when goals and measurable objectives are established and goal-attaining activities are identified. After implementing the intervention, only the meeting of objectives and effects of the intervening activities have to be assessed. Participation by community members and improvement of the health of the population may be measured through the evaluation phase, but would not demonstrate completion of this phase. The evaluation phase may determine if the participants' health improved, but improvement of health is not part of the completion of this phase.

A nurse who was working in the 1960s used the code of ethics to guide making an ethical decision. Which code of ethics would have been used? a. Nightingale Pledge b. Code for Professional Nurses c. Code of Ethics for Nurses with Interpretive Statements d. International Council of Nurses (ICN) Code of Ethics for Nurses

ANS: B Florence Nightingale lived in the 1800s. The Code for Professional Nurses was adopted in 1950, the Code of Ethics for Nurses with Interpretive Statements was adopted in 2001, and the ICN Code of Ethics for Nurses was adopted in 2000.

A case manager submits documentation that a nursing visit was completed at a client's home, but it was never performed. According to the general areas of legal risk, how would this action best be categorized? a. Liability for managing care b. Fraud/abuse c. Negligent referral d. Confidentiality/security

ANS: B Fraud/abuse occurs when false statements of claims are made and filed. Confidentiality/security occurs when sensitive information is not secured. Negligent referral occurs when referrals or treatments are inappropriate. Liability for managing care occurs when case management is not used appropriately, when there is harassment, or when care is inappropriately delegated.

A nurse administrator is educating her new nursing staff about the use of health care by rural residents. Which of the following statements is she most likely to include in her presentation? a. "Rural adults are less likely to identify a particular medical provider." b. "Rural adults are more likely to see general practitioners and advanced practice nurses than their urban counterparts." c. "Rural adults are more likely to seek medical treatment from a specialist than those in urban areas." d. "Rural adults usually have to travel an average of 60 minutes to see a provider."

ANS: B General practitioners and advanced practice nurses are seen more often by rural adults; urban adults are more likely to get specialized care. Rural adults are more likely to identify a particular medical provider. Rural adults are less likely to seek medical treatment from a specialist than those in urban areas. Rural adults are more likely to travel more than 30 minutes to reach their usual source of care.

A public health nurse (PHN) understands that the emergence of new infectious diseases is influenced by the: a. increased availability of immunizations. b. globalization of food supplies. c. decreased use of child care facilities. d. creation of sanitation systems in third world countries.

ANS: B Globalization of food supplies is one of the many factors that can influence the emergence of infectious diseases. The increased use of childcare facilities could contribute to the emergence of new infectious diseases. The lack of sanitation systems in third world countries could contribute to the emergence of new infectious diseases. The increased availability of immunizations should decrease the emergence of diseases.

The health priorities in HEALTH21 are based on the concept of: a. eradication of communicable disease worldwide. b. social justice and solidarity with an emphasis on the responsibility for health. c. world trade and knowledge sharing about health concerns. d. provisions for the creation of the United Nations Children's Fund (UNICEF).

ANS: B HEALTH21 is not a single, finite goal but a strategic process that can lead to progressive improvement in the health of people. In essence, it is a call for social justice and solidarity with an emphasis on individual, family, and community responsibility for health. The concepts of eradication of communicable disease worldwide, world trade and knowledge sharing about health concerns, and provisions for the creation of UNICEF are not part of these health priorities.

A nurse is using health status indicators to complete a community assessment. Which of the following best explains what the nurse is doing? a. Interviewing key informants in the community b. Examining morbidity and mortality rates in the community c. Surveying local businesses in the community d. Observing community members

ANS: B Health indicators are numerical measures of health outcomes, such as morbidity and mortality, as well as determinants of health and population characteristics. Generally, these data are from secondary sources such as websites or printed materials. Interviewing key informants, surveying local businesses, and making observations are all methods of primary data collection.

A nurse is working to improve the health status of a population in a lesser-developed country. Which of the following interventions would the nurse most likely support? a. Decrease the number of people who need outside resources b. Increase the number of children who can attend school c. Increase the monetary resources that can be used to treat disease and illness d. Improve the ability to pay debt to more developed countries

ANS: B Health status improvement in children can lead to better education that eventually helps the country's economic growth. Because children are educated, there is more economic growth. This allows for additional monetary resources to be spent on economic development rather than on treatment of illness and disease.

Immunity is a characteristic of the: a. agent factor. b. host factor. c. environmental factor. d. epidemiologic triad.

ANS: B Immunity refers to species-determined resistance to an infectious agent and is determined by the characteristics of the host. The agent is the infection that is causing the infection/disease. Environmental factors may influence the susceptibility of the host. The epidemiologic triad involves the interaction of the host, agent, and environment.

A nurse practitioner reports a case of gonorrhea to the local health department. Which type of surveillance system is being used? a. Active b. Passive c. Sentinel d. Special

ANS: B In the passive system, case reports are sent to local health departments by health care providers. In the active system, the public health nurse may begin a search for cases through contacts with local health providers and health care agencies. In the sentinel system, trends in commonly occurring diseases or key health indicators are monitored. Special systems are developed for collecting particular types of data and may be a combination of active, passive, and/or sentinel systems.

A nurse wants to find more information about indoor air quality. Which of the following websites would be most helpful? a. The National Institutes of Health (NIH) b. The American Lung Association c. "Right to Know" d. The Occupational Safety and Health Administration (OSHA)

ANS: B Indoor air quality is a growing public health concern in office buildings, schools, and homes and is reflected in the alarming rise in asthma incidence in the United States, particularly among children. Sources of information about indoor air quality include the EPA and the American Lung Association. The NIH is part of the U.S. Department of Health and Human Services (USDHHS) and is responsible for health and biomedical research. "Right to Know" is part of a workplace safety legislation. OSHA regulates safety in factories and businesses.

A community health nurse is conducting informant interviews in a small community. Which of the following would the nurse most likely contact? a. The state department of health for death records b. A local priest for congregation information c. Surrounding communities for crime comparison d. The Centers for Disease Control and Prevention (CDC) for illnesses in the area

ANS: B Informant interviews are methods of directly collecting data. County health department nurses and church officials are often good key informants. Key informants need not hold any formal title, but are generally viewed as community leaders by other community members and often have a long history in the community. The state department of health, surrounding communities, and CDC do not meet the definition of an informant.

A nurse states that she has been reading about an epidemic that has been occurring in the world. Which of the following is she most likely referring to? a. "Bird" flu in China b. Adult obesity in the United States c. An isolated case of smallpox in Africa d. The nursing shortage in the United States

ANS: B It is estimated that 30% of the adults in the United States are obese. According to the Centers for Disease Control and Prevention, this is an epidemic. The other examples are not epidemics.

A nurse is working in a health department when a patient arrives who has been traveling to South America and has been diagnosed with malaria. Which of the following considerations should be made by the nurse? a. The Morbidity and Mortality Weekly Report should be consulted to investigate the rate of malaria in the United States. b. This is a disease that must be reported to the state health department. c. The nurse should take precautions to wear a mask and gown to avoid exposure. d. The patient is very ill and should be sent to the hospital immediately.

ANS: B Malaria is on the list of infectious diseases notifiable at the national level. The Morbidity and Mortality Weekly Report is published weekly with the rates of disease; however, this would not be the first consideration by the nurse. Malaria is not spread by direct contact, rather from a bite from an infected mosquito. Malaria begins with flu-like symptoms, or the client may have very few symptoms.

A nurse is advocating for increased funding for an international agency. Which of the following best describes how these funds will likely be used? a. Development and installation of technology b. Food relief, worker training, and disaster relief c. Support of dictatorships and corrupt governments d. Funding of local private organizations

ANS: B Many international organizations have an ongoing interest in global health. Despite the presence of these well-meaning organizations, it is estimated that the lesser-developed countries still bear most of the cost for their own health care and that contributions from major international organizations actually provide for less than 5% of needed costs. Recent reports indicate that the majority of funds raised by international organizations are used for food relief, worker training, and disaster relief.

A nurse is working with a vulnerable group experiencing multiple risk factors. Which of the following best describes this group? a. Smokers who use chewing tobacco as well as cigarettes b. Substance abusers who test positive for HIV c. Persons with limited access to care because they live in a rural area d. New mothers needing information about baby and child care

ANS: B Multiple risk factors are present in substance abusers, including contracting HIV and hepatitis B virus. Homelessness is another risk factor. Vulnerable populations of concern to nurses are persons who are poor or homeless, have special needs, pregnant teens, migrant workers and immigrants, individuals with mental health problems, people who abuse addictive substances, persons who have been incarcerated, people with communicable diseases and those who are at risk, and persons who are HIV positive or have hepatitis B virus or STDs.

Which ethical principle requires "doing no harm?" a. Respect for autonomy b. Non-maleficence c. Beneficence d. Distributive justice

ANS: B Non-maleficence refers to doing no harm. Respect for autonomy requires that individuals be permitted to choose those actions and goals that fulfill their life plans unless those choices result in harm to another. Beneficence requires that we do good. Distributive justice requires that there be a fair distribution of the benefits and burdens in society based on the needs and contributions of its members.

Which is considered a nonpoint source of pollution? a. Hazardous waste site b. Animal waste from wildlife c. Chlorine poured down a well d. Stagnant water

ANS: B Nonpoint sources come from more diffuse exposures to pollution. Animal waste is the only diffuse exposure given. The others are considered point sources; point sources are individual, identifiable sources such as smoke stacks.

A nurse is educating a client about reducing her risk factors for the development of colorectal cancer. Which of the following lifestyle changes should the nurse recommend? a. Increase in consumption of dairy products b. Reduction in consumption of processed meats c. Decrease in exposure to the sun d. Increase in sedentary lifestyle

ANS: B Obesity, physical inactivity, smoking, heavy alcohol consumption, a diet high in red or processed meats, and insufficient intake of fruits and vegetables are risk factors for colorectal cancer.

Which of the following describes a barrier to obtaining care in a rural area? a. Readily available transportation b. Inadequate provider understanding about rural health needs c. Short distances to health care facilities d. Available specialists

ANS: B One common barrier to health care is inadequate provider attitudes and understanding about rural populations. The barriers to health care among the rural population are associated with one of the following characteristics: whether services and professionals are available, affordable, accessible, or acceptable to rural consumers.

Which of the following describes the consequence of the successful implementation of the Affordable Care Act? a. Americans will pay closer attention to their health status. b. The majority of the population will be covered by insurance. c. Public health departments will need to increase the number of nursing positions. d. The prevalence of obesity will decrease.

ANS: B One consequence of successful implementation of the Affordable Care Act might actually be that the majority of the population would be covered by insurance and public health agencies will not need to provide direct clinical services in order to assure that those who need them can receive them. The Affordable Care Act will not directly cause Americans to pay closer attention to their health status or decrease the prevalence of obesity.

A nurse is investigating an outbreak of a gastrointestinal illness from a food-borne pathogen. Which of the following terms is being applied? a. Attack rate b. Point epidemic c. Secular trend d. Event-related cluster

ANS: B One temporal and spatial pattern of disease distribution is the point epidemic. A point epidemic is most clearly seen when the frequency of cases is plotted against time. The sharp peak characteristic of such graphs indicates a concentration of cases in some short interval of time. Attack rate is defined as the proportion of persons who are exposed to an agent and develop the disease. Secular trends are long-term patterns of morbidity or mortality rates. Event-related clusters are patterns in which time is not measured from fixed dates on the calendar but from the point of some exposure or event, presumably experienced in common by affected persons, although not occurring at the same time.

PHNs are looking to improve population-focused care in the community. Which of the following best describes a key opportunity for the nurses to accomplish this goal? a. Assuming traditional nursing roles b. Influencing public health policy c. Conducting community assessments d. Specializing in community-based nursing practice

ANS: B PHNs must move into positions in which they can influence policy formation. They must assume positions that are not traditionally considered nursing. Conducting community assessments is part of public health nursing practice, not necessarily a key opportunity to improve practice. Specializing in community-based nursing practice is setting-specific and does not relate to a key opportunity to improve population-focused care.

The public health nurse (PHN) is taking care of a poor family, whose parents, grandparents, and great-grandparents also have been poor. Which of the following terms best describes this family? a. Near poor b. Persistent poor c. Impoverished d. Neighborhood poverty

ANS: B Persistent poor refers to individuals and families who remain poor for long periods and who pass poverty on to their descendants. The near poor are persons and families whose income is above the federal income guidelines but insufficient to meet living expenses. Neighborhood poverty refers to geographically defined areas of high poverty, characterized by run-down housing, high unemployment rates, and poorer health outcomes. Impoverished refers to those living in poverty.

A nurse is conducting a community assessment in a poor neighborhood. Which of the following is the nurse most likely to find? a. Fewer minority groups b. Increased exposure to environmental hazards c. More available health care services d. Lower rates of crime and substance abuse

ANS: B Poor neighborhoods are more likely to have exposure to environmental hazards. Lower socioeconomic neighborhoods have been linked with poorer general health status, chronic conditions, mortality, birth outcomes, disability, injury and violence, and other health indicators. They are more likely to have higher rates of crime and poorer access to health care services.

In comparison with urban adults, rural adults: a. seek medical care more often. b. have poorer overall health status. c. are less likely to identify a usual source of medical care. d. are usually seen by a specialist rather than a general practitioner.

ANS: B Poorer overall health status is attributed to impaired access to health care providers and services coupled with other rural factors. Rural residents seek medical care less often, are usually seen by a general practitioner, and are more likely to identify a usual source of medical care.

Population health is an approach and perspective that focuses on: a. control of the spread of the HIV virus worldwide. b. the broad range of factors and conditions that influence health. c. community-based care for all citizens. d. prevention and diagnosis of disease worldwide.

ANS: B Population health is a holistic approach that considers the total health system. The focus of population health is broader than only considering spread of the HIV virus. Population health considers more than the community-based care in the community. Population health examines spread of disease on a smaller scale.

A nurse is using population management as part of the nursing care that is provided. Which of the following activities is the nurse most likely completing? a. Census taking to determine the total number of people in the population b. Assessing the needs of the client population through compilation of health histories c. Providing case management services for every citizen in the community d. Selecting programs for wellness that are repeated annually

ANS: B Population management involves assessing the needs of the client population through health histories, claims, use-of-service patterns, and risk factors. Population management does not include census taking, providing case management services for all, or repeating wellness programs on an annual basis.

Which problem does a health department usually have the legal authority to investigate? a. Pandemics b. Unusual clusters of illness c. World trends of disease d. Cases of the common cold and pneumonia

ANS: B Powers of local government include surveillance of unusual clusters of illness. A health department does not have legal authority to investigate pandemics, world trends of disease, and cases of the common cold and pneumonia.

What happened in the United States as a result of prohibition? a. Substance abuse declined b. Violent crime and corruption increased c. The need for community health nurses declined d. Facilities to treat substance abuse improved

ANS: B Prohibition led to increased crime and corruption among law officials related to the illicit market. Because of the problems that were caused, prohibition was eventually recognized as a failure and repealed.

Which type of cancer is the second leading cause of cancer deaths in the United States? a. Gastrointestinal b. Prostate c. Skin d. Testicular

ANS: B Prostate is the most common non-skin cancer and second leading cause of cancer deaths in the United States. Testicular cancer is rare. Skin cancer and gastrointestinal cancer are not among the leading causes of cancer deaths.

A nurse is giving advice using the acronym FRAMES. Which of the following actions would most likely be taken by the nurse? a. Offer a suggestion as to how the behavior should change b. Provide clear guidance to change risky behavior c. Refer the client to a local support group d. Emphasize the need for others to support the client's change

ANS: B Providing clear advice to change risky behavior is one of the elements of the FRAMES method of intervention, "A"—advice. The other FRAMES are feedback, responsibility, menu, empathy, and self-efficacy.

To better address emerging public health issues, a PHN plans to complete continuing education in this area. Which of the following content areas should be included in the course that is chosen? a. Leadership b. Ethics c. Communication d. Finance

ANS: B Public health workers should be educated in eight content areas to be able to address emerging public health issues and advances in science and policy: (1) informatics, (2) genomics, (3) cultural competence, (4) community-based participatory research, (5) policy, (6) law, (7) global health, and (8) ethics.

A person has returned from overseas and it is determined that he has Q fever. Which of the following must occur? a. This is not a disease that is on the notifiable list; nothing need be reported. b. The case must be reported to the state and local health departments. c. The patient must be isolated immediately. d. The patient will probably die, so it is not reportable.

ANS: B Q fever is on the notifiable list and must be reported to the state and local health departments. Thus, it needs to be reported, but does not imply immediate isolation or probable death.

A nurse collects data about seat belt usage by interviewing key informants and observing behaviors in the community. Which of the following types of data is being collected? a. Quantitative b. Qualitative c. Focus groups d. Participant observation

ANS: B Qualitative data is collected through interviews and observation. This data is the descriptions provided by these individuals through interviews. Quantitative data is the numbers that can be found about the community, such as crime statistics. A focus group is similar to an interview in that it collects data mainly through asking open-ended questions to participants, but to a small group rather than an individual. Participant observation refers to the deliberate sharing in the life of a community, such as participating in a local fair or festival, or attending a political or social event.

Which statement fits the Liberal Democratic Theory of John Rawls? a. One should reject any idea that societies, states, or collectives of any form can be the bearers of rights or can owe duties. b. Inequalities result from birth, natural endowment, and historic circumstances. c. Everyone has a right to private property. d. Government should be limited.

ANS: B Rawls acknowledges that inequities are inevitable in society, but he tries to justify them by establishing a system in which everyone benefits, especially the least advantaged. This is an attempt to address the inequalities that result from birth, natural endowments, and historic circumstances. The other choices relate to libertarianism.

A nurse is implementing a tertiary prevention strategy. Which of the following activities would the nurse most likely implement? a. Diabetes screening b. Rehabilitation for musculoskeletal injury c. Birth control d. Pregnancy testing

ANS: B Rehabilitation for musculoskeletal injury is an example of tertiary prevention. Diabetes screening and pregnancy testing are secondary prevention strategies, and birth control is a primary prevention.

A nurse is planning to conduct research in a rural setting. Which of the following areas of research may be of most importance to rural nursing practice? a. Information about residents living in the intermountain and Midwestern regions b. Information on the stressors and rewards of rural practice c. Studies related to moving from rural environments to urban ones d. Studies about the effects of air pollution

ANS: B Research on the stressors and rewards of rural practice, in particular public and community health nursing, is needed. Other priority areas of research include rural nursing practice, empirical data on the particular nursing needs of rural-client systems, international perspective on the health of rural populations, the use of technology and communication, and rural-urban disparities in health status.

The probability an event will occur within a specified period of time is called: a. rate b. risk c. epidemiology d. epidemic

ANS: B Risk is the probability an event will occur within a specified period of time. Rate is a measure of the frequency of a health event in a defined population, usually in a specified period of time. Epidemiology is the study of the occurrence and distribution of health-related states or events in specified populations, including the study of the determinants influencing such states, and the application of this knowledge to control the health problems. Epidemic occurs when the rate of disease, injury, or other condition exceeds the usual level of that condition.

A nurse assists an injured victim in finding a safe location to live after being abused. Which of the following levels of prevention is being implemented? a. Primary b. Secondary c. Tertiary d. Primary care

ANS: B Secondary prevention focuses on early detection and prompt treatment of disease, injury, or disability. Finding shelter for a victim of abuse demonstrates secondary prevention. Primary prevention refers to those interventions aimed at preventing the occurrence of disease, injury, or disability. Tertiary prevention refers to interventions aimed at treatment and rehabilitation. Primary care is not a level of prevention.

A nurse administers a rabies immunization post-exposure to an animal bite. Considering the interventions used with infectious disease, which of the following levels of prevention is being used? a. Primary prevention b. Secondary prevention c. Tertiary prevention d. Assessment

ANS: B Secondary prevention focuses on early detection and prompt treatment of disease, injury, or disability. Primary prevention seeks to reduce the incidence of disease by preventing occurrence. Tertiary prevention reduces complications through treatment and rehabilitation. Assessment refers to the systematic collection of data.

A nurse offers a screening for hearing defects at a local community center. Which of the following best describes the action of the nurse? a. Primary prevention b. Secondary prevention c. Tertiary prevention d. Health promotion

ANS: B Secondary prevention focuses on early detection and prompt treatment of disease, injury, or disability. Screenings are part of secondary prevention interventions. Primary prevention refers to interventions aimed at preventing the occurrence of disease, injury, or disability. Tertiary prevention includes interventions aimed at disability limitation and rehabilitation from disease, injury, or disability. Health promotion is a specific primary prevention strategy.

A nurse is educating a community women's group about the importance of monthly breast self-exam and annual mammograms. Which of the following best describes why this community health education is important? a. Increased screening will lead to a decreased incidence of breast cancer. b. Early detection of breast cancer typically leads to a better prognosis. c. The death rate of breast cancer continues to rise. d. The potential harm from screening needs to be discussed.

ANS: B Secondary prevention measures, including screening techniques, make a difference in the death rates for breast cancer. Early detection often means cure, whereas late detection typically results in an unfavorable prognosis. A potential harm from screening does not exist.

A nurse is considering the potential for selection bias. Which of the following best describes the situation that is the nurse has encountered? a. Determining the population to be studied b. Considering how the participants will enter the study c. Studying cause and effect relationships d. Documenting results of the study

ANS: B Selection bias is attributable to the way subjects enter a study. It has to do with selection procedures and the population from which subjects are drawn. Determining the population to be studied relates to the design of the study. Studying cause and effect relationship has to do with ecological studies. Documenting the results of the study is completed at the end of the study after the participants have been selected.

Which factor has contributed to the problem of substance abuse? a. Increased knowledge about the use of drugs b. Social acceptability of certain drugs c. Recognition of substance abuse as a health problem d. Control of the content and strength of illegal drugs

ANS: B Social acceptance of certain drugs, such as caffeine, alcohol, and tobacco, contributes to the substance abuse problem. Not all substances that are abused are illegal.

A nurse is examining social determinants of health. Which of the following is the nurse looking at? a. Ethnicity b. Income c. Gender d. Marital status

ANS: B Social determinants of health are such factors as economic status, education, environmental factors, nutrition, stress, and prejudice that lead to resource constraints, poor health, and health risk. Ethnicity, gender, and marital status are not considered social determinants of health.

A community health nurse has recently become involved in surveillance. Which of the following describes the situation that the nurse is in? a. Educating clients about influenza immunizations b. Collecting information about occurrence of measles c. Evaluating the effectiveness of an HIV/AIDS prevention program d. Advocating for changes in the national disease reporting requirements

ANS: B Surveillance gathers the "who, when, where, and what"; these elements are then used to answer "why." Nurses are frequently involved in surveillance by collecting data, making diagnoses, investigating and reporting cases, and providing information to the general public. Client education, program evaluation, and advocating for changes are not part of the surveillance process.

The use of any substance that threatens a person's health or impairs his or her social or economic functioning is: a. drug dependence. b. substance abuse. c. drug addiction. d. illegal.

ANS: B Substance abuse is the use of any substance that threatens a person's health or impairs his or her social or economic functioning. Drug dependence is a state of neuroadaptation caused by the chronic, regular administration of a drug. Drug addiction is a pattern of abuse characterized by an overwhelming preoccupation with the use of a drug, securing its supply, and a high tendency to relapse if the drug is removed. Illegal means that it is against the law, not all substances are illegal.

A nurse upholds a client's right to make a choice and to act on the choice. Which of the following best describes the action being taken by the nurse? a. Affirming b. Supporting c. Informing d. Amplifying

ANS: B Supporters approve and support clients' actions. Affirming is based on an advocate's belief that a client's decision is consistent with the client's values and goals. Informing includes educating clients about the nature of their choices, the content of those choices, and the consequences to the client and is not a one-way activity. Amplifying occurs between the nurse and the client to assess the needs and demands that will eventually frame the client's decision.

Which are examples of knowledge domains used in case management? a. Legal issues, malpractice recognition, and community involvement b. Teaching, counseling, and education skills c. Advocacy, political campaigning and legislative change d. Grant application, bargaining contracts, and securing funding

ANS: B Teaching, counseling, and education skills are an important part of case management in addition to program evaluation and research. The other domains listed are not used in case management.

A nurse is using telehealth technology in the home setting. Which of the following best describes the intervention that is being used by the nurse? a. Uses Web TV to teach clients about their health b. Shares health information using electronic communications c. Makes regular visits to clients to check the technology d. Risks violation of the Health Insurance Portability and Accountability Act

ANS: B Telehealth includes sharing health information between the client and clinicians using electronic communications. Telehealth may or may not include video technology for live interactions. Telehealth allows for monitoring health status and symptom recognition, providing education, increasing communication, and enabling clients to become active partners in their own care.

Testicular cancer is commonly found in which age-group? a. 10 to 14 years b. 15 to 40 years c. 41 to 50 years d. 51 to 70 years

ANS: B Testicular cancer is commonly found in men 15 to 40 years old, with the peak incidence between the ages of 18 and 40 years.

Which organization publishes the current scope and standards of home care practice? a. Centers for Medicare and Medicaid Services b. American Nurses Association (ANA) c. Robert Wood Johnson Foundation d. Visiting Nurse Association

ANS: B The ANA publishes scope and standards for Home Health Nursing and Hospice and Palliative Nursing. The Centers for Medicare and Medicaid Services control the regulations and reimbursement mechanisms for the majority of home care services. The Robert Wood Johnson Foundation provides grant funding for a variety of projects that support nursing. The Visiting Nurse Association is a home care agency.

A nurse is organizing a fundraiser for a philanthropic organization. Which of the following organizations is the nurse most likely working with? a. International Red Cross b. Carnegie Foundation c. Nestlé d. Johnson & Johnson

ANS: B The Carnegie Foundation is a philanthropic organization, receiving funding from private endowment funds. The Red Cross is a nongovernmental agency, and Nestlé and Johnson & Johnson are private voluntary organizations.

A community health nurse is investigating international diseases. Which of the following would the nurse track by using the global burden of disease (GBD)? a. The number of cases of HIV/AIDS in a country b. Losses from premature death and losses of healthy life that result from disability c. The economic impact of disability on lesser-developed countries d. The rate of disability in industrialized nations

ANS: B The GBD combines losses from premature death and losses of healthy life that result from disability. The GBD represents units of disability-adjusted life-years (DALYs). This statistic is not specific to a particular disease or a particular developmental stage of a country.

Which concepts are used to evaluate client outcomes when using the Omaha System Problem Rating Scale for Outcomes? a. Problem, Intervention, and Outcome b. Knowledge, Behavior, and Status c. Knowledge, Skill, and Attitude d. Problem, Category, and Modifiers

ANS: B The Omaha System Problem Rating Scale for Outcomes uses the concepts of Knowledge, Behavior, and Status.

The Refugee Act of 1980 had what effect on refugees who were immigrating into the United States? a. Allowed specific provisions for refugees from Southern and Eastern Europe b. Provided a uniform procedure for refugees to be admitted into the United States c. Permitted refugees to set up communities in or around major metropolitan areas d. Allowed refugees access to "green cards" that would allow them to work in the United States

ANS: B The Refugee Act of 1980 provided a uniform procedure for refugees to be admitted into the United States. Refugees were immediately eligible to receive Temporary Assistance for Needy Families, Supplemental Security Income, and Medicaid. This procedure was in effect for all refugees not just those from specific countries. It did not provide "green card" access or permit development of specific communities for refugees.

A bioterrorism attack has occurred in the United States. Which of the following programs would be used to provide large quantities of medications to the American public? a. Cities Readiness Initiative b. Strategic National Stockpile (SNS) c. Public Health Information Network (PHIN) d. Project BioShield

ANS: B The SNS has the capacity to provide large quantities of medicine and medical supplies to protect the American public in a public health emergency. The Cities Readiness Initiative aids cities in increasing their capacity to deliver medicines and medical supplies during a public health emergency. The PHIN helps ensure information access and sharing. Project BioShield is a program to develop and produce new drugs and vaccines against potential bioweapons.

A community health nurse is utilizing the health care system in the United Kingdom. Which of the following expectations should the nurse have? a. An equal employer-citizen share in cost of health care b. Ownership and operation of the system by the government c. Nurse practitioners as the primary providers of care in the system d. Primary care provided in hospitals

ANS: B The United Kingdom has a tax-supported health system that is owned and operated by the government. Services are available to all without cost or for a small fee. Physicians are the primary providers in this system; nurses and allied health professionals are also recognized and used. Services are made available through hospitals, private physicians and allied health professional clinics, health outreach programs such as hospice, boroughs, and environmental health services.

A nurse is assessing a refugee who fled the genocide in Darfur. Which of the following disease/disorder should the nurse anticipate finding? a. TB b. Post-traumatic stress disorder (PTSD) c. Chronic obstructive pulmonary disorder (COPD) d. Malnutrition

ANS: B The biopsychological changes seen following genocide include physical stress reactions (cardiovascular, neurological) and mental stress responses, especially PTSD and depression. The biopsychosocial changes are the major concern over any physical illnesses that may be present.

A case manager provides a formal communication link among all parties concerning the plan of care management. Which of the following roles of the nurse is being implemented? a. Facilitator b. Liaison c. Coordinator d. Negotiator

ANS: B The case manager providing a formal communication link among all parties concerning the plan of care management is performing the role of liaison. As a facilitator, the nurse supports all parties in work toward mutual goals. When working as a coordinator, the case manager arranges, regulates, and coordinates needed health care services for clients at all necessary points of services. The case manager who uses effective collaboration and team strategies to make arrangements for services is performing the role of negotiator.

A case manager uses effective collaboration and team strategies to make arrangements for services. Which of the following roles is being applied? a. Broker b. Negotiator c. Liaison d. Facilitator

ANS: B The case manager who uses effective collaboration and team strategies to make arrangements for services is performing the role of negotiator. The role of broker is used when acting as an agent for provider services that are needed by clients to stay within coverage according to budget and cost limits of the health care plan. When acting as a liaison, the nurse provides a formal communication link among all parties concerning the plan of care management. As a facilitator, the nurse supports all parties in work toward mutual goals.

A nurse is caring for the community as the client. Which of the following is most likely the focus of the nurse's care? a. Providing care for aggregates living in the community b. The collective good of the population c. The provision of care for families in the home setting d. Providing health education in the community

ANS: B The community is considered the client when the nursing focus is on the collective good of the population. Providing education is one way the nurse can care for the community as the client; however, the focus of nursing care is larger in scope than only education. Provision of care for aggregates and families does not address the larger scope of community as client.

A nurse is applying the Minnesota Model of Public Health Interventions: Applications for Public Health Nursing Practice steps of surveillance. Which of the following describes the initial action that the nurse would take? a. Analyze data b. Consider whether surveillance is appropriate to the situation c. Evaluate the impact of the surveillance d. Collect data

ANS: B The first step in this model considers whether the surveillance was appropriate. Analyzing data is the fifth step in this model. Evaluation of the impact of the surveillance is the last step of the model. Collecting data is the fourth step in this model.

A nurse is providing contraceptive counseling to a female. Which of the following goals does the nurse plan to obtain? a. Encourage the individual to choose abstinence b. Ensure the individual is educated to make an informed choice about reproduction c. Advocate for increased funding for reproductive services d. Reduce the health risks of the individual

ANS: B The goal of contraceptive counseling is to ensure that women have appropriate instruction to make informed choices about reproduction. The nurse should provide a nonjudgmental approach during counseling and allow the woman to choose the appropriate contraceptive method. Nurses do advocate for reproductive services for women, but that is not a goal of contraceptive counseling. Reduction in the health risks of the individual is a goal of preconceptual counseling.

The time interval between invasion by an infectious agent and the first appearance of signs and symptoms of the disease is called: a. communicable period. b. incubation period. c. infectiousness. d. endemic.

ANS: B The incubation period is the time interval between invasion by an infectious agent and the first appearance of signs and symptoms. The communicable period is the interval during which an infectious agent may be transferred directly or indirectly from an infected person to another person. Infectiousness is the measure of the potential ability of an infected host to transmit the infection to other hosts. Endemic refers to the constant presence of a disease within a geographic area or a population.

The leading cause of birth defects in the United States is: a. cigarette smoking. b. fetal alcohol syndrome. c. marijuana use. d. HIV from sharing needles.

ANS: B The leading preventable cause of birth defects in the United States is fetal alcohol syndrome, causing mental and behavioral impairment.

Which statement about migrant and seasonal farm workers is true? a. There are more than 10 million working in the United States. b. The majority are Mexican. c. Most are employed in the Northeastern states. d. Thirty-five percent are undocumented workers.

ANS: B The majority of migrant farm workers are foreign-born (72%) and predominantly Mexican (68%). Estimates suggest there are approximately 1.0 to 1.8 million hired farmworkers. Five states account for 65% of all farm workers: (1) California, (2) Florida, (3) Washington, (4) Oregon, and (5) North Carolina. Of the migrant and seasonal farmworkers, 52% have legal authorization to work in the United States.

The most important predictor of overall mortality is: a. race. b. age. c. gender. d. income.

ANS: B The mortality curve by age drops sharply during and after the first year of life to a low point in childhood, then begins to increase through adolescence and young adulthood and then increases sharply through middle and older ages. Race, gender, and income are not the most important predictor for overall mortality.

The nurse is involved in a conflict resolution situation with the parents of a 2-year-old boy. The parents are deciding if "spanking" the child is a disciplinary method that they will employ. The mother says, "I do not believe in spanking. I see it as abusive and demeaning." Which of the following best describes the statement by the mother? a. Cooperation b. Assertiveness c. Bargaining d. Collaboration

ANS: B The mother's statement is an example of assertiveness, the ability to present one's own needs. Cooperation is the ability to understand and meet the needs of others. Bargaining is part of the negotiation process when debates include gathering facts based on reasoning that will generate understanding and promote relearning. Collaboration is a dynamic highly interactive and interdependent process in which people work together to share resources and a vision for a goal.

A nurse is working with a family who has an income above the federal income guidelines, but is unable to meet their living expenses. Which of the following terms best describes this family? a. Persistent poverty b. Near poor c. Impoverished d. Neighborhood poverty

ANS: B The near poor are persons and families whose income is above the federal income guidelines but insufficient to meet living expenses. Persistent poor refers to individuals and families who remain poor for long periods and who pass poverty on to their descendants. Neighborhood poverty refers to geographically defined areas of high poverty, characterized by run-down housing, high unemployment rates, and poorer health outcomes. Impoverished refers to those living in poverty.

One case of smallpox occurs in a population in which it was considered to be previously eliminated. Which of the following best describes what has occurred? a. Endemic b. Epidemic c. Pandemic d. Infectivity

ANS: B The occurrence of one case of smallpox in a population in which it was considered to be previously eliminated is an epidemic. Endemic refers to the constant presence of a disease within a geographic area or population. Pandemic refers to an epidemic occurring worldwide and affecting large populations. Infectivity is the ability to enter and multiply in the host.

The original goal of deinstitutionalization was to: a. reduce state costs of caring for the chronically mentally ill. b. replace state psychiatric hospitals with community-based treatment programs. c. modernize psychiatric hospitals. d. allow mentally ill persons to live on their own to test their life skills.

ANS: B The original goal was to replace large psychiatric hospitals with community-based treatments centers. However, federal and state governments failed to allocate the needed funds.

A nurse has collected data about the services available in the community and is using the seven "A"s to evaluate these services. Which of the following questions will the nurse most likely ask? a. Can the community members allocate resources to support the service? b. Can the community members afford the service? c. Will the community members allow strategies to be implemented to improve the service? d. Will the community members approve of the services provided?

ANS: B The seven "A"s include: Is the community aware of its needs and of the service? Is it accessible to community members? Is the service available when the community needs it? Can the community members afford the service? Does the community find the service acceptable? Is the service adequate to meet the needs of the community? And, are the services appropriate to meet the needs of the community? The following questions do not address the seven "A"s: Can the community members allocate resources to support the service? Will the community members allow strategies to be implemented to improve the service? And, will the community members approve of the services provided?

Which model addresses the structure and processes of using the population-based tools of disease management and critical pathways to offer care for client populations? a. Client-focused b. System-focused c. Social service d. Long-term care

ANS: B The system-focused model addresses the structure and processes of using the population-based tools of disease management and critical pathways to offer care for client populations. The concern in client-focused models is with the relationship between case manager and client to support continuity of care and to access providers of care. The social service models provide services to clients to assist them in living independently in the community and in maintaining their health by eliminating or reducing the need for hospital admissions or long-term care. Long-term care is not a model of case management.

A nurse is examining the future of disaster management. Which of the following should the nurse anticipate? a. Vague and unorganized future due to constant emergence of new disasters b. Increasing sophistication in technology and surveillance c. Lack of involvement by national and state officials d. Decreasing need for public health workers to provide care

ANS: B The terrorist event of September 11, 2001, and the anthrax cases have increased the awareness for the need to plan for disasters. Technology and surveillance will continue to advance. Disaster management continues to change and become more organized as learning occurs after each incident, producing progressive best practices. National and state officials are very involved in disaster management. PHNs are particularly critical members of the multidisciplinary disaster health team given their population-based nursing focus and specialty knowledge in epidemiology and community assessment skills.

When would it be appropriate for a nurse to use a Geographic Information System (GIS)? a. Recording client data collected at a foot clinic b. Determining neighborhoods that have an increased incidence of lead poisoning c. Evaluating effectiveness of a farm safety program d. Scheduling health promotion programs in the community

ANS: B The use of a GIS allows the PHN to apply the principles of epidemiology into practice. GIS allows nurses to code data so that it is related spatially to a place on earth and is helpful in determining concentrated areas for incidence of disease and illness. An electronic health record (EHR) would be the most appropriate place to record client data from a foot clinic. Evaluation of effectiveness of a farm safety program would need to be done through some type of assessment or evaluation method—GIS would not be useful to collect this data. GIS is not a scheduling system and would not be helpful when scheduling health promotion programs in the community.

Acts of bioterrorism or natural disasters, such as earthquakes, will often have more casualties because: a. they cause the most widespread destruction. b. victims have little time to make evacuation preparations. c. those with chronic conditions cannot escape in time. d. the early warning systems are not effective.

ANS: B The use of weapons of mass destruction or natural disasters, such as earthquakes, will often have more casualties because victims have little time to make evacuation preparations. The other answers are false.

The wide variations in health services and health status between certain population groups are called: a. vulnerable population groups. b. health disparities. c. disadvantaged populations. d. risk markers.

ANS: B The wide variations in health services and health status between certain population groups are called health disparities. Vulnerable populations are typically considered to be those who are at greater risk for poor health status and who have poor access to health care. Disadvantaged populations have fewer resources for promoting health and treating illness than does the average person in the United States. Risk describes that some people have a higher probability of illness than others.

A nurse is writing a nursing diagnosis at the community level using the North American Nursing Diagnosis Association (NANDA) system as a guide. Which of the following diagnoses would most likely be developed? a. Risk of hypertension related to poor diet and sedentary lifestyle b. Risk of obesity among school-age children related to lack of opportunities to engage in physical activity c. Risk of ineffective health maintenance among individuals who do not have access to a primary care provider d. Ineffective coping related to multiple stressors, as evidenced by client crying and stating she has no support system

ANS: B There are three parts of the NANDA system: (1) identification of the problem or potential problem; (2) its relation to factors, stressors, or health issues; and (3) supporting data that documents the problem. The "risk of" identifies a specific problem or health risk faced by the community. "Among" identifies the specific community client with whom the nurse will be working in relation to the identified problem or risk. "Related to" describes characteristics of the community.

A nurse refers to the Code of Ethics for Nurses or the Public Health Code of Ethics. Which of the following describes why the nurse has referred to this document? a. To provide answers for ethical dilemmas b. To guide professional practice related to ethics c. To increase moral leadership in ethics d. To find a framework for ethical decision making

ANS: B These codes provide general ethical principles and guide personnel in thinking about the underlying ethics of the profession. They do not provide answers for ethical dilemmas, only serve as a guide. They do not increase moral leadership, nor do they provide a framework for ethical decision making.

A nurse is planning to assist homeless clients at a local shelter with finding community resources to meet their health care needs. Which of the following actions should the nurse take before interacting with this population? a. Change personal views about homelessness b. Understand the concepts of homelessness c. Receive special training about where to locate homeless people d. Have experience in mental health nursing and counseling

ANS: B To implement effective nursing interventions for homeless clients, nurses need to be aware of their own beliefs and values about the homeless as well as understand the concepts of homelessness.

A nurse is trying to provide effective nursing care to poor persons, families, and populations in a variety of settings. Which of the following actions should be taken by the nurse? a. Move to the area where people are living to understand their plight b. Understand the concept of poverty on many levels c. Take a trip abroad to see how poverty exists in other countries d. Become fully immersed in the concept of poverty for at least a year

ANS: B Understanding the concept of poverty with historical, social, political, economic, biological, psychological, and spiritual dimensions is the first step in providing effective care to this population. It is not necessary to move to the area, take a trip abroad, or become immersed in the concept of poverty in order to provide effective nursing care to this population.

A public health nurse (PHN) is working in a community where there have been multiple closures of large factories, leaving many individuals without employment. Which of the following is this population most at risk for? a. Becoming an abuser at home b. Committing violence in and outside the home c. Developing addictive behaviors d. Neglecting the needs of their children

ANS: B Unemployment is associated with violence within and outside the home. Abuse may increase both in and outside of the home due to unemployment. Applying for unemployment benefits is not a risk for this population. Neglecting the needs of their children only addresses what could happen inside of the home and not outside of the home.

A nurse centers his practice around the principle of doing the greatest good for the greatest number. Which of the following ethical principles is being applied? a. Distributive justice b. Utilitarianism c. Social justice d. Health disparities

ANS: B Utilitarianism means doing the greatest good for the greatest number. Distributive justice means treating people fairly, and distributing resources and burdens equitably among the members of a society. Social justice means ensuring that vulnerable groups are included in the equitable distribution of resources. Health disparities are the inequalities that exist among different populations.

A nurse is working with a vulnerable population. Which of the following is the nurse most likely working with? a. Families earning more than $50,000/year b. Families earning less than $15,000/year c. Residents of urban areas d. African American physicians

ANS: B Vulnerable populations of concern to nurses are persons who are poor or homeless (families earning less than $15,000 per year would be considered poor), persons who have special needs, pregnant teens, migrant workers and immigrants, individuals with mental health problems, people who abuse addictive substances, persons who have been incarcerated, people with communicable diseases and those who are at risk, and persons who are human immunodeficiency virus (HIV) positive or have hepatitis B virus or sexually transmitted diseases (STDs).

A nurse is caring for a client who has been diagnosed with West Nile Virus. Which of the following types of illness does the client have? a. Food-borne b. Vector-borne c. Water-borne d. Zoonoses

ANS: B West Nile virus is carried by a mosquito, which is a vector. Food-borne illnesses are carried by food. Water-borne illnesses are transmitted through water. Zoonoses are infections that are transmitted from vertebrate animal to a human under natural conditions.

A public health nurse is examining several issues within daily practice. Which of the following issues would be considered an ethical dilemma? a. Whether or not to establish a community health center in a rural area b. Allocating resources in a natural disaster c. Deciding to withdraw care on a hospice patient d. Applying the principles of Florence Nightingale in Bangladesh

ANS: B When resources are scarce, a dilemma may exist as to how to allocate them. Considering establishing a community health center may be a dilemma, but it probably does not involve ethics. Withdrawing care from a hospice patient would most likely not be an issue encountered by a public health nurse, as this represents community health nursing practice, not public health nursing practice. Applying the principles of Florence Nightingale would not be ethical issue.

Which statement about homelessness is true? a. Homeless persons are not found in rural areas. b. Gentrification can lead to homelessness. c. Deinstitutionalization led to a small number of homeless. d. Community-based clinics provide much needed health care.

ANS: B When urban housing is upgraded, the supply of low-income housing drops. It is often said that when neighborhoods were "gentrified" or upgraded, they reduced the ability of the poor people to live in them because they became unaffordable for poorer former residents. Homeless can be found in both rural and urban areas. Deinstitutionalization of the chronically mentally ill individuals from public psychiatric hospitals in the 1980s increased the number of homeless persons. After this happened, federal and state governments failed to allocate the needed funds to provide community-based services.

A nurse gathers information about the condition of homes, size of lots, neighborhood hangouts, road conditions, and modes of transportation. Which method of data collection is being used? a. Participant observation b. Windshield survey c. Focus group d. Informant interviews

ANS: B Windshield surveys are a method of simple observation, providing a quick overview of a community. By making observations of the community, the nurse is completing a windshield survey. Participant observation refers to the deliberate sharing in the life of a community, such as participating in a local fair or festival, or attending a political or social event. A focus group is similar to an interview in that it collects data mainly through asking open-ended questions to participants, but to a small group rather than an individual. An informant interview is a method of community data collection that involves directed conversation with selected community members.

Which group has the highest rate of violence? a. Older, Caucasian men b. Young, minority men c. Young, Caucasian women d. Older, minority women

ANS: B Young, minority men have the highest rate of unemployment in the United States, ranging upward to 50% even in times of prosperity. Unemployment may precipitate aggressive outbursts. Most analyses conclude that the differential rates of violence between African Americans and Caucasians in the United States have more to do with socioeconomic disparities, such as poverty, unemployment, and overcrowding, than with race.

A nurse is reviewing information about the local health department to prepare for an interview. Which of the following services should the nurse expect the local health department to provide? (select all that apply) A. managing the WIC program B. providing education to achieve community health goals C. coordinating directives from state personnel D. reporting communicable diseases to the CDC E. licensing of registered nurses

ANS: B, C

Which are categories of classifying interventions according to the Omaha System Intervention Scheme? (Select all that apply.) a. Health education b. Case management c. Treatments and procedures d. Direct care services e. Skilled nursing care

ANS: B, C The Omaha System Intervention Scheme is comprised of four broad categories of interventions: (1) teaching, guidance, and counseling; (2) treatments and procedures; (3) case management; and (4) surveillance.

A nurse states that he has incorporated epidemiology into his practice and functions in epidemiologic roles. Which of the following best describes the actions taken by the nurse? (Select all that apply.) a. Policy making b. Collection, reporting, analysis, and interpretation of data c. Environmental risk communication d. Documentation on patient charts and records e. Law enforcement

ANS: B, C, D Collection, reporting, analysis, and interpretation of data, environmental risk communication, and documentation on patient charts and records are examples of the use of epidemiology in practice. Policy making and law enforcement do not apply to epidemiology.

A nurse is assessing for environmental health risks in the community. Which of the following approaches would be most appropriate for the nurse to use? (Select all that apply.) a. Ask legislators to provide a list of environmental pollutants in the area b. Develop a list of exposures associated with urban, rural, or suburban settings c. Assess the risk by medium such as air, water, soil, or food d. Divide the environment into functional locations: home, school, workplace, and community e. Recruit community members to sign a petition to decrease pollution

ANS: B, C, D Developing a list of exposures associated with urban, rural, or suburban settings, assessing the risk by medium such as air, water, soil, or food, and dividing the environment into functional locations: home, school, workplace, and community are ways a nurse can assess the environment. Asking legislators for a list of pollutants or recruiting community members to sign a petition are not effective means to assess for environmental health risks.

A nurse is assessing the safety in the community using primary data. Which of the following data would be useful for the nurse to collect? (Select all that apply.) a. Number of billboards in the area b. Interviews with health care providers who are familiar with the community c. Observation of community members d. Nurse's own observations e. Morbidity and mortality rates

ANS: B, C, D Other nurses, social workers, health care providers, community members, and the nurse's own observations are reliable sources of information about the safety of an area. The number of billboards would not be important data to collect when assessing for safety in the community. Morbidity and mortality rates would be considered secondary data.

Which are the objectives of a disease investigation? (Select all that apply.) a. Enhance the knowledge of the nurse working in public health b. Control and prevent disease or death c. Identify factors that contribute to the disease outbreak/event occurrence d. Implement measures to prevent occurrences e. Participate in the core functions of public health

ANS: B, C, D The objectives of an investigation are to control and prevent disease or death, identify factors that contribute to the disease outbreak/event occurrence, and implement measures to prevent occurrences. Enhancing the knowledge of the nurse working in public health and participating in the core functions of public health are not objectives of an investigation.

Why is it difficult to know the exact number of homeless persons? (Select all that apply.) a. Homeless people refuse to provide demographic information. b. Many homeless people refuse to be interviewed. c. It is difficult to generalize from one location to another. d. Some persons experience short intervals of homelessness. e. Homeless individuals are most likely hiding within the community.

ANS: B, C, D Those who are homeless are hard to find since they may sleep in boxcars, on building roofs, in doorways, or under freeways or pedestrian overpasses; temporarily stay with friends or family; refuse to be interviewed or deliberately hide the fact that they are homeless; have only short or intermittent episodes of being without a home; or may not fit in a general category.

A nurse is employed by an agency that addresses global health needs. Which of the following roles is the nurse most likely to provide? (Select all that apply.) a. Medications and vaccines for worldwide use b. Direct patient care for members of the community c. Knowledge and skill in countries where nursing is an organized profession d. Consultation to auxiliary personnel e. Facilitation of education and health promotion within the community

ANS: B, C, D, E Nurses perform many roles related to global health, including providing direct patient care, facilitating education and health promotional needs of the community, providing knowledge and skill in countries where nursing is an organized profession, and providing consultation to auxiliary personnel. Providing medications and vaccines for worldwide use would not be part of the role of the nurse in relation to global health.

During the twentieth century, a nurse participated in research that examined the epidemiology of various diseases. Which of the following best describes the factors that influenced these studies? (Select all that apply.) a. Increasing rate of poverty b. Declining child mortality rates c. Overcrowding in major cities d. Development of new vaccinations e. Advancements in medical equipment

ANS: B, D Factors contributing to the development and application of epidemiologic methods in the twentieth century were: improved nutrition, new vaccines, better sanitation, the advent of antibiotics and chemotherapies, and declining infant and child mortality and birth rates. A rise in the standard of living occurred for many following the Great Depression and World War II. The advancements in medical technology have not influenced the examination of the epidemiology of various diseases.

A nurse will be using an interpreter during a client encounter. Which of the following considerations should be made by the nurse? (Select all that apply.) a. It is appropriate to use family members as interpreters. b. Written materials should be available in the client's primary language. c. Observe the interpreter's gestures to assure client understanding. d. The gender, age, and educational level of the interpreter should be evaluated. e. The nurse should face the interpreter when speaking.

ANS: B, D Family members should be used with caution. The client's gestures and nonverbal messages should be observed to assure understanding. Written materials should be available in the client's primary language. The gender, age, educational level, socioeconomic status, religion, and dialect should all be considered when selecting the proper interpreter. The nurse should face the client during the dialogue, not the interpreter.

A community health nurse is developing strategies to prevent or improve mental health issues in the local area. In which of the following situations is the nurse implementing a tertiary prevention strategy? A. providing support programs for new parents B. screening a client whose partner recently died for suicide risk C. teaching a client who has schizophrenia about medication interactions D. discussing stress reduction techniques with employees at an industrial site

ANS: C

A nurse is planning measures to reduce the incidence of obesity. Which of the following interventions affects the environment, according to the epidemiological triangle? A. determine whether the clients have a family history of obesity B. measure clients' BMI C. provide low-fat meal options at public schools D. ask affected adults to keep a diary of food intake

ANS: C

A nurse is preparing a community health program on communicable disease. when discussing modes of transmission, the nurse should include which of the following illnesses as airborne? A. cholera B. malaria C. influena D. salmonellosis

ANS: C

A school nurse is scheduling visits with a physical therapist for a child who has cerebral palsy. In which of the following roles is the nurse functioning? A. direct caregiver B. consultant C. case manager D. counselor

ANS: C

A nurse has discovered that a client should be receiving Meals on Wheels. Which of the following steps of the nursing process is the nurse performing? a. Diagnosis b. Planning c. Assessment d. Implementation

ANS: C The Standards of Care and the Standards of Professional Performance both include the six steps of the nursing process: (1) assessment, (2) diagnosis, (3) outcomes identification, (4) planning, (5) implementation, and (6) evaluation. As a nurse performs an assessment, the nurse recognizes the current needs of the client.

Which example contains the components necessary to form an epidemiologic triangle? a. Pesticides, water, food b. Lead, mercury, soil c. Trichloroethylene, water, infants d. Children under 12, elderly, temperature

ANS: C The epidemiologic triangle consists of an agent (chemical), host (community consisting of several variants), and environment (air, water, soil, etc.). The other examples do not contain an agent, host, and environment.

A nurse is providing preconceptual counseling to a woman. Which of the following supplements would be recommended to help prevent neural tube defects? a. Iron b. Calcium c. Folic acid d. Vitamin C

ANS: C 400 µg of folic acid are recommended daily. Supplements of iron and Vitamin C are not recommended during preconceptual counseling. Supplements of calcium may be recommended to prevent bone loss, but not for preconceptual counseling.

The type of epidemiologic study that is used to describe a group of persons enrolled in a study who share some characteristic of interest and who are followed over a period of time to observe some health outcome is a(n): a. case-control study. b. cross-sectional study. c. cohort study. d. experimental study.

ANS: C A cohort study is the type of epidemiologic study that is used to describe a group of persons enrolled in a study who share some characteristic of interest and who are followed over a period of time to observe some health outcome. A case-control study uses a sample from the cohort rather than following the entire cohort over time. A cross-sectional study provides a snapshot of a population or group at one point in time. An experimental study is one in which the investigator initiates some treatment or intervention that may influence the risk or course of the disease.

A nurse responds to a natural disaster. Which of the following is the nurse most likely responding to? a. Transportation accident b. Pollution c. Communicable disease epidemic d. Fire

ANS: C A communicable disease epidemic would be considered a natural disaster. Transportations accidents, pollution, and fire are considered human-made disasters.

Voters have recently decided to have fluoride added to the city water system. Epidemiologists now want to study the effect of fluoride on dental caries in this population. Which of the following would be conducted by the epidemiologists? a. Ecological study b. Double-blind study c. Community trial d. Screening

ANS: C A community trial is similar to a clinical trial, but the issue is often health promotion and disease prevention rather than treatment of existing disease. An ecological model considers the multiple factors that contribute to disease development. A double-blind study is one in which neither the subject nor the investigator knows who is receiving the treatment. A screening involves the testing of groups of individuals who are at risk for a certain condition but are not yet symptomatic.

Which is referred to as a lesser-developed country? a. France b. Japan c. Indonesia d. Sweden

ANS: C A country that is not yet stable with respect to its economy and technological development is lesser developed, which describes Indonesia. France, Japan, and Sweden are considered developed countries.

A nurse is using a critical path when providing care for a client. Which of the following most likely describes the situation that the nurse is in? a. Implementing the six "rights" of case management b. Demonstrating competencies required for practicing case management c. Attempting to achieve a measurable outcome for a specific client d. Effectively managing conflict resolution

ANS: C A critical path is a case management tool used to achieve a measurable outcome for a specific client case. The critical path details the essential and sequential activities in care, so that the expected progress of the client was known at a point in time. A critical path does not address the six "rights" of case management or demonstrate the competencies required for practicing case management. A critical path is not used for conflict resolution.

A nurse builds a vacation home in a remote area having fewer than six people per square mile so he can "get away from it all." Which of the following best describes the classification of this area? a. Standard metropolitan statistical area b. Metropolitan county c. Frontier d. Rural area

ANS: C A frontier region has fewer than six people per square mile. A standard metropolitan statistical area is a region with a central city of at least 50,000 residents. A metropolitan county describes regions with a central city of at least 50,000 residents. Rural describes communities having less than 20,000 residents or fewer than 99 persons per square mile.

A nurse is analyzing one of the components of a unit of DALYs. From which of the following individuals would the nurse be able to calculate this measurement? a. A woman who lives to be 100 years old b. A woman who contracts HIV after working as a prostitute c. A father with five children who dies at the age of 50 from malaria d. A woman who gives birth to a premature infant

ANS: C A male who dies from malaria at age 50 would represent 30 DALYs. Measurement is based on the potential limit for life, which has been set at 82.5 years for women and 80 years for men. In order to calculate this statistic, the nurse must know the client's age and the potential limit for life.

A nurse is caring for a client who is considered obese. Which of the following best describes this client? a. An individual with a body mass index (BMI) of 15 b. An individual with a BMI of 20 c. An individual with a BMI of 30 d. An individual with a BMI of 25

ANS: C A person is considered obese when the BMI reaches 30. A person is considered overweight with a BMI of 25 to 29.9. A BMI of 18.5 to 24.9 is considered of healthy weight. An individual with a BMI of less than 18.5 is considered to be underweight.

A college student goes to the Student Health Center with an extremely swollen neck. The advanced practice nurse determines that he has mumps. His roommate also is experiencing malaise and a sore throat. Which of the following defines these two students? a. Mixed outbreak b. Common source c. Point source d. Propagated outbreak

ANS: C A point source outbreak involves all persons exposed becoming ill at the same time, during one incubation period. A mixed outbreak is a common source followed by secondary exposures related to person-to-person contact. A common source outbreak refers to a group exposed to a common noxious influence such as the release of noxious gases. A propagated outbreak does not have a common source and spreads gradually from person to person over more than one incubation period.

A population is best defined as a: a. high-risk group. b. those interacting within a school or institutional setting. c. collection of individuals who share at least one common characteristic. d. geographical location within a community.

ANS: C A population or aggregate is a collection of individuals who have one or more personal or environmental characteristics in common. High risk groups, school or institutional setting, and geographical location within the community all describe types of populations, not the definition of population.

A nurse is caring for a client who is experiencing a drug addiction. Which of the following is the client most at risk for having? a. Infant with a high birth weight b. Easily cured illness or disease c. Accident or committing suicide d. Problem finding employment

ANS: C ATOD abuse and addiction is associated with many problems, including neonates with low birth weights and congenital anomalies; accidents, homicides, and suicides; chronic diseases such as cardiovascular disease, cancer, lung disease, hepatitis, and human immunodeficiency virus (HIV)/acquired immune deficiency syndrome (AIDS); and mental illness.

A nurse is using the principles of virtue ethics in decision making. Which of the following describes the action that the nurse would take? a. Provide efficient and effective nursing care b. Identify the meaningful facts in the situation c. Seek ethical community support to enhance character development d. Plan ways to restructure the social practices that oppress women

ANS: C According to Aristotle, virtues are acquired and include interest in the concept of the good, including benevolence, compassion, trustworthiness, and integrity. One part of the process is seeking ethical community support to enhance character development. Nurses can demonstrate advocacy when providing efficient and effective nursing care. Identifying the meaningful facts in the situation is part of the ethical decision-making process. Planning ways to restructure the social practices that oppress women is part of the feminist ethics decision-making process.

The percentage of U.S. residents living in rural settings is about: a. 5%. b. 15%. c. 20%. d. 40%.

ANS: C According to the Bureau of the Census estimates, almost 20% of all U.S. residents live in rural settings.

A home care agency has applied for accreditation from the Joint Commission. What is the next step the agency will take? a. Attend a conference to learn more about the accreditation process b. Schedule a site visit with the Joint Commission c. Complete a self-study of the agency d. Improve methods of documentation of client visits

ANS: C After applying for accreditation, a lengthy self-study must be completed that addresses all aspects of the agency's operation. Following completion of the self-study, an accreditation team schedules a site visit. Attending a conference to learn more about accreditation should be done before applying. A site visit with the Joint Commission occurs after the self-study has been reviewed. Improving methods of documentation of client visits should be addressed before applying for accreditation.

A large amount of data related to hypertension rates is collected in a community. The data is analyzed. Which of the following describes the next step in the surveillance process? a. Collecting data from multiple valid sources b. Evaluating the impact on the surveillance system c. Interpreting the data and disseminating it to decision makers d. Asking political officials to finance a hypertension clinic

ANS: C After data is collected and analyzed, the findings must be disseminated. Collection of data is the step described in this scenario and the question asks what must be done next. Evaluating the impact on the surveillance system occurs after dissemination. Dissemination may occur to a broader audience than only political officials.

What effect does alcohol consumption have on migrant farm workers? a. Provides relaxation and is important for migrant workers' well-being b. Adds additional risks to pesticide exposure c. Poses safety hazards for farm workers d. Allows migrant workers to sleep better, thus improving daytime productivity

ANS: C Alcohol can contribute to farm-related injuries. Drug and alcohol use has been identified as a significant source of stress. Alcohol can also contribute to health problems, greater risk for human immunodeficiency virus (HIV), violence in camps/home sites, domestic violence, and decreased funds for personal and family needs.

A nurse is assessing persons arriving at an alternate care center following a disaster. Which of the following actions should the nurse take first? a. Assess the amount of equipment and medications needed for each client b. Determine if the client has a psychological condition requiring special attention c. Assess whether this type of facility is appropriate for the client d. Determine if the client has a support system to assist with additional care needs

ANS: C Alternate care centers may be used to shelter patients with medical needs designated as "non-ambulatory care/hospital overflow" e.g., care of non-ambulatory patients with less intense medical needs. After determination of the appropriateness of the facility, the nurse can then provide medical care as needed. It should not be necessary to initially determine if the client has a psychological condition or limit the amount of equipment or medications.

A nurse is providing education for a client who is living in an area of the world where malaria is endemic. Which of the following interventions would be appropriate to include? a. Receive a dose of intravenous (IV) quinine b. Avoid direct contact with individuals who have malaria c. Use insecticide-treated bed nets d. Apply dichlorodiphenyltrichloroethane (DDT) spray

ANS: C An effective antimalarial intervention is the use of insecticide-treated bed nets. Malaria is contracted by being bit by an infected mosquito; it cannot be passed by direct person-to-person contact. IV quinine is the drug of choice for treating malaria, not preventing it. Some mosquitoes are resistant to DDT, so this may not be effective.

Medicare-certified home health agencies place emphasis on _____ care. a. chronic b. distributive c. intermittent d. primary

ANS: C As part of the Conditions of Participation, Medicare-certified home health agencies must place an emphasis on intermittent care. Home care services that are provided must be intermittent and provide a skilled service. Primary care refers to the care that is often provided in a physician's office. Medicare-certified agencies provide care following an acute hospitalization or medical change; they do not provide long-term care for chronic illnesses.

A nurse is providing maternal-child care for rural women. The nurse would like to intervene with pregnant women who are at risk for poor health outcomes. Which of the following populations would be the target population for the nurse's care? a. Latinos who live in urban environments b. Caucasians who live in remote areas c. Those who live on or near Indian reservations d. Those who seek health from general practitioners

ANS: C As per the National Center for Farmworker Health, pregnant women who are particularly at risk are those who live on or near Indian reservations, migrant workers, and African American women who live in the deep South. Latinos living in urban environments, Caucasians living in remote areas, and those receiving health care from general practitioners are not as much of a concern in comparison to the Indian population.

A nurse fulfills the environmental health competency of "assessment and referral." Which of the following actions has the nurse most likely completed? a. Advocating for public policy changes b. Understanding policy framework and major pieces of legislation c. Completing an environmental health history d. Describing the scientific principles about environmental health

ANS: C Assessment and referral states that all nurses should be able to successfully complete an environmental health history, recognize the potential environmental hazards and sentinel illnesses, and make appropriate referrals for conditions with probably environmental causes. Advocating for public policy changes is part of advocacy. Understanding policy framework and legislation is part of legislation and regulation. Describing the scientific principles of environmental health is part of basic knowledge and concepts.

A nurse provides for the availability of essential personal health services for people who would otherwise not receive health care. Which of the public health core functions is being used? a. Assessment b. Prevention c. Assurance d. Policy development

ANS: C Assurance deals with the availability of health services. Prevention is not a core function, assessment refers to systematic data collection, and policy development refers to the need to provide leadership in developing health policies.

A public health department makes sure that the essential community-oriented health services are available in the community. Which of the following core public health functions is being implemented? a. Policy development b. Assessment c. Assurance d. Scientific knowledge-based care

ANS: C Assurance focuses on the responsibility of public health agencies to ensure certain activities have been appropriately carried out to meet public health goals and plans. Policy development seeks to build constituencies that can help bring about change in public policy. Assessment includes activities that involve collecting, analyzing, and disseminating information on both the health status and the health-related aspects of a community or a specific population. Public health is based on scientific knowledge, but is not a core function.

Public health administrators in a community provide a health department to serve an indigent population of immigrants providing translators on certain days of the week. Which of the following best describes what is being done? a. Policy development b. Quality c. Assurance d. Libertarian philosophy

ANS: C Assurance refers to the role of public health in making sure that essential community-oriented health services are available, which may include providing essential personal health services for those who would otherwise not receive them. Policy development refers to the need to provide leadership in developing policies that support the health of the population, including the use of the scientific knowledge base in making decisions about policy. Quality refers to providing the best care possible. Libertarian philosophy refers to the view that the right to private property is the most important right.

A nurse is assessing an individual who has been the victim of domestic violence. Which part of the body should the nurse assess first? a. Front of the body b. Back of the body c. Proximal parts of the body d. Distal parts of the body

ANS: C Attacks are often inflicted on proximal parts of the body that can easily be disguised by clothing, so the nurse should assess this area first.

A community is concerned about the threat of bioterrorism. Which of the following best describes the basis for this concern? a. Bioterrorism has the potential to dissolve community-based programs. b. This threat could cause the health care system to collapse. c. The threat of bioterrorism may divert funds from other public safety health care programs. d. Fear of bioterrorism will increase the need for shelters.

ANS: C Bioterrorism may have an impact on the availability of resources for public safety health care programs. Because funds are diverted it is possible that community-based programs would be eliminated, the health care system could experience changes, and that there would be an increase in the need for shelter. However, all of these things would happen because of the diversion of funds.

A child has reported to the school nurse that he is being bullied. Which of the following is most likely happening to this child? a. Difficulty talking to the guidance counselor b. Inclusion in group activities c. Intimidation by a peer d. Discipline from parents at home

ANS: C Bullying has become a major problem in schools. Bullying can be physical and/or psychological abuse, intimidation, or verbal abuse; the exclusion of some children in group activities is another form of bullying.

A nurse wants to obtain information on the alternative methods of health care her 45-year-old female client uses. Who would be the best person to ask about this? a. The husband of the client b. A community leader of the ethnic group c. The client herself d. The religious leader of the ethnic group

ANS: C Clients provide a rich source of information about their own cultures. The client would be the preferred person to collect this information instead of the husband, community leader, or religious leader.

A nurse lives in a community that demonstrates commitment. Which of the following would most likely be supported within this community? a. Creation of a community club by the city council to facilitate community involvement b. Collaboration among area restaurant owners to develop healthier menus c. Collaboration with the health department to build a new recreation center d. Development of a cooperative agreement with a neighboring city to share needed services

ANS: C Commitment to the health of the community requires a process of change at each appropriate level on the continuum. The most successful change processes often arise from collaborative practice models that involve the community and nurses in joint decision making. Participants must see themselves as part of a group effort and share in the process, beginning with planning and including decision making. Collaboration by restaurant owners to develop healthier menus only addresses one population of the community and does not show involvement by nurses and the larger community. Creation of a community club by the city council does not engage the larger community in this decision. Developing a cooperative agreement with a neighboring city does not show commitment between the members of both communities.

The tendency to ignore all differences between cultures and to act as though the differences do not exist is defined as: a. cultural conflict. b. culture shock. c. cultural blindness. d. cultural imposition.

ANS: C Cultural blindness occurs when people state that everyone is treated the same, regardless of their cultural orientation. Cultural conflict is a perceived threat that may arise from a misunderstanding of expectations when nurses are unable to respond appropriately to another individual's cultural practice because of unfamiliarity with the practice. Culture shock is the feeling of helplessness, discomfort, and disorientation experienced by an individual attempting to understand or effectively adapt to a cultural group whose beliefs and values are radically different from the individual's culture. Cultural imposition is the act of imposing one's cultural beliefs, values, and practices on individuals from another country.

A nurse demonstrates cultural desire to provide culturally competent care. Which of the following actions would be taken by the nurse? a. Relying on a textbook for information about an ethnic group b. Bringing a translator to the local community clinic c. Taking Spanish classes in the evening at a local college d. Judging others using his or her own cultural values

ANS: C Cultural desire refers to the nurse's intrinsic motivation to want to engage in the elements necessary to provide culturally competent care. The activity that suits the definition of cultural desire is one that a nurse would want to do instead of being directed to do so, referring to the intrinsic motivation of the nurse. Relying on a textbook for information, bringing a translator, and judging others do not demonstrate the definition of cultural desire as they are not demonstrating the intrinsic motivation of the nurse.

A nurse supports the use of traditional home remedies in conjunction with Western medicine to promote healthy behaviors. Which of the following is being demonstrated by the nurse? a. Cultural accommodation b. Cultural awareness c. Cultural preservation d. Cultural repatterning

ANS: C Cultural preservation refers to assistive, supportive, facilitative, or enabling nurse actions and decisions that help the clients of a particular culture to retain and preserve traditional values, so they can maintain, promote, and restore health. Cultural accommodation refers to assistive, supportive, facilitative, or enabling nurse actions and decisions that help clients of a particular culture accept nursing strategies or negotiate with nurses to achieve satisfying health care outcomes. Cultural awareness refers to the self-examination and in-depth exploration of one's own biases, stereotypes, and prejudices as they influence behavior toward other cultural groups. Cultural repatterning refers to assistive, supportive, facilitative, or enabling nurse actions and decisions that help clients of a particular culture to change or modify a cultural practice for new or different health care patterns that are meaningful, satisfying, and beneficial.

A nurse resigns from a position in a hospital to accept a job in a community setting. After starting the new job, the nurse feels helpless and confused, wondering if this was the right career choice. Which of the following terms best describes how the nurse is feeling? a. Cultural conflict b. Cultural relativism c. Culture shock d. Culture brokering

ANS: C Culture shock can happen to individuals within their own culture when they are having experiences such as starting a new job or career. Culture shock is brought on by anxiety from losing familiar signs and symbols of social interaction. Feelings associated with culture shock are helplessness, discomfort, and disorientation. Cultural conflict is a perceived threat that may arise from a misunderstanding of expectations when nurses are unable to respond appropriately to another individual's cultural practice because of unfamiliarity with the practice. Cultural relativism recognizes that clients have different approaches to health, and that each culture should be judged on its own merit and not on the nurse's personal beliefs. Culture brokering is advocating, mediating, negotiating, and intervening between the client's culture and the biomedical health care culture on behalf of clients.

What does each state do with the information that it receives about notifiable diseases? a. Uses the information for surveillance purposes b. Reports the information to the local branch of the World Health Organization (WHO) c. Transmits the data electronically, weekly, to the Centers for Disease Control and Prevention (CDC) d. Does nothing with the information

ANS: C Data is transmitted weekly to the CDC through the National Electronic Telecommunications System for Surveillance. States need to transmit the information to the CDC rather than only keeping its own records. The reports are sent to the CDC, not to the WHO.

A nurse is caring for a client who has designated someone else to make health care decisions when he or she is unable to do so. Which of the following is being used by the client? a. An advanced medical directive b. A living will c. A durable medical power of attorney d. The Patient Self-Determination Act

ANS: C Durable medical power of attorney is the legal way for a client to designate someone else to make health care decisions when he or she is unable to do so. An advanced medical directive includes both a living will and a durable medical power of attorney. A living will allows the client to express the wishes regarding the use of medical treatments in the event of a terminal illness. The Patient Safe Determination Act requires those providers receiving Medicare and Medicaid funds to give clients written information regarding their legal options for treatment choices if they become incapacitated.

A nurse is working with a client during a smoking cessation program. The client has developed an action plan for smoking cessation, but has not yet implemented it. Which stage of change is the client experiencing? a. Precontemplation b. Contemplation c. Preparation d. Action

ANS: C During preparation, the individual is prepared for action and may reduce the problem behavior but has not taken effective action yet. During contemplation, the individual is aware that a problem exists and is seriously thinking about overcoming it but has not yet made a commitment to take action. The nurse can encourage the individual to weigh the pros and cons of the problem and the solution to the problem. In precontemplation, the person does not intend to change in the foreseeable future. In the action stage, the individual modifies the behavior, experiences, or environment to overcome the problem.

A person is already homeless and is receiving shelter. Which type of housing is being used? a. Low income b. Supportive c. Emergency d. Adequate

ANS: C Emergency housing is shelter for persons who are already homeless. These shelters typically are only open at night and in many cities only on extremely cold nights.

The nurse teaches food handlers to wash utensils after contact with raw meat. Which of the following best describes the focus of this education? a. Agent b. Host c. Environment d. Food handler

ANS: C Environmental factors facilitate the transmission of an infectious agent from an infected host to other susceptible hosts. Teaching food handlers to wash utensils after contact with raw meat is a prevention that focuses on the environment. An agent is described by its ability to cause disease and the nature and the severity of the disease. The four major categories of agents are: (1) bacteria, (2) parasites, (3) fungi, and (4) viruses. A food handler is an example of a host, which is a human or animal that can harbor an infectious agent.

Epidemiology: a. is a science that studies the poisonous effects of chemicals. b. explains the association between learning disabilities and exposure to lead-based paint at the cellular level. c. helps nurses understand the strength of the association between exposure and health effects. d. is a method for tracking the prevalence of health outcomes.

ANS: C Epidemiology studies the incidence and prevalence of disease, helping nurses understand the strength of the association between exposure and health effects. Toxicology is the science that studies the poisonous effects of chemicals. Environmental surveillance provides data with which to track and analyze the incidence and prevalence of health outcomes. Epidemiology does not examine causes at the cellular level.

Which of the following statements about public health is accurate? a. Prevention of early deaths can be more effectively accomplished by medical treatment than by public health approaches. b. Expenditures and resources for public health have increased in recent years. c. Historically, gains in the health of populations have been related largely to changes in safety, sanitation, and personal behavior. d. Reform of the medical insurance system is the single change needed to improve the health of Americans.

ANS: C Fielding and Tilson have asserted that most of the increase in life span has been made through improvements in sanitation, clean water supplies, making workplaces safer, improving food and drug safety, immunizing children, and improving nutrition, hygiene, and housing. Medical treatment has not made as significant of an impact on the life span as public health measures. Funding for public health in recent years has been on a gradual decline. Although reform of the medical insurance system may help improve the health of Americans, there are many other factors that will need to be addressed.

Which data source provides information about the function of the community? a. Local restaurant b. Elected officials c. County health department d. Civic groups

ANS: C Function refers to the aims and activities of the community. Civic groups and elected officials refer to people. Local restaurant refers to a place.

Which of the following characteristics indicates a man is at a higher risk for developing prostate cancer? a. Being of Caucasian descent b. Has not had a prostate-specific antigen (PSA) test c. Has a father or brother who has had prostate cancer d. Has benign prostatic hypertrophy (BPH)

ANS: C Having a father or brother who has had prostate cancer places a man at higher risk for developing prostate cancer. Being of African American descent places a man at higher risk for developing prostate cancer. The PSA test is not accurate in terms of sensitivity or specificity, but it is used as a screening test. BPH can cause an elevated PSA test result.

The government is preparing for a bioterrorism event. Which of the following would be the best strategy to use? a. Threaten lesser-developed countries so they don't develop biological weapons b. Ban biological weapons worldwide c. Effectively detect pathogens and manage services d. Develop antidotes for biological agents

ANS: C Health care professionals and public health officials need to be able to effectively detect pathogens, manage services, and communicate during a bioterrorism threat. Any country can develop biological weapons that can be used for bioterrorism, not only lesser-developed countries. By effectively detecting pathogens, one is able to know what antidotes need to be developed.

The PHN analyzes data related to the number and type of United States Environmental Protection Agency air quality standards that a community failed to meet. This data is an example of using which community health profile indicator? a. Sociodemographic characteristics b. Health status c. Health risk factor d. Functional status

ANS: C Health risk factors discuss proportions of populations who have particular health conditions or health risks; breathing air of poor quality is a risk to health. The number and proportion of people is a sociodemographic characteristic. Health status includes birth and death rates. Functional status refers to reports of good health status by members of the population.

A hospital is using surveillance. Which of the following describes the rationale for this action? a. To protect the public against isolated patients b. To eliminate pathogens from the environment c. To improve quality of care and outcomes d. To decrease the incidence of ventilator-acquired pneumonia

ANS: C Hospital surveillance is used to improve quality of care and outcomes. An example is an infection that occurs in patients who had procedures at that facility. Reduction of the incidence of ventilator-acquired pneumonia is one reason why a hospital uses surveillance, but it does not address the entire issue. Hospital surveillance is unable to eliminate pathogens from the environment and cannot protect the public against isolated patients.

Which of the following statements about health disparities is accurate? a. African American women have a much higher incidence of breast cancer. b. Health disparities are the result of negligent health care practices. c. Health disparities vary among racial/ethnic groups. d. Men have more obstacles to receiving health care than women.

ANS: C Improvements have been made in some areas of health disparities among various populations. However, health disparities vary among each racial/ethnic group. African American women have much higher death rates from breast cancer, not necessarily a higher incidence. The awareness of health disparities remains low among the general public. These health disparities are typically not the result of negligent health care practices.

A nurse is assessing a client for addiction. Which of the following primary symptoms of addiction should the nurse recognize? a. Anger b. Apathy c. Denial d. Violence

ANS: C In assessing for addiction, the nurse should be aware that a primary symptom of addiction is denial. Anger, apathy, and violence are not considered the primary symptoms of addiction.

Which group is over-represented in rural areas when compared with urban areas? a. African Americans b. Asian-Pacific Islanders c. Caucasians d. Native Americans

ANS: C In general, there are a higher proportion of whites in rural area than in urban areas.

Which statement regarding the effects of homelessness on health is accurate? a. Peripheral vascular disease and hypertension are lessened by this lifestyle. b. The incidence and virulence of infections are decreasing. c. Trauma is a significant cause of death and disability. d. Crowded living conditions result in decreased risk for exposure to infections.

ANS: C Intended and unintended injuries such as traumas are a significant cause of morbidity and mortality in the homeless. Those who are homeless are at greater risk for peripheral vascular disease and hypertension. Because of their poor living conditions, a minor skin injury or infections left untreated can result in widespread infection.

A nurse has promoted the introduction of health care technology into lesser-developed countries, but this has led to less-than-satisfactory results. Which of the following best describes why this has happened? a. People do not want to use newer technology. b. The cost is so high that they cannot afford to use new technology. c. The community may not be ready for this technology. d. Technology is too complex for people in lesser-developed countries.

ANS: C It is not generally a lack of desire that is a barrier to use of the new technological systems, but the lack of knowledge about them. It is essential to conduct needs assessments to learn what a community has, what a community wants, and what it can sustain. Well-intended projects have failed because the most basic needs were not met, nor was recognition given to what resources and services the country could sustain.

Which of the following statements about race is true? a. In the United States, children of biracial parents are usually assigned the race of the father. b. Ethnicity and race are synonymous terms. c. Individuals may be of the same race but of different cultures. d. No social significance is usually placed on race.

ANS: C It is often a misconception that persons of the same race have the same culture. For example, African Americans may have been born in Africa, the Caribbean, North America, or elsewhere and have very different cultures. In the United States, children of biracial parents are usually assigned the race of the mother. Ethnicity is a contrasting term to race. Race is a characteristic that allows for some groups to be separated, treated as superior, and given access to power and other valued resources, while others are treated as inferior and have limited access to power and resources.

A community health nurse visits a home and notices multiple bruises on a 4-year-old child. The mother reports that her husband is an alcoholic and is currently sleeping. What is the most appropriate action for the nurse to take? a. Question the mother about sexual abuse b. Initiate interventions directed at family violence c. Report the case of suspected child abuse to authorities d. Refer the mother to a social worker or Al-Anon group

ANS: C It is required by law that nurses report all cases of suspected child abuse. The mandatory reporting laws protect reporters from legal action on cases that are never substantiated. It is essential that this is the first action to be taken by the nurse. After this action is taken, it may be appropriate to initiate interventions related to decreasing family violence or asking specifically about abuse. Referral of the mother to a social worker would not assist in decreasing the potential abuse that may be occurring.

Which statement about nicotine and smoking is accurate? a. Cigars contain higher doses of nicotine than cigarettes. b. Tolerance to nicotine takes days to develop. c. Sidestream smoke contains more toxic compounds than mainstream smoke. d. The harsher smoke of pipes and cigars is more hazardous than cigarette smoke.

ANS: C It is true that sidestream smoke contains greater concentrations of toxic and carcinogenic compounds than mainstream smoke. Pipes are less hazardous than cigarettes because the harsher smoke discourages deep inhalation. The body quickly develops tolerance to nicotine.

A nurse is using life care planning when working with a client. Which of the following would be the most appropriate time for this to be used? a. When organizing a timeline of life events b. When documenting client information and requests c. When assessing present and future client needs d. When estimating future costs for medical care

ANS: C Life care planning is a customized, medically based document that provides assessment of present and future needs. Typically, a life care plan incorporates medical, financial, psychological, vocational, built environment, and social costs during the remaining life of the client. Life care planning is a tool used in case management. A life care plan assesses the current and future needs of a client for catastrophic or chronic disease over a life span.

The most common vector-borne disease in the United States is: a. malaria. b. yellow fever. c. Lyme disease. d. Rocky Mountain spotted fever.

ANS: C Lyme disease is the most common vector-borne disease in the United States, with over 30,000 confirmed cases and probable cases reported to CDC in 2012. Yellow fever and Rocky Mountain spotted fever are both vector-borne diseases, but are not the most common. Malaria is most prevalent vector-borne disease worldwide.

A nurse is working with colonias. Which of the following considerations should be made by the nurse? a. They have developed roads, transportation, and electrical services. b. They are frequently driven off, only to return to the same area. c. Their settlements have led to an increase in several disease conditions. d. Their contributions have improved the local economy.

ANS: C Many immigrants have settled on unincorporated land, known as colonias, outside the major metropolitan areas in California, Arizona, New Mexico, and Texas. These colonies may not have developed roads, transportation, water, or electrical services. Conditions in these settlements have led to an increase in disease conditions such as amebiasis, respiratory, and diarrheal diseases. Environmental health hazards in the colonias are associated with poverty, poor sanitation, and overcrowded conditions.

A nurse is working in the community during the industrial revolution. Which of the following population groups would have been least likely to receive community assistance? a. Widowed women b. Orphaned children c. Mentally ill d. Injured laborers

ANS: C Mentally ill individuals were considered undeserving poor and did not receive any community assistance. Widowed women, orphaned children, and injured laborers were deserving poor because their poverty was considered to be beyond their control.

The greatest single source of air pollution in the United States is from: a. waste incineration. b. power plants. c. motor vehicles. d. molds.

ANS: C Mobile sources such as cars and trucks are the greatest single source of air pollution in the United States. Waste incineration and power plants are major contributors after motor vehicles. Molds contribute to poor indoor air quality.

A nurse is working in a lesser-developed country and is caring for the women in the local community. Which of the following would be an important consideration for the nurse? a. They are at risk for a vitamin D deficiency. b. They are likely to have one child. c. They are at risk for death during pregnancy and childbirth. d. They are viewed as the leader of the family unit.

ANS: C Most deaths to women worldwide are related to pregnancy and childbirth. Most of these deaths occur in lesser-developed countries. Common nutritional deficiencies in this population are zinc, iodine, vitamin A, folic acid, and calcium. One of the reasons this problem exists is because women are not seen as valued members of society. In developing nations there is a significant incidence of lack of prenatal care during pregnancy and high fertility rates, often due to a lack of access to contraception and other family planning and reproductive health services, as well as cultural belief systems that increase the lifetime risk of maternal death.

A nurse who is studying chronic disease considers the multifactorial etiology of illness. What does this imply? a. Genetics and molecular structure of disease is paramount. b. Single organisms that cause a disease, such as cholera, must be studied in more detail c. Focus should be on the factors or combinations and levels of factors contributing to disease. d. The recent rise in infectious disease is the main focus.

ANS: C Multifactorial etiology implies a focus on combinations and levels of factors. There are many factors to consider other than only genetics or single organisms with multifactorial etiologies. The focus of studying multifactorial etiology is on chronic disease.

Mutual benefit with limited loss for everyone is a goal of: a. negotiating. b. assertiveness. c. conflict management. d. cooperation.

ANS: C Mutual benefit with limited loss for everyone is a goal of conflict management. This involves using skills directed toward learning all parties' needs and desires, detecting their areas of agreement and disagreement, determining their abilities to collaborate, and assisting in discovering alternatives and activities for reaching a goal. The process of moving conflicting parties toward an outcome is called negotiation. Assertiveness is the ability to present one's own needs. Cooperation is the ability to understand and meet the needs of others.

A nurse is using surveillance to collect outcome data. What information would most likely be collected? a. Number of clinic services which use evidence-based protocols b. Proportion of the population vaccinated against influenza c. Incidence of breast cancer in the population d. Probability of a bioterrorism attack occurring in the community

ANS: C Outcome data focus on change in health status; incidence rates are one example of this type of information. Process data focus on what is done, such as services provided or protocols for health care delivery. An example of process data is collection of data about the proportion of the eligible population vaccinated against influenza in any 1 year.

Which is a characteristic of persons living in poverty? a. Longer life expectancy b. Simple health problems c. Higher rates of chronic illness d. Fewer hospitalizations

ANS: C Not only do persons living in poverty have higher rates of chronic illness, but less access to care. They have a shorter life expectancy, more complex health problems, and hospitalization rates greater than those for persons with higher incomes.

A nurse works at the individual level to reduce pollution in the environment. Which of the following actions would most likely be taken by the nurse? a. Provide a tax incentive to factories that do not pollute b. Make laws related to allowed levels of pollution in the area c. Recycle paper, glass, cans, and plastic d. Move to an area with less pollution

ANS: C Nurses can reduce pollution by doing their part, which can include choosing to recycle paper, glass, cans, and plastic. Providing a tax incentive and creating laws would not be completed at the individual level to reduce pollution in the environment. Moving to an area with less pollution would not help to improve the problem.

A nurse is caring for a client who has been diagnosed with a parasitic infection. Which of the following information should the nurse know when caring for this client? a. The medication to prescribe to treat these infections b. The nature and symptoms of all parasitic illnesses c. What specimens to collect and how and when to collect them d. Public policy about parasitic infections

ANS: C Nurses need to be cognizant about what specimens to collect, how and when to collect, and what laboratory techniques to use. Proper specimen collection is necessary so that the clinical diagnosis can be confirmed. It is not necessary for the nurse to know the signs and symptoms for all parasitic infections in order to provide care for the client. Public policy about parasitic infections is not important to know when providing care for the client.

A nurse is demonstrating advocacy in his nursing practice. Which of the following actions best demonstrates this principle? a. Offering a smoking cessation program b. Screening for hypertension c. Lobbying for health care reform d. Conducting home visits

ANS: C Nurses should participate in implementing new directions for health care and help envision these new directions. Nurses can be an important voice in advocating for access to consistent, effective, and efficient health care for all. This is best accomplished by performing interventions at the population level.

A nurse is forced to evaluate his beliefs, values, and knowledge about poverty. Which of the following experiences most likely explains the situation the nurse is experiencing? a. A client asking for a small amount of money at a clinic b. Setting up a homeless shelter c. Making a home visit to a home that is unkempt d. Reading about poverty in a textbook

ANS: C Nurses' behaviors in situations are influenced by their relationship with clients. It is important to evaluate clients and populations in the context of the environment to develop nursing interventions. Reading a book is not adequate. Setting up a homeless shelter or having a client ask for money does not force the nurse to self-reflect as much as when the nurse is in the client's environment.

A nurse is assessing the occupational health risks of a migrant farm worker. Which health risk is the nurse most likely to find? a. Denial of care when going to the emergency room b. Difficulty obtaining affordable health insurance c. Increased exposure to chemicals and work-related injuries d. Episodic violence from farm owners and employers

ANS: C Occupational health risks for migrant farm workers include working conditions, exposure to chemicals, and using machinery. Denial of care and difficulty obtaining health insurance are not occupational health risks. Episodic violence from farm owners is not a known occupational health risk.

A nurse is providing primary prevention when working with the homeless. Which of the following best describes what is being implemented? a. Providing emergency housing aid b. Creating drug and alcohol treatment options c. Offering effective job training programs d. Evaluating comprehensive case management programs

ANS: C Offering effective job training programs is an example of primary prevention—attempting to prevent the problem before it occurs. Emergency housing is secondary prevention. Drug and alcohol treatment and comprehensive case management are tertiary preventions.

A nurse is a political advocate for the migrant population. Which of the following best describes the action being taken by the nurse? a. Upholding immigration laws and deporting illegal aliens b. Passing laws to improve the living conditions of migrant farm workers c. Educating communities about the health problems of migrant farm workers d. Learning to speak Spanish for better communication with migrant farm workers

ANS: C Political advocacy can include educating communities about these individuals. This education helps to provide legislators and policy makers at local, state, and national levels with the information needed to make policy decisions.

A nurse is using a population focus when providing public health nursing care. Which of the following statements best describes the care that is being provided? a. Priority is given to the highest risk population. b. Direct caregiving is limited to preventive measures, such as administration of immunizations. c. Attention is given to the population or community as a whole, regardless of whether they do or do not access the health care system. d. Only populations outside institutional settings are considered.

ANS: C PHNs are concerned with the health of the entire population. Priority is given to the entire population, not just the group at highest risk or those outside of institutional settings. Direct caregiving is not the focus of public health nursing care.

A nurse reads the local community newspaper to gather data about the community. Which method of data collection is being used? a. Informant interview b. Focus group c. Participant observation d. Windshield survey

ANS: C Participant observation refers to the deliberate sharing in the life of a community, such as participating in a local fair or festival, or attending a political or social event. The nurse is deliberately sharing in the life of the community by reading the newspaper. An informant interview is a method of community data collection that involves directed conversation with selected community members. A focus group is similar to an interview in that it collects data mainly through asking open-ended questions to participants, but to a small group rather than an individual. Windshield surveys are a method of simple observation, providing a quick overview of a community.

What are the critical attributes in the definition of community? a. Families, groups, and health organizations b. Health needs, geographical boundaries, and target population c. People, place, and functions d. Populations and health resources

ANS: C People, place, and function are the critical attributes in the definition of community. These attributes are found in most definitions of community. The people are community members or residents; place refers to geographic and time dimensions; and function refers to the aims and activities of the community.

A nurse is addressing the problem of air pollution in the community. Which of the following would be the first step the nurse would take? a. Setting standards b. Monitoring c. Permitting d. Compliance

ANS: C Permitting is a process by which the government places limits on the amount of pollution emitted into the air or water. Environmental standards may describe a permitted level of emissions, a maximum containment level, an action level for environmental cleanup, or a risk-based calculation. Monitoring is an ongoing process after an action has happened. Compliance refers to the processes for ensuring that permit/standard/regulatory requirements are met.

A nurse is counseling a client who has been abusive toward a spouse. Which of the following experiences has the client most likely had in the past? a. Perceived or actual crisis b. History of social isolation c. Previous exposure to violence d. Long-term marital strain

ANS: C Perpetrators of intimate partner violence often believe that violence within an interpersonal relationship is a normal behavior pattern, based on their upbringing, living conditions, and increased stress.

A nurse is caring for a child who lives in extreme poverty. Which of the following types of abuse is this child at greatest risk for? a. Sexual abuse b. Emotional neglect c. Physical neglect d. Child abuse

ANS: C Physical neglect is failure to provide adequate food, proper clothing, shelter, hygiene, or necessary medical care. Emotional neglect is the omission of basic nurturing, acceptance, and caring of a child. Child abuse ranges from violent physical attacks to passive neglect. Sexual abuse is a form of child abuse when a child has had nonconsensual sexual contact of any kind.

The proportion of persons with positive test results who actually have a disease, interpreted as the probability that an individual with a positive test result has the disease, is the: a. sensitivity. b. specificity. c. positive predictive value. d. negative predictive value.

ANS: C Positive predictive value refers to the proportion of persons with positive test results who actually have the disease, interpreted as the probability that an individual with a positive test result has the disease. Sensitivity quantifies how accurately the test identifies those with the condition or trait. Specificity indicates how accurately the test identifies those without the condition or trait. Negative predictive value is the proportion of persons with a negative test who are actually disease free.

A nurse interviews the school nurses in a community to determine their roles in schools because this data is not available. Which of the following processes is the nurse using? a. Photovoice b. Spatial data c. Primary data d. Secondary data

ANS: C Primary data includes using informant interviews, focus groups, and participant observation in order to collect information about a community. Spatial data involves looking at the locations of places within the community. Photovoice, also called photo elicitation, is a community assessment technique in which community members take photos to represent a topic or theme about community health. Secondary sources include published data about the community, such as census data.

A PHN implements a primary prevention intervention in the community. Which of the following is most likely being implemented? a. Pap smear b. Blood pressure screening c. Diet and exercise d. Physical therapy

ANS: C Primary prevention refers to those interventions aimed at preventing the occurrence of disease, injury, or disability. Blood pressure screening and pap smears are secondary prevention interventions. Physical therapy is a tertiary prevention intervention.

A home care nurse is employed by a proprietary agency. What does this mean? a. The agency is exempt from federal income taxes. b. The agency is governed by a board of directors. c. The agency is a profit-making agency. d. The agency is reimbursed primarily by tax funds.

ANS: C Proprietary agencies are profit-making agencies. Proprietary agencies are free-standing, for-profit agencies that are required to pay taxes. Many are part of large chains and now dominate the home care industry.

A nurse implements nursing interventions considering the uniqueness of the person's culture. Which of the following best describes this action? a. Cultural diversity b. Cultural knowledge c. Cultural competence d. Cultural awareness

ANS: C Providing care based on the uniqueness of the client's cultural norms and values is one of the three guiding principles of culturally competent nursing care. Cultural diversity refers to the degrees of variation represented among populations based on lifestyle, ethnicity, race, interest, across place, and place of origin across time. Cultural knowledge refers to the process of searching for and obtaining a sound educational understanding about culturally diverse groups. Cultural awareness refers to the self-examination and in-depth exploration of one's own biases, stereotypes, and prejudices as they influence behavior toward other cultural groups.

A nurse is developing a one-stop service to meet the needs of a vulnerable group. Which of the following would the nurse most likely create? a. Wrap-around services where mental services are linked b. Giving all immunizations on a single clinic visit c. Providing multiple services during a single clinic visit d. Providing free services to the medically indigent

ANS: C Providing multiple services during a single clinic visit makes services more responsive to the combined effects of social and economic stressors. Wrap-around services provide comprehensive health as well as social and economic services, so it would include more than the linkage of mental health services. Administering all immunizations or providing free services would not provide a one-stop shop for all needed services.

Which health problem causes more deaths, illnesses, and disabilities than any other in the United States? a. Tobacco addiction b. Alcohol abuse c. Substance abuse d. Caffeine addiction

ANS: C Substance abuse is the number one national health problem, causing more deaths, illnesses, and disabilities than any other health condition. Tobacco addiction, caffeine addiction, and alcohol abuse are part of substance abuse.

ANS: A Community-oriented nursing emphasizes the prevention of disease and disability. Community-based nursing practice is a setting-specific practice whereby care is provided for clients and families where they live, work, and attend school. Policy development seeks to build constituencies that can help bring about change in public policy. Tertiary care focuses on highly specialized medical care.

ANS: C Public health staff nurses who have a clear understanding of population-focused care improve their effectiveness and efficiency of practice; have professional satisfaction by seeing how individual care contributes to health at the population level; and appreciate the practice of others who are population-focused specialists. PHN specialists are prepared at the master's level and function in an administrative role.

A nurse is working with a population that exhibits a large amount of diversity. The nurse recognizes that skin color of individuals within this population is an example of: a. multiculturalism. b. ethnicity. c. race. d. culture.

ANS: C Race is primarily a social classification that relies on physical markers. Multiculturalism is the blending of diverse cultures. Ethnicity is the shared feeling of peoplehood among a group of individuals and relates to cultural factors, such as nationality, geographic region, culture, ancestry, language, beliefs, and traditions. Culture is a set of beliefs, values, and assumptions about life that are widely held among a group of people and transmitted intergenerationally.

A city council discusses how former city laws promoted segregation in the community years ago. Which of the following was being demonstrated when segregation occurred? a. Prejudice b. Cultural imposition c. Racism d. Stereotyping

ANS: C Racism is a form of prejudice that occurs through the exercise of power by individuals and institutions against people who are judged to be inferior in, for example, intelligence, morals, beauty, and self-worth. Prejudice is the emotional manifestation of deeply held beliefs about a group. Cultural imposition is the act of imposing one's cultural beliefs, values, and practices on individuals from another country. Stereotyping is ascribing certain beliefs and behaviors about a given racial and ethnic group to an individual without assessing for individual differences.

A 35-year-old man from Russia comes to the United States seeking asylum because of religious persecution in his native country. Which of the following best describes this type of immigrant? a. Legal immigrant b. Lawful permanent resident c. Refugee d. Unauthorized immigrant

ANS: C Refugees are people who seek protection in the United States because of fear of persecution in their homeland. Legal immigrants are people who are not citizens but are legally allowed to live and work in United States, usually because they fulfill labor demands or have family ties. Lawful permanent resident is another name for legal immigrants. Unauthorized immigrants may have crossed a border into the United States illegally, or their legal permission to stay in the United States may have expired.

Requirements for disease reporting in the United States are mandated by: a. the Centers for Disease Control and Prevention (CDC). b. federal laws. c. state laws and regulations. d. the World Health Organization (WHO).

ANS: C Requirements for disease reporting in the United States are mandated by state laws and regulations. These diseases are then reported to the CDC. State public health officials collaborate with the CDC to determine which diseases should be nationally notifiable. The federal government and WHO are not involved in disease reporting.

A nurse is assessing a school-age child whose parents have reported recent violent activity from the child that is atypical from this child. Which of the following may have contributed to the child's change in behavior? a. Getting a new pet b. Watching too much television c. Playing violent video games d. Having a new sibling at home

ANS: C Research on violent television and films, video games, and music demonstrates that media violence increases the likelihood of aggressive and violent behavior, both immediately and long-term. Parents and caregivers are critical in monitoring what reaches their children.

Compared with urban Americans, rural residents: a. are more likely to engage in preventive health behavior. b. are less likely to be exposed to occupational and environmental hazards. c. have a higher rate of chronic illness. d. rate their overall health status more favorably.

ANS: C Rural residents in America have a higher rate of chronic illness than urban residents. Nearly half of all rural adults have been diagnosed with at least one of the following: heart disease, chronic obstructive pulmonary disease, hypertension, arthritis and rheumatism, diabetes, cardiovascular disease, or cancer, compared with about one fourth of non-rural adults. In general, people in rural areas have a poorer perception of their overall health and functional status than their urban counterparts. Rural residents are less likely to engage in preventive health behavior and are more likely to be exposed to occupational and environmental hazards.

A nurse is caring for a woman who has been battered for a number of years. Which of the following has the client most likely seen an increase in over the years? a. Self-blame b. Remorse by the man c. Severity of the abuse d. Emotional strength

ANS: C Severity of abuse and its frequency is most likely to increase over time. Both the man and the woman tend to blame the incident on something external. The man's remorse tends to lessen over time.

A nurse is promoting social justice. Which of the following actions would the nurse most likely take? a. Contacting lawmakers about environmental health issues b. Assisting at homeless shelters c. Advocating for policies to improve social conditions d. Serving on a local coalition to prevent obesity

ANS: C Social justice refers to providing equitable care and social supports for the most disadvantaged members of society. Nurses can function as advocates for policy changes to improve social, economic, and environmental factors that predispose vulnerable populations to poor health.

Hispanics tend to believe that the needs of the family take priority over those of the individual. Which of the following types of cultural variations is being demonstrated? a. Communication b. Personal space c. Social organization d. Environmental control

ANS: C Social organization refers to the way in which a cultural group structures itself around the family to carry out role functions. Communication is the means by which culture is shared (verbal and nonverbal). Personal space is the physical distance between two individuals during an interaction. Environmental control refers to the person's relationship with nature and to plan and direct factors in the environment that affect them.

A nurse is told that a screening test has high specificity. Which of the following is the best interpretation of this information? a. The test provides precise and consistent readings. b. The test accurately identifies those with the condition or trait. c. The test accurately identifies those without the trait. d. The test has a high level of false positives.

ANS: C Specificity refers to the test accurately identifying those without the trait. High specificity is needed when rescreening is impractical and when reduction of false positives is important. The test would have a low level of false positives.

A nurse refers a client to an assistance program that requires the individual to find a job and/or enroll in job training program. Which program is being used? a. Aid to Families with Dependent Children (AFDC) b. Medicaid c. TANF d. Women, Infants, and Children (WIC)

ANS: C TANF requires participants to find jobs and/or enroll in job training programs. The current TANF program was formerly called AFDC. Medicaid provides health insurance to poor families, but does not require participants to find jobs. WIC provides food vouchers entitling participants to free nutritious foods and infant formulas from local grocers, but does not require participants to find jobs.

A nurse works with a group of abused women to enhance their levels of self-esteem. Which of the following levels of prevention is being performed? a. Primary level of prevention b. Secondary level of prevention c. Tertiary level of prevention d. Health promotion

ANS: C Tertiary prevention includes those interventions aimed at disability limitation and rehabilitation from disease, injury, or disability. Primary prevention aims to prevent disease and illness before it occurs. Secondary prevention focuses on screening and early detection. Health promotion is not a level of prevention.

A nurse is developing a community-based exercise program for a group of women who have cardiovascular disease. Which of the following levels of prevention is being used? a. Primary prevention b. Secondary prevention c. Tertiary prevention d. Health promotion

ANS: C Tertiary prevention includes those interventions aimed at disability limitation and rehabilitation from disease, injury, or disability. Primary prevention focuses on prevention on disease before it occurs. Secondary prevention aims for early detection and screening. Health promotion is not a level of prevention.

A nurse refers a client with a neuromuscular disease to a vocational rehabilitation program. Which of the following best describes the action of the nurse? a. Primary prevention b. Secondary prevention c. Tertiary prevention d. Health promotion

ANS: C Tertiary prevention includes those interventions aimed at disability limitation and rehabilitation from disease, injury, or disability. Referral of a client with a disease is an example of tertiary prevention. Primary prevention refers to interventions aimed at preventing the occurrence of disease, injury, or disability. Secondary prevention focuses on early detection and prompt treatment of disease, injury, or disability. Health promotion is a specific primary prevention strategy.

The nurse provides direct care services to a stroke victim to avoid complications. Which of the following levels of prevention is being implemented? a. Primary prevention b. Secondary prevention c. Tertiary prevention d. Assessment

ANS: C Tertiary prevention includes those interventions aimed at disability limitation and rehabilitation from disease, injury, or disability. Thus, direct care for a stroke victim is focusing on limiting disability and encouraging rehabilitation. Primary prevention focuses on preventing the disease before it occurs. Secondary prevention focuses on early detection and screening. Assessment is not a level of prevention.

A nurse advises a client who has HIV not to donate blood, plasma, or organs. Which of the following levels of prevention is being used? a. Primary prevention b. Secondary prevention c. Tertiary prevention d. Health promotion

ANS: C Tertiary prevention includes those interventions aimed at disability limitations and rehabilitation from disease, injury, or disability. Primary prevention focuses on prevention on disease before it occurs. Secondary prevention aims for early detection and screening. Health promotion is not a level of prevention.

The nurse provides footwear and gloves to leprosy clients to prevent trauma to their insensitive and deformed hands and feet. Which of the following best describes the intervention used by the nurse? a. Primary level of prevention b. Secondary level of prevention c. Tertiary level of prevention d. Primary health care

ANS: C Tertiary prevention reduces complications through treatment and rehabilitation. Primary prevention seeks to reduce the incidence of disease by preventing occurrence. Secondary prevention seeks to prevent the spread of infection and/or disease once it occurs. Primary health care is considered to be the essential health care services provided by physicians and other health care providers.

A nurse is providing tertiary prevention. Which of the following interventions should be included? a. Education b. Prevention c. Referral d. Detection

ANS: C Tertiary prevention refers to interventions aimed at treatment and rehabilitation. Nurses should know about available community resources for abuse victims and perpetrators. Education and prevention primarily deal with primary prevention strategies. Detection is part of secondary prevention.

A nurse recommends to a school board that other methods besides the "just say no" approach be considered when providing drug use prevention education to the elementary students. Which of the following best describes the rationale for this recommendation? a. Children do not like to hear the word "no," and the approach is too negative. b. Project DARE is more effective and has decreased drug use. c. Children are naturally curious and may experiment with drugs. d. "Just say no" to drugs does not include smoking and alcohol abuse.

ANS: C The "just say no" approach does not help young people because children are naturally curious and drug experimentation is often a part of normal development, children from dysfunctional homes often use drugs to get attention and escape intolerable environments, and it does not address the powerful component of peer pressure. Most recent studies have found that the DARE program is less effective than other interactive prevention programs and may even result in increased drug use.

A client is self-employed as a mechanic and has no health insurance coverage. Which of the following best describes the legislation that will assist this client in obtaining health insurance? a. Balanced Budget Act b. Health Insurance Portability and Accountability Act (HIPAA) c. Patient Protection and Affordable Care Act (ACA) d. Social Security Act

ANS: C The ACA of 2010 provides the opportunity for all to purchase health insurance. The Balanced Budget Act shifted payment in home health care. The HIPAA was intended to help people keep their health insurance when moving from one place to another. The Social Security Act created the largest federal support program for elderly and poor Americans.

A nurse is providing information to a local newspaper about the presence of infectious diseases in the United States. Which of the following statements by the nurse is accurate? a. "It is the goal of the WHO to prevent the transmission of the plague by avoiding direct contact with inflicted individuals." b. "Rabies is easily spread by contact with animals." c. "The United States is a certified polio free country." d. "The onset of tularemia is characterized by a distinct skin lesion often called a bull's-eye lesion."

ANS: C The Americas are certified polio free. The plague is a vector-borne disease and cannot be spread by direct contact with inflicted individuals. Rabies is a rare event because of the widespread vaccination of dogs in the 1950s. The Americas were certified as polio free in 1994. The onset of Lyme's disease is characterized by a bull's-eye lesion.

Which environmental law sets basic structure for regulating pollutants to United States waters? a. Safe Drinking Water Act b. Toxic Substance Control Act c. Clean Water Act d. Pollution Prevention Act

ANS: C The Clean Water Act sets basic structure for regulating pollutants to United States waters. The Safe Drinking Water Act authorized the EPA to establish safe standards of purity and required all owners or operators of public water systems to comply with primary standards. The Toxic Substance Control Act gives the EPA the ability to track the 75,000 industrial chemicals currently produced or imported into the United States. The Pollution Prevention Act focused industry, government, and public attention on reduction of the amount of pollution through cost-effective changes in production, operation, and use of raw materials.

A nurse is discussing eligibility for federal food assistance with a family. Which federal guideline would a nurse refer to when considering financial eligibility? a. Poverty Threshold Guideline b. Consumer Price Index (CPI) c. Federal Income Poverty Guideline d. Temporary Assistance to Needy Families (TANF)

ANS: C The Federal Income Poverty Guideline is issued by the U.S. Department of Health and Human Services and is used to determine if a person or family is financially eligible for assistance or services from various federal programs. The Poverty Threshold Guidelines are used primarily for statistical purposes. The CPI is a measure of the average change over time in the prices paid by households for a fixed market basket of consumer goods and services. TANF is a program, not a guideline, that assists families in need.

Employees working with hazardous chemicals have the "Right to Know" about the chemicals they are working with through the creation of the: a. Material Safety Data Sheet (MSDS). b. Consumer Confidence Report. c. Hazard Communication Standard. d. EPA.

ANS: C The Hazard Communication Standard requires employers to maintain a list of all hazardous chemicals that are used on site. Each of the chemicals should have an associated chemical information sheet known as an MSDS, which is written by the chemical manufacturer. Consumer Confidence Reports summarize the results of the annual testing of the public water supply. The EPA sets forth policies, regulations, and public laws for the safety of the environment and the population.

What is the purpose of the National Incident Management System (NIMS)? a. Create a new branch of government that deals with bioterrorism b. Establish a way for the Red Cross to carry out its mission c. Develop a nationwide all-hazards approach to domestic incident management d. Extend presidential power to act quickly upon acts of bioterrorism

ANS: C The National Response Framework is a unified, all-discipline, all-hazards approach to domestic incident management. It is built upon scalable, flexible, and adaptable coordinating structures to align key roles and responsibilities linking all levels of government and organizations. The Homeland Security Act of 2002 created a new branch of government that deals with bioterrorism, the U.S. Department of Homeland Security. The NIMS did not impact the Red Cross or extend presidential power to act upon bioterrorism.

The public health nurse (PHN) must participate in the essential services of public health. Which of the following most accurately describes one of the essential services of public health? a. Monitoring health status by completing a community assessment b. Diagnosing and investigating health problems in the world c. Informing, educating, and empowering people about health issues d. Working in law enforcement to regulate health and ensure safety

ANS: C The PHN monitors health status in several ways, completing a community assessment is only one way that health status is monitored. The PHN would not diagnose or solve "world" problems, or work in law enforcement. Rather, the PHN would participate with local regulators to protect communities and empower people to address health issues.

Which legislation changed the availability of federal services accessible to certain immigrants to the United States? a. Migrant Health Act of 1962 b. Supplemental Security Income (SSI) c. Welfare reform legislation of 1996 d. Food stamps

ANS: C The Welfare reform legislation of 1996 changed the availability of federal services accessible to certain immigrants in the United States. The Migrant Health Act provides funds for primary and supplemental health services to migrant workers and their families. Because of the welfare reform legislation of 1996, many legal immigrants and unauthorized immigrants are ineligible for services such as SSI and food stamps.

What is the purpose of writing a community nursing diagnosis? a. To assist with developing the evaluation measures for program planning b. To clearly describe the strengths and weaknesses of the community c. To lead to the outcomes and strategies to address and improve the identified health problem d. To increase the likelihood that the problem will be solved

ANS: C The community nursing diagnosis, no matter which classification system is used, leads to expected outcomes and evidence-based health promotion strategies to address and improve the problem identified in the diagnosis. This becomes the nursing care plan. The expected outcomes and evaluations derived from the nursing diagnosis systems suggest subsequent evaluation measures for identified needs or problems. A community assessment describes the strengths and weaknesses of a community. The likelihood of solving a problem is not influenced by writing a community nursing diagnosis.

A nurse performs activities to meet the primary goals of public health. Which of the following is the nurse most likely to complete? a. Ensuring that a newly diagnosed 40-year-old hypertensive man takes his medication b. Finding home care for a 70-year-old client recuperating from a hip replacement c. Conducting an infant car seat safety check d. Contacting a local hospice to admit a terminally ill 60-year-old woman

ANS: C The correct answer is concerned with the health of many people, while the other answers address individual interventions. According to the Core Functions Project, all levels of health care, including population-based public health care, must be funded or the goal of health of populations may never be reached.

A nurse is trying to facilitate interprofessional collaboration. Which of the following actions should be taken by the nurse? a. Understand who is in charge of the client's care b. Make appropriate referrals c. Recognize what other professionals do and how they view their roles d. Request assistance when performing complex skills

ANS: C The factors for successful interprofessional functioning include the categories of knowledge, skill, and attitudes with subheadings including the understanding of the roles of each professional.

A nurse is completing the first phase of a risk assessment. Which of the following questions would the nurse most likely try to answer? a. Has the chemical been released into the environment? b. How much and by which route of entry can the chemical enter the body? c. Is the chemical known to be associated with a negative health effect? d. What is the prediction for potential harm?

ANS: C The first phase is determining if a chemical is known to be associated with negative health effects (in animals or humans): Is the chemical known to be associated with a negative health effect? The second step is determining if the chemical has been released into the environment: Has the chemical been released into the environment? The third step is estimating how much and by which route of entry the chemical might enter the human body: How much and by which route of entry can the chemical enter the body? The final step takes into account the previous steps: What is the prediction for potential harm?

A community health nurse is using the Omaha System. Which of the following best describes the rationale for using this system? a. To define each health care provider's role b. To use nursing diagnoses in community health nursing practice c. To foster collaborative practice d. To provide practitioners a method for communication

ANS: C The goals of developing the Omaha System were to develop a structured and comprehensive system that could be both understood and used by members of various disciplines and to foster collaborative practice. Therefore, the Omaha System was designed to guide practice decisions, sort and document pertinent client data uniformly, and provide a framework for an agency-wide, multidisciplinary clinical information management system capable of meeting the daily needs of clinicians, managers, and administrators.

The greatest risk factor for suicide in adult women is: a. divorce. b. unemployment. c. intimate partner violence. d. terminal illness.

ANS: C The greatest risk factor for actual and attempted suicide in adult women is intimate partner violence.

A case manager supports a client's decision to return home after having a total hip replacement rather than go to a skilled nursing facility. Which of the following phases of the nursing process is being used? a. Assessment b. Planning/outcome c. Implementation d. Evaluation

ANS: C The implementation phase is similar to the advocacy process when the nurse aids the client in decision making and supports the client's decisions. The assessment/diagnosis phase is similar to the advocacy process of information exchange, gathering data, and illuminating values. The planning/outcome phase is similar to the advocacy process when generating alternatives and consequences and prioritizing actions. The evaluation phase is most similar to the advocacy processes of affirmation, evaluation, and reformulation.

A nurse is working with an individual who pursues neither his concerns nor another's concerns. Which of the following conflict management behaviors is being used? a. Accommodating b. Collaborating c. Avoiding d. Compromising

ANS: C The individual would be "avoiding." Avoiding is defined as when an individual pursues neither his or her concerns nor another's concerns. Accommodating occurs when an individual neglects personal concerns to satisfy the concerns of another. Collaborating is when an individual attempts to work with others toward solutions that satisfy the goals of both parties. Compromising occurs when an individual attempts to find a mutually acceptable solution that partially satisfies both parties.

Public health professionals refer to three levels of prevention as tied to specific stages in the: a. epidemiologic triangle. b. web of causation. c. natural history of disease. d. surveillance process.

ANS: C The natural history of disease is the course of the disease process from onset to resolution. The three levels of prevention provide a framework commonly used in public health practice to depict this process. The epidemiologic triangle consists of the interaction between an agent, a host, and the environment. The web of causality reflects the more complex interrelationship among the numerous factors interacting, sometimes in subtle ways, to increase (or decrease) risk of disease. The surveillance process involves the systematic collection, analysis, and interpretation of data related to the occurrence of disease and the health status of a given population.

The oldest and most widely used psychoactive drug in the world is: a. marijuana. b. heroin. c. alcohol. d. cocaine.

ANS: C The oldest and most widely used psychoactive drug in the world is alcohol. Marijuana is the most widely used illicit drug in the United States.

What is the primary cause of vulnerability? a. Race b. Age c. Poverty d. Illness

ANS: C The primary cause of vulnerability is poverty. Race, age, and illness may cause populations to be vulnerable, but these factors are not the primary cause.

A nurse is caring for a client with TB. Which of the following statements would the nurse recognize as being true? a. Meningitis is the leading cause of death from TB. b. Chemotherapy, if made widely available, could eradicate the disease. c. Many TB cases are associated with HIV. d. The TB bacillus is difficult to isolate.

ANS: C The rise in HIV has led to an increase in TB because the disease depresses the immune system. Chemotherapy is not always effective, especially in the new drug-resistant strains. Meningitis is not a leading cause of death from TB. TB is becoming more difficult to treat because of the new drug-resistant strains, but the TB bacillus is not difficult to isolate.

There are two medically indigent clients in the clinic who have come to get their monthly supply of free insulin. There is only enough for one client. Which of the following actions would the nurse take first? a. Identify all options b. Make a decision c. Gather additional information d. Act and assess decisions made

ANS: C The steps of the ethical decision-making framework are to first identify the ethical issues and dilemmas, then place them within a meaningful context, obtain all relevant facts, reformulate ethical issues and dilemmas if needed, consider appropriate approaches to actions or options, make decisions and take action, and evaluate decisions and action.

A nurse has used the steps of the ethical decision-making process when making a decision. Which of the following is most similar to this process? a. Healthy People 2010 b. Deontology c. The nursing process d. Advocacy

ANS: C The steps of the ethical decision-making framework are to first identify the ethical issues and dilemmas, then place them within a meaningful context, obtain all relevant facts, reformulate ethical issues and dilemmas if needed, consider appropriate approaches to actions or options, make decisions and take action, and evaluate decisions and action. The nursing process involves the same basic steps: assessment, diagnosis, planning, implementation, and evaluation. Healthy People 2010, deontology, and advocacy do not involve these multiple steps described by the ethical decision-making framework and the nursing process.

A nurse is providing home care for several elderly clients in the community. Which of the following situations would most likely be indicative of elder abuse? a. A daughter refuses to visit her mother due to work commitments. b. A child runs around a grandparent's house breaking items. c. A young man repeatedly steals money from his grandmother. d. An older person demands that the family come for dinner.

ANS: C Theft or mismanagement of money or resources is an element of abuse. Recognition of abuse includes these other possibilities as well: willful infliction of physical pain or injury, inflection of debilitating mental anguish and fear, or unreasonable confinement or the depriving of services. The other examples do not describe these elements of abuse.

A PHN provides a clinic for HIV-positive citizens in the community. Which of the following best describe this activity? a. Primary prevention b. Health education c. Tertiary prevention d. Policy making

ANS: C This clinic is for clients who already have a disease process; therefore, it is tertiary prevention (preventing deterioration in a patient, a relapse, or disability and dependency by anticipatory nursing and medical care). Primary prevention is using general and specific measures in a population to promote health and prevent the development of disease (incidence) and using specific measures to prevent disease in those who are predisposed to developing a particular condition. Health education would involve providing specific education to this population. Policy making is formulating plans by an organization in order to make decisions.

A nurse is caring for a homeless population. Which of the following characteristics should the nurse anticipate as a need of this population? a. Need more nursing care than other vulnerable groups b. Have no desire to seek medical care c. Have even fewer resources than poor people who have adequate housing d. Are living in despair with no hope or resilience

ANS: C Those who are homeless have even fewer resources than poor people who have adequate housing. Homeless and marginally housed people must struggle with heavy demands as they try to manage daily life because their resources are limited. These individuals must cope with finding a place to sleep at night and a place to stay during the day or moving frequently from one residence to another, as well as finding food, before even thinking about health care.

Which statement about eating disorders is correct? a. Individuals with anorexia frequently complain about weight loss. b. Purging is associated with anorexia. c. Most women with bulimia are concerned with the shape and weight of their body. d. Bulimia has more medical complications than anorexia.

ANS: C Those with bulimia are usually concerned with the shape and weight of their body. Those with anorexia view themselves as normal or overweight; purging is associated with bulimia; and anorexia is considered to have more complications than bulimia.

Why must nurses examine their attitudes about alcohol, tobacco, and other drug (ATOD) abuse and addiction before working with individuals with this health problem? a. Working with clients who have addiction problems often puts the nurse at risk for violence. b. The population of persons with drug and alcohol addiction is found in low-income neighborhoods. c. To be therapeutic, a nurse must develop a trusting nonjudgmental relationship with clients. d. Nurses are the primary persons who treat addiction problems.

ANS: C To be therapeutic, the nurse must develop a trusting, nonjudgmental relationship with clients. Therefore, nurses must examine their own attitudes ahead of time. Working with clients who have addiction problems does not put the nurse at risk for violence. People with drug and alcohol addiction can be found in any neighborhood. Nurses do not treat addiction problems.

A PHN would like to increase the immunization coverage of infants and toddlers. Which of the following strategies would be appropriate for the nurse to use? a. Read the Mortality and Morbidity Weekly Report to learn about vaccinations b. Require that children have all their immunizations before going to public school c. Track children known to be at risk for underimmunization d. Lead teams of health care workers to enforce laws related to immunizations

ANS: C Tracking children known to be at risk for underimmunization is a function of PHNs who work in health departments where immunizations are given and tracked. Reading the Mortality and Morbidity Weekly Report, requiring that children have all of their immunizations before attending school, and leading teams of health care workers would not be as effective in increasing immunization coverage for this population.

A community health nurse investigates an outbreak of pinworm at a local day care center. In order to minimize the spread of infection, which of the following suggestions would the nurse provide to the day care workers? a. Close the day care until all surfaces are cleaned. b. No action is necessary because it is easily treated with oral vermicides. c. Using good hand washing is important to prevent the transmission. d. Every child in the day care should be treated because they all are probably infected.

ANS: C Transmission of pinworm occurs through the fecal-oral route, so good hand washing after toileting is essential. It is not necessary to treat all children or close the day care. It is necessary that action be taken, as without any action being taken, the pinworm outbreak will continue.

A nurse partners with multiple agencies in the community to create a support group for those diagnosed with addiction in the community. Which of the following levels of prevention is being implemented? a. Primary b. Secondary c. Tertiary d. Health promotion

ANS: C When using tertiary prevention, the nurse focuses on treatment and rehabilitation. The support group for those with addiction is helping the addict and the addict's family, who already "have a problem." Primary prevention focuses on health promotion and disease prevention. Secondary prevention focuses on early detection and screening. Health promotion is not a level of prevention.

A bachelor's prepared nurse is providing case management services. Which of the following activities would most likely be provided by this nurse? a. Working with community aggregates b. Working with systems of disease c. Working with individuals d. Working with outcomes management processes

ANS: C Working with individuals would be the role of this nurse. Working with community aggregates, systems of disease, and outcomes management processes are implemented by an advanced practice nurse (APN).

A nurse is preparing an education program on disease transmission for employees at a local day care facility. When discussing the epidemiological triangle, the nurse should include which of the following factors as agents? (select all that apply) A. resource availability B. ethnicity C. toxins D. bacteria E. altered immunity

ANS: C, D

A community health nurse is developing an education program on substance use disorders for a group of adolescents. Which of the following information should the nurse include when discussing nicotine and smoking? A. smoking is the fifth-most preventable causes of death in the United States B. nicotine is a central nervous system stimulant C. withdrawal effects from smoking are minimal D. tolerance to nicotine develops quickly

ANS: D

A nurse is talking to a client who asks for additional information about hospice. which of the following statements should the nurse make? A. "clients who require skilled nursing care at home qualify for hospice care" B. "one function of hospice is to provide teaching to clients about life-sustaining measures" C. "hospice assists clients to develop skills needed to care for themselves independently" D. "a component of hospice care is to control the clients manifestations"

ANS: D

A nurse refers a client to alcoholics anonymous (AA). Who will play the most important role in the treatment of alcoholism at this support group? a. Self b. Spouses c. Children d. Peers

ANS: D AA began a strong movement of peer support to treat a chronic illness. The fellowship, support, and encouragement among AA members provide a vital social network for the person recovering from an addiction.

What is the purpose of adult day health centers? a. Provide support of life until death occurs b. Provide assistance to people who may need help with activities of daily living (ADLs) c. Provide treatment for those needing rehabilitation d. Provide respite care relief for caregivers

ANS: D Adult day health care is for individuals whose mental or physical function requires them to obtain more health care and supervision. It serves as more of a medical model than the senior center, and often individuals return home to their caregivers at night. Hospice provides support of life until death occurs. Assisted living provides assistance to people who may need help with ADLs. Long-term care facilities provide treatment for those needing rehabilitation.

A collection of individuals who have one or more personal or environmental characteristics in common is the definition of a(n): a. community. b. group. c. family. d. aggregate.

ANS: D An aggregate is a collection of individuals who have one or more personal or environmental characteristics in common. A community is a group of people that share something in common, such as geographic location, interests, or values. A group is people who are located close together. A family is considered parents and children living together in a household.

An elderly person is living independently in a single room in a full-service life care community. Which of the following terms best describes this living arrangement? a. Adult daycare b. Home care c. Nursing home d. Assisted living

ANS: D An elderly person living independently in a single room in a full-service life care community is a client of an assisted living facility. Adult day health care is for individuals whose mental or physical function requires them to obtain more health care and supervision; it serves as more of a medical model than the senior center, and often individuals return home to their caregivers at night. Nursing home facilities provide treatment for those needing rehabilitation. Home care provides individual and environmental assessments, direct skilled care and treatment, and short-term guidance for individuals in the home.

A city sets the standard of an acceptable level of emissions or a maximum contaminant level allowed for factories. Which of the following concepts is being applied? a. Controlling pollution b. Waste minimization c. Land use planning d. Environmental standard

ANS: D An example of an environmental standard is an acceptable level of emissions or a maximum contaminant level allowed. Although this does help to control pollution, environmental standard is best applied in this example. Setting a standard for emissions does not impact waste minimization or land use planning.

The nurse is providing community health education about how alcohol affects the body. Which of the following individuals would the nurse describe as being most likely to have the lowest blood alcohol concentration? a. An individual who consumes a drink with a high concentration of alcohol b. An individual who drinks without consuming food c. An individual of the female gender d. An individual with a high body weight

ANS: D An individual with a high body weight would likely have the lowest blood alcohol concentration. Increased alcohol concentration, drinking without consuming food, and being a female are all factors that increase blood alcohol concentration.

What occurs during biological terrorism? a. The intentional release of hazardous chemicals into the environment b. Environmental and occupational exposure to biological toxins c. Immunity to toxins related to repeated exposures d. An intentional release of viruses, bacteria, or other toxins

ANS: D An intentional release of viruses, bacteria, or other toxins would be described as biological terrorism. Chemical terrorism is the intentional release of hazardous chemicals into the environment for the purpose of harming or killing. Surveillance is used to monitor environmental and occupational exposures. Immunity to toxins after repeated exposures does not occur during biological terrorism.

A public health nurse (PHN) reports an attack rate. Which of the following has most likely been reported? a. Number of cases of cancer recorded at a medical center b. Number of people who died of Ebola in a given year c. Number of beef cattle inoculated against mad-cow disease on a farm d. Proportion of people becoming ill after eating at a fast-food restaurant

ANS: D Attack rates are often specific to exposures, such as food-specific attack rates. The number of cases of cancer, exposure to Ebola, and beef cattle are not significant without knowing the total number of people so that a proportion can be calculated.

Which statement about migrant and seasonal farm workers is true? a. The economic status of migrant workers has improved over the last decade. b. This is a transient population that does not need nursing care. c. Most migrant workers are eager to accept health care services. d. Availability of food in the United States depends on these individuals.

ANS: D Availability and affordability of food in the United States depends on these individuals, yet their economic and social status has not changed significantly over the past decades. Most migrant workers do not seek out health care services.

A nurse is working to remove barriers to receiving health care. Which of the following actions is the nurse most likely to support? a. Discrimination against certain groups b. Treatment of pets at the same facility c. Provision of free food at a food bank d. Providing services for a rural population by using a mobile clinic

ANS: D Barriers to access are policies and financial, geographic, or cultural features of health care that make services difficult to obtain or so unappealing that people do not want to seek care. Examples of removing barriers include providing extended clinic hours, low-cost or free health services for people who are uninsured or underinsured, transportation, mobile vans, and professional interpreters, which can help improve access to care.

A nurse is investigating a serious epidemic of influenza. Which of the following best describes the amount of cases that are being examined? a. 50 cases b. 100 cases c. 500 cases d. Unable to determine

ANS: D One cannot tell the degree of seriousness without a denominator, which represents the total population

A nurse is applying the knowledge and processes of ethics to the examination of ethical problems in health care. Which of the following describes the actions of the nurse? a. Values b. Morality c. Ethics d. Bioethics

ANS: D Bioethics applies the knowledge and processes of ethics to the examination of ethical problems in health care. Values are beliefs about the worth or importance of what is right or esteemed. Morality is shared and generational societal norms about what constitutes right or wrong conduct. Ethics is a branch of philosophy that includes both a body of knowledge about the moral life and a process of reflection for determining what persons ought to do or be regarding this life.

The growing multiculturalism of American society can contribute to ethnicity conflicts when: a. providing care to different cultural groups. b. individual values align with the cultural norms. c. ethnic groups overburden the health care system. d. the greater community's values are jeopardized by specific ethnic values.

ANS: D Callahan offered perspectives on judging diversity and suggests a thoughtful tolerance and some degree of moral persuasion (not coercion) for ethnic groups to alter values so that they are more in keeping with what is normative in American culture. Providing care to different cultural groups should not produce an ethnicity conflict. Individual alignment with cultural norms would make it less likely that an ethnicity conflict would occur. Ethnic groups using the health care system will not cause it to be overburdened or result in an ethnicity conflict.

Which of the following disorders has been identified as an occupational health risk for agricultural workers? a. Stroke b. Heart disease c. Diabetes d. Cancer

ANS: D Cancer is an identified but not well-documented health problem for migrant farm workers associated with their exposure to chemicals. Other identified health risks are musculoskeletal injuries, traumatic injuries, respiratory problems, dermatitis, infectious diseases, and eye problems

A nurse is working as a case manager. Which of the following best describes the diagnoses that the case manager is most likely to encounter? a. Bankruptcy, financial distress, and depression b. Flu, colds, and frequent headaches c. Malaria, bird flu, and Dengue fever d. AIDS, spinal cord injury, and ventilator dependency

ANS: D Case-managed conditions include many chronic conditions. AIDS, spinal cord injury, and ventilator dependency are the only chronic conditions listed. The other options are acute or nonmedical conditions.

A nurse is working with a religious organization to provide food to starving people in a lesser-developed country. Which of the following organizations is the nurse most likely working with? a. International Red Cross b. Church World Service c. Maryknoll Missionaries d. Catholic Relief Services

ANS: D Catholic Relief Services alleviates suffering and provides assistance to people in need affected by war, starvation, famine, drought, and national disasters without regard to race, religion, or nationality. The Red Cross is not a religious organization, and the Church World Service and Maryknoll Missionaries do other things besides providing food.

Which statement about rape is true? a. Rural areas are high-risk areas for rape to occur. b. Most rapes occur in the winter. c. Male victims suffer more emotional trauma than females. d. College women are at high risk for experiencing rape.

ANS: D College women are at particularly high risk for sexual victimization in the United States, with research estimating that approximately 25% of female college students will experience an attempted or completed rape at some point during their college experience. It appears that the emotional trauma for a male rape victim is at least as serious as that for a woman.

What factor associated with crime and violence is particularly important to community health nursing practice? a. Violence is a universal problem. b. Violence impacts everyone in the United States. c. Nurses must spend a great deal of time on the results of violence. d. Significant mortality and morbidity result from violence.

ANS: D Communities across the United States are concerned about crime and violence rates because of significant mortality and morbidity that often result. These issues are closely related to community health nursing. While it is true that violence is universal, and so to some degree affects all citizens, resulting in the need for nursing care, these issues relate to nursing in general and especially those involved in direct care of the patient's physical and emotional needs.

A nurse is trying to develop community partnerships. Which of the following interventions would be the most appropriate for the nurse to use? a. Involve the community residents b. Use nurses as the source of information and leadership c. Rely on the power of local officials d. Include a variety of disciplines

ANS: D Community partnerships occur when community residents and health workers come from a variety of disciplines. Partnerships should involve a variety of individuals from various backgrounds. There should be a balance of power and information sharing among all of the participants.

Community preparedness is being used to plan for a disaster. Which of the following tasks is being implemented? a. Assembling emergency supplies b. Understanding the workplace disaster plan c. Taking a disaster training course d. Developing an evacuation plan to remove individuals from danger

ANS: D Community preparedness involves developing an evacuation plan to remove individuals from danger. Assembling emergency supplies is an example of individual preparedness. Understanding the workplace disaster plan is an example of preparedness in the workplace within the community. Taking a disaster training course is an example of professional preparedness.

A nurse is conducting vision screenings on children in the school setting. Which type of nursing practice is the nurse performing? a. Community-oriented b. Public health c. Community health d. Community-based

ANS: D Community-based nursing practice is a setting-specific practice whereby care is provided for clients and families where they live, work, and attend school. Community-oriented nursing emphasizes the prevention of disease and disability. Public health nursing focuses on the care within the community as a whole. Community health nursing focuses on the health status of individuals and the effect of their health status on the community as a whole.

A nurse operates a school-based clinic in a local school where multiple providers and disciplines offer care to children, making it easier for children to access health care. Which of the following best describes this approach? a. Advocacy b. Wrap-around services c. Social justice d. Comprehensive services

ANS: D Comprehensive services are health care services that focus on more than one health problem or concern. Stationary or mobile clinics that provide a wide array of health promotion, illness prevention, and illness management services in migrant camps, schools, and local communities are examples of this. Wrap-around services describe a system in which comprehensive health services are available and social and economic services are "wrapped around" these services. Advocacy refers to actions taken on behalf of another. Social justice describes justice with respect to the concepts of egalitarianism and equality.

A nurse has experienced a cultural encounter. Which of the following best describes what has happened? a. Sharing significant assessment findings with members of a racial minority b. Visiting the native land of the clients served at a community health center c. Telephoning the priest at a Hispanic church to discuss the health issues of a client d. Learning about traditional healing practices from an American Indian client

ANS: D Cultural encounter refers to the process that permits nurses to seek opportunities to directly engage in cross-cultural interactions with clients of diverse cultures to modify existing beliefs about a specific cultural group and possibly avoid stereotyping. Learning about traditional healing practices is an example of a direct cultural encounter. This occurs when a nurse engages in cross-cultural interactions. Sharing significant assessment findings demonstrates the nurse sharing information, not an engagement with the population. When visiting a native land, there may not be interaction with any other culture. Telephoning a priest at a Hispanic church does not demonstrate an interaction, but rather the nurse calling the priest to accomplish a nursing task.

A nurse is assisting clients to improve their health status. Which of the following types of management is being used by the nurse? a. Care management b. Case management c. Disease management d. Demand management

ANS: D Demand management seeks to control use by providing clients with correct information and education strategies to make healthy choices, to use healthy and health-seeking behaviors to improve their health status, and to make fewer demands on the health care system. Care management is an enduring process in which a manager establishes systems and monitors the health status, resources, and outcomes for a targeted aggregate of the population. Case management is defined as a collaborative process of assessment, planning, facilitation, care coordination, evaluation, and advocacy for options and services to facilitate an individual's and family's comprehensive health needs through communication and available resources to promote quality cost-effective outcomes. Disease management constitutes systematic activities to coordinate health care interventions and communications for populations with disease conditions in which client self-care efforts are significant.

A nurse is examining the income levels and social networks of individuals in a community. Which of the following is the nurse investigating? a. Culture b. Development c. Community d. Determinants

ANS: D Determinants are conditions and factors that are important considerations in population health. Culture refers to the beliefs and customs of a certain group. Development refers to something that is being created. Community is a group of people living in the same geographic location.

The factors, exposures, characteristics, and behaviors that determine patterns of disease are described using: a. Descriptive epidemiology b. Analytic epidemiology c. Distribution d. Determinants

ANS: D Determinants are the factors, exposures, characteristics, and behaviors that determine patterns of disease, which may be individual, relational, social, communal, or environmental. Descriptive epidemiology seeks to describe the occurrence of a disease in terms of person, place, and time. Analytic epidemiology focuses on the investigation of causes and associations. Distribution describes who has the disease and where and when the disease occurs.

A nurse is participating in the preparedness stage of disaster management. Which of the following describes what is happening? a. Heightened inspection and increased security in the community b. Incorporation of provision of pets into local disaster plans c. Purchase of personal protective equipment for all citizens d. Assembly of disaster kits for the home, workplace, and car

ANS: D Disaster kit assembly for the home, workplace, and car, especially by nurses, occurs during the preparedness phase. Heightened inspection and increased security in the community is part of the prevention activities. Purchase of personal protective equipment for all citizens and incorporation of provision of pets into local disaster plans are not realistic plans for preparedness.

A nurse is assessing the drug use patterns of a client. Which of the following questions would be most appropriate for the nurse to ask? a. Where was the drug acquired? b. Who gave the drug to you? c. Why do you use the drug? d. How often do you use the drug?

ANS: D During an assessment, all relevant drug-use history is collected and aids in the assessment of drug-use patterns. The nurse should note any changes in drug-use patterns over time. Thus, the most appropriate question is the nurse asking how often a client uses drugs.

What information is included in a case definition? a. The precise point of contact b. Laboratory confirmation c. Source of contamination d. Clinical symptoms

ANS: D Each case has a unique set of criteria based on what is known about the particular disease. It may include clinical symptoms, laboratory values, and epidemiologic criteria. The precise point of contact, laboratory confirmation, and source of contamination are not part of a case definition.

Asians may perceive illness as disharmony with other forces and that medicine is only able to relieve the symptoms rather than cure the disease. They may look to naturalistic solutions and acupuncture to resolve or cure health problems. Which of the following types of cultural variations is being demonstrated? a. Communication b. Personal space c. Social organization d. Environmental control

ANS: D Environmental control refers to the ability of individuals to control nature and to influence factors in the environment that affect them. Communication is the means by which culture is shared (verbal and nonverbal). Personal space is the physical distance between two individuals during an interaction. Social organization refers to the way in which a cultural group structures itself around the family to carry out role functions.

A facility is inspected after it has obtained a permit. The inspection is completed for the purpose of observing whether the plans submitted in the permit application are being implemented as approved. Which of the following environmental protection strategies is being applied? a. Controlling pollution b. Waste minimization c. Land use planning d. Environmental monitoring

ANS: D Environmental monitoring would be an inspection of a facility after a permit is obtained to observe whether the plans submitted in the permit application are being implemented as approved. Permitting is an important step in controlling pollution. Waste minimization and land use planning are activities aimed at prevention of pollution.

To understand the causes of health and disease, epidemiology studies: A. individuals B. families C. groups D. populations

ANS: D Epidemiology monitors health of populations, understands determinants of health and disease in communities, and investigates and evaluates interventions to prevent disease and maintain health. Epidemiology does not focus on individuals, families, and groups.

A nurse is implementing an intervention at both the primary and tertiary levels of prevention. Which of the following best describes the nursing intervention? a. Providing emergency shelter housing b. Offering physical and mental health services c. Developing a targeted case management program d. Establishing a needle exchange program

ANS: D Establishing a needle exchange program could be considered both primary (preventing a problem before it occurs by helping clients avoid disease transmission) and tertiary (working with a population who already has a "problem" of drug abuse). Providing emergency shelter housing and offering physical and mental health services are both secondary and tertiary prevention interventions. Developing a targeted case management program is secondary prevention.

Which statement about feminist ethics is correct? a. Feminists include only women in their worldview. b. Persons who ascribe to feminist ethics are passive and wish to pursue their ideals through the legislative process. c. Feminists believe that men should not be nurses. d. Women's thinking and moral experiences are important and should be taken into account.

ANS: D Feminist theory ascribes to the idea that women's thinking and moral experiences are important and should be considered. A feminist perspective leads us to think critically about connections among gender, disadvantage, and health as well as the distribution of power in public health processes. Feminists are women and men who hold a worldview advocating economic, social, and political equality for women that is equivalent to that of men.

Homicides are least likely to be perpetrated by a(n): a. friend. b. acquaintance. c. family member. d. stranger.

ANS: D Homicides are least likely to be committed by a stranger. When strangers are involved, many of these are related to the illegal substance abuse network. Homicides are usually perpetrated by friends, acquaintances, or family members during an argument.

An older person is in the last stages of dying. Which type of care would be the best for him? a. Home health b. Assisted living c. Nursing home d. Hospice

ANS: D Hospice would be the best form of care for a person in the last stages of dying. Home health care provides individual and environmental assessments, direct skilled care and treatment, and short-term guidance for individuals in the home. Assisted living offers a wide variety of living choices, from a single shared room to a full-service, life-care community. Nursing homes provide care to meet the health care needs of those needing rehabilitation, as well as for those needing a permanent supportive residence.

A nurse is providing comprehensive education about HIV transmission for an injection drug user (IDU). Which of the following would be the most important information to include? a. How and where to acquire needles b. Proper needle usage and possible injection sites c. Spreading of the virus via homosexual activities d. Using bleach between needle uses

ANS: D IDUs represent the most rapidly growing source of new cases of AIDS, and they are the greatest risk for spread of the virus in the heterosexual community. Emphasis is being placed on reduction of the transmission of this disease through contaminated needles. Nurses should provide education on cleaning needles with bleach between uses to decrease the spread of the virus. Education on needle exchange programs is also appropriate; however, this population is not as likely to use these programs even when they are available.

A nurse is planning to provide culturally effective care to Mexican migrant farm workers. Which of the following strategies should be used by the nurse? a. Learn how to speak Spanish to improve communication b. Understand that traditional beliefs and practices are more common among the well-educated population c. Consult the female head of household about health care matters d. Recognize that cultural backgrounds of the workers will vary depending on their place of origin

ANS: D It would be incorrect to assume that all Mexicans have the same cultural beliefs. The nurse must remember that beliefs and practices differ between regions and localities of a country, and among individuals. Mexico is a multicultural country; therefore, the cultural backgrounds of Mexican immigrants vary depending on their place of origin.

Care in a managed system is provided by less experienced providers. Which of the following ethical principles is being influenced in this situation? a. Autonomy b. Beneficence c. Nonmaleficence d. Justice

ANS: D Justice calls for equal distribution of health care with reasonable quality. Autonomy is the individual's right to choose a provider. Beneficence is influenced when excessive attention to cost containment supersedes or impairs the nurse's duty to provide measures to improve health or relieve suffering. Nonmaleficence is doing no harm, which is addressed when incorporating outcomes measures, evidence-based practice, and monitoring processes in plans of care.

A community health nurse is caring for a client with methicillin-resistant Staphylococcus aureus (MRSA). Which of the following should the nurse be aware of? a. Persons with MRSA usually have a chronic illness. b. MRSA is a hospital-acquired infection and not often seen in the community. c. Vancomycin-resistant Staphylococcus aureus (VRE) follows MRSA. d. MRSA is becoming more common in the community.

ANS: D MRSA is being seen more and more in the community with outbreaks frequently associated with school athletic programs and prison populations. MRSA is still largely a health care-associated infection, but it is becoming more common. VRE was found before MRSA. MRSA is not associated with chronic illness.

The most widely used illicit drug in the United States is: a. alcohol. b. heroin. c. cocaine. d. marijuana.

ANS: D Marijuana is the most widely used illicit drug in the United States. The oldest and most widely used psychoactive drug in the world is alcohol.

A nurse provides care to rural adults after working in an urban clinic for many years. Which of the following considerations should the nurse make when working with this population? a. Rural adults are more likely to have health insurance. b. Rural adults are more likely to have access to health care providers. c. Rural adults are more concerned about their health status. d. Rural adults are more likely to experience mobility limitations.

ANS: D More rural adults experience at least three of the limitations of walking one block, walking uphill or climbing stairs, bending, lifting, stooping, feeding, dressing, bathing, and toileting. In general, rural residents are less concerned about their health status, less likely to have access to health care providers, and less likely to have health insurance.

How have nurses historically learned to identify a possible relationship between environmental chemical exposures and their potential harm? a. Extrapolation by toxicologists b. Biomonitoring c. Completing chemistry courses d. Observing signs and symptoms in clients

ANS: D Nurses have historically made discoveries related to chemical exposure when people presented with signs and symptoms related to known chemical toxicity. Extrapolation by toxicologists and biomonitoring are modern methods. Completing a chemistry course does not show how to identify these possible relationships.

Official agencies are financed primarily by: a. charities. b. individual clients. c. third-party payers. d. tax funds.

ANS: D Official agencies are financed primarily by tax funds. These agencies are typically operated by state, county, city, or other local government units, such as health departments.

A community coalition monitors the increasing obesity rate of children in their schools. Based on this data, they consider a variety of programming options which may possibly help decrease this trend. What was the purpose of conducting this surveillance? a. Protect the children from diseases that affect obese children b. Teach parents that obesity will not be tolerated in this community c. Educate children on surveillance techniques d. Demonstrate that new clinical and effective protocols need to be developed

ANS: D Ongoing surveillance in a community can lead to new clinical and effective protocols to address an issue. Ongoing surveillance makes it possible to have ongoing monitoring in place to ensure that disease and event patterns improve rather than deteriorate. This surveillance cannot protect children from diseases, teach parents that obesity will not be tolerated, or educate children on surveillance techniques.

A nurse is working with children who may be at risk for experiencing abuse. In which situation is child abuse most likely to occur? a. When the parents have high expectations for their children b. When the parents' sense of control is threatened c. When the parents are sensitive to their children's needs d. When the parents lack a social support network

ANS: D Parents with a lack of social support, history of abuse as a child, or who have minimum education, a tendency toward depression, or multiple stress factors may be at risk for abusing their children. Parents who have high expectations for their children or are sensitive to their children's needs do not have a tendency toward abuse. Parents who have their sense of control threatened do not have a tendency toward abuse.

In the Vietnamese culture, individuals may focus on wishes and memories of their ancestors and look to them to provide direction for current situations. Which of the following types of cultural variations is being demonstrated? a. Communication b. Personal space c. Social organization d. Perception of time

ANS: D Perception of time is the duration or period between successive events, where some cultures assign greater or lesser emphasis to events that occur in the past, present, or future. Communication is the means by which culture is shared (verbal and nonverbal). Personal space is the physical distance between two individuals during an interaction. Social organization refers to the way in which a cultural group structures itself around the family to carry out role functions.

A PHN develops and implements local public health policies through partnerships with agencies, organizations, and consumers within the community. Which of the following core public health functions is being used? a. Assessment b. Prevention c. Assurance d. Policy development

ANS: D Policy development deals with developing and implementing health policies. Prevention is not a core function, assurance is making sure essential services are available, and assessment refers to systematic data collection.

Which of the following factors has the largest impact on health disparities among all populations? a. Ethnicity b. Education level c. Lifestyle choices d. Poverty

ANS: D Poverty is a strong and underlying current factor that affects all special groups. Other factors that may also influence disparities are: education, insurance status, segregation, immigration status, health behaviors and lifestyle choices, health care provider behavior, employment, and the nature and operation of the health system in the community.

A nurse is responding to a disaster. Which of the following is the priority of the disaster response? a. Clean up the environment b. Handle the stress reaction of the victims c. Bring in as many aid workers and nurses as possible d. Reestablish sanitary barriers and focus on basic needs

ANS: D Reestablishing sanitary barriers and focusing on water, food, waste disposal, vector control, shelter, and safety are the first goals. After this has been established, the nurse can address the cleanup of the environment, the stress reactions of the victims, and the need to bring in additional workers.

Which statement is discussed in the Code of Ethics for Nurses with Interpretive Statements? a. The profession of nursing is responsible for making political statements and supporting nurse-friendly candidates for office. b. The nurse's primary focus is on acute bedside nursing, followed by community health care to promote seamless care. c. The nurse owes duty primarily to the physician to strive to protect health, safety, and the rights of the patient. d. The profession of nursing is responsible for articulating nursing values, for maintaining the integrity of the profession, and for shaping social policy.

ANS: D Provision 9 of the Code of Ethics for Nurses with Interpretive Statements discusses the need for the nursing profession to address national and global health concerns as well as be involved with shaping policies through political action. The Code of Ethics for Nurses with Interpretive Statements does not address the practice setting for nursing care. According to the Code of Ethics for Nurses with Interpretive Statements, public health should achieve community health in a way that respects the rights of individuals in the community, not owing duty primarily to the physician. Public health should seek the information needed to implement effective policies and programs that protect and promote health.

A nurse is implementing a primary prevention strategy. Which of the following activities would the nurse most likely implement? a. Urine screening for pesticide exposure b. Tuberculosis (TB) skin testing c. Treatment of lead poisoning d. Reduction of pesticide exposure

ANS: D Reduction of pesticide exposure is an example of primary prevention. Urine screening and TB skin testing are secondary prevention, and treatment of lead poisoning is tertiary prevention.

A nurse is caring for a client who has had long-term marijuana use. Which of the following physical effects should be of priority concern to the nurse? a. Constipation b. Cardiac dysrhythmia c. Abdominal pain d. Respiratory tract damage

ANS: D Respiratory tract damage occurs from smoking the drug and is the greatest physical concern among chronic users. Cardiac dysrhythmias may be a concern of chronic alcohol use. Constipation and abdominal pain are not of priority concern.

A PHN is organizing a multidisciplinary team to address the issue of water pollution in the community. The most likely members that would be invited to address this issue would be: a. physicians, water sanitation workers, and occupational therapists. b. pharmacologists, radiologists, and epidemiologists. c. nurse practitioners, pharmacologists, and environmentalists. d. geologists, meteorologists, and chemists.

ANS: D Scientists who study how pollutants travel in air, water, and soil are geologists, meteorologists, and chemists. The other professionals are not experts in the area of water pollution in the community.

A nurse implements a program that focuses on secondary prevention. Which of the following is most likely the topic of this program? a. Rehabilitation b. Avoidance of high-risk behaviors c. Immunization d. Mammogram

ANS: D Secondary prevention focuses on early detection and prompt treatment of disease, injury, or disability. Mammograms are a screening test. Avoidance of high-risk behaviors and immunizations are examples of primary prevention. Rehabilitation is an example of tertiary prevention.

A nurse is providing secondary prevention when working with the homeless. Which of the following best describes what is being implemented? a. Employer incentives b. Safe sex education c. Comprehensive case management d. Soup kitchens

ANS: D Secondary preventive services target persons on the verge of homelessness as well as those who are newly homeless. An example of secondary prevention is a soup kitchen. Employer incentives and safe sex education are primary prevention strategies; comprehensive case management is a tertiary prevention strategy.

A nurse is assessing the community for potential settings of drug experiences. Which of the following describes what the nurse would most likely identify? a. A back alley or abandoned building where people are using crack b. An area at the local high school where students are known to smoke cigarettes c. The individuals using the drug, including that individual's expectations d. The physical, social, and cultural environment within which the use occurs

ANS: D Setting is the influence of the physical, social, and cultural environmental within which the use occurs. All of these environments influence the use of drugs. A back alley, an area at the local high school, and individuals using the drug do not address all of the aspects of setting.

A client is suffering from stress related to the depressed lumber industry in his county. Which of the following best describes this client's condition? a. Manic-depression b. Psychosis c. Anticipatory grieving d. Farm stress

ANS: D Stress related to the depressed lumber industry can be classified as farm stress. Manic-depression is alternating abnormal highs and lows of feelings and behaviors. Psychosis is a mental illness in which connection with reality is lost. Anticipatory grief refers to grief before an impending loss.

A nurse experiences a stress reaction while working with the survivors of a disaster. Which of the following best describes what has happened? a. The nurse makes decisions to assist with care coordination of the survivors. b. The nurse tells the supervisor of the disaster the details of the care that was provided. c. The nurse cries after returning home at the end of the day. d. The nurse refuses to follow orders.

ANS: D Symptoms that may signal a need for stress management assistance include the following: being reluctant or refusing to leave the scene until the work is finished; denying needed rest and recovery time; feelings of overriding stress and fatigue; engaging in unnecessary risk-taking activities; difficulty communicating thoughts, remembering instructions, making decisions, or concentrating; engaging in unnecessary arguments; having a limited attention span; and refusing to follow orders.

a. AIDS b. Malaria c. Hepatitis d. TB

ANS: D TB is the leading killer of people with HIV, and up to 80% of TB clients are HIV positive in countries with a high prevalence of HIV. AIDS, malaria, and hepatitis are not the leading killer of people with HIV.

A nurse is planning and implementing care for vulnerable populations. Which of the following would be the most appropriate action for the nurse to take? a. Setting up multiple clinics in a wide geographic area b. Advising legal consultants on a variety of issues c. Making laws to protect the homeless d. Teaching vulnerable individuals strategies to prevent illness and promote health

ANS: D Teaching vulnerable individuals, families, and groups strategies to prevent illness and promote health is one of the ways nurses provide care for these populations. Setting up clinics, advising legal consultants, and making laws do not address the direct care provision role of the nurse.

A nurse is developing and coordinating an emergency response plan for a community. With which of the following organizations would the nurse collaborate? a. American Red Cross (ARC) b. National Disaster Medical System (NDMS) c. Centers for Disease Control and Prevention (CDC) d. Emergency Management Agency (EMA)

ANS: D The EMA is responsible for developing and coordinating emergency response plans in a defined area. The ARC works with communities in the preparedness, response, and recovery phases of a disaster. The CDC studies the effect that disasters have on population health and continuously develops new prevention strategies. The NDMS provides nurses the opportunity to work on specialized disaster response teams.

Which statement about poor people living in the seventeenth century is accurate? a. They were likely alcoholics or prostitutes. b. They were ashamed to be living in poverty. c. They traveled from community to community for food and shelter. d. They would be given assistance by their community.

ANS: D The Elizabethan Poor Laws began in seventeenth-century England by helping the poor receive help within the boundaries of their original community. Needy travelers would not be helped and were sent back to their original community where they would be helped by their own folk. It was no disgrace to live in poverty because nearly everyone was poor.

A woman needs to take some time off from work to care for her invalid mother. Which of the following health policies allows her to take an extended leave from work to care for a family member? a. Personal Responsibility and Work Opportunity Reconciliation Act b. Women's Health Equity Act (WHEA) c. Temporary Assistance for Needy Families (TANF) d. Family and Medical Leave Act (FMLA)

ANS: D The FMLA provides job protection and continuous health benefits where applicable for eligible employees who need extended leave for their own illness or to care for a family member.

A nurse is using the Omaha System Problem Classification Scheme. Which of the following situations is the nurse most likely experiencing? a. Measuring client outcomes b. Collaborating with other professionals c. Providing health education d. Organizing a comprehensive assessment

ANS: D The Omaha System Problem Classification Scheme is used to organize a comprehensive assessment according to four priority domains. It's simple and concrete terms are used to organize a comprehensive assessment, an important standard of nursing practice. It does not measure client outcomes, promote collaboration with other professionals, or provide health education.

After conducting a comprehensive client assessment, the nurse would organize information about the client's medication regimen into which domain of the Omaha System Problem Classification Scheme? a. Environmental b. Psychosocial c. Physiological d. Health-related behaviors

ANS: D The Omaha System Problem Classification Scheme uses four domains to organize client information. The Health-related behaviors domain includes information related to patterns of activity that maintain or promote wellness, promote recovery, and decrease the risk of disease. The environmental domain includes material resources and physical surroundings both inside and outside the living area. The psychosocial domain includes patterns of behavior, emotion, communication, relationships, and development. The physiological domain includes functions and processes that maintain life.

The ability of an agent to produce a severe pathologic reaction is known as: a. antigenicity. b. invasiveness. c. toxicity. d. virulence.

ANS: D The ability of an agent to produce a severe pathologic reaction is known as virulence. Antigenicity is the ability to stimulate an immunological response. Invasiveness is the ability to penetrate and spread throughout a tissue. Toxicity is the ability to produce a poisonous reaction.

How much alcohol can the liver metabolize in an hour? a. 1 oz of whiskey b. 3 oz of wine c. 5 oz of wine d. 12 oz of beer

ANS: D The amount of alcohol the liver can metabolize per hour is equal to about 3/4 oz of whiskey, 4 oz of wine, or 12 oz of beer.

The interaction between an agent, a host, and the environment is called: a. natural history of disease. b. risk. c. web of causality. d. the epidemiologic triangle.

ANS: D The epidemiologic triangle consists of the interaction between an agent, a host, and the environment. The natural history of disease is the course of the disease process from onset to resolution. Risk is the probability an event will occur within a specified period of time. The web of causality reflects the more complex interrelationship among the numerous factors interacting, sometimes in subtle ways, to increase (or decrease) risk of disease.

A nurse plans on implementing a community-wide influenza immunization program. Which of the following factors should the nurse consider when implementing this program? a. The number of community members who have already received the immunization b. The existence of formal groups in the community c. Public policy that mandates influenza immunization for certain populations d. The community's readiness to participate in the program

ANS: D The factors that influence implementation in the community are the nurse's chosen roles, the type of health problem selected as the focus for intervention, the community's readiness to take part in problem solving, and characteristics of the social change process. The community's readiness to participate is more important than public policy, the existence of formal groups in the community, and the number of community members who have already received the vaccine.

The first stage of collaboration is: a. trust building. b. consensus. c. collegiality. d. awareness.

ANS: D The first stage of collaboration is awareness, when one makes a conscious entry into a group process. This is followed by tentative exploration and mutual acknowledgement, trust building, collegiality, consensus, commitment, and collaboration

A community health nurse would like to reduce the prevalence of sexually transmitted diseases (STDs) in the community. Which of the following activities would most likely be used? a. Establish immunization clinics to prevent STDs b. Educate people with HIV about the mode of transmission c. Explain to women that HIV is transmitted to women usually by intravenous drug abuse d. Develop an STD clinic to increase community access to services

ANS: D The goal of Healthy People 2020 is to promote responsible sexual behaviors, strengthen community capacity, and increase access to quality services to prevent STDs and their complications. Nursing activities should align with these goals through development of an STD clinic to increase community access to services. Immunization clinics will not prevent the spread of most STDs. Education without access to services will not assist with reducing the problem.

A case manager has contacted providers and has negotiated services and prices. Which of the following phases of the nursing process is being demonstrated? a. Assessment b. Diagnosis c. Planning/outcome d. Implementation

ANS: D The implementation phase of the nursing process occurs when a case manager advocates for clients' interests and arranges for the delivery of service. Examples of assessment include developing networks with target populations and dissemination of written materials. Diagnosis includes the identification of a problem/opportunity. Examples of activities used during the diagnosis phase include holding conferences, determining conclusions on the basis of assessment, and using interprofessional teams. Examples of planning for outcomes include validating and prioritizing problems and selecting evidence-based interventions.

The key to achieving the goals of Health for All in the 21st Century (HEALTH21) is the worldwide: a. cure of communicable disease. b. technological development of all nations. c. availability of health care insurance. d. implementation of primary health care.

ANS: D The major key to attaining Health for All by the Year 2000 was the worldwide implementation of primary health care; this has extended into the next century with the document HEALTH21. The World Health Assembly in 1977 stated that all citizens of the world should enjoy a level of health that would permit them to lead a socially and economically productive life. Curing communicable disease, technological developments of all nations, and availability of health care insurance are not part of the key strategies to achieve the goals of HEALTH21.

A nurse is trying to increase participation in a free colorectal screening program for middle-aged adults who lack health insurance. Which of the following implementation mechanism would be most effective? a. Small interacting groups b. Health policy c. Lay advisors d. Mass

ANS: D The mass media (newspapers, television, and radio) represent an impersonal and formal type of communication and are useful in providing information quickly to a large number of people. The other methods will take much longer for the information to spread to the community members.

Which statement about poverty is true? a. All cultures view poverty as a pitiful existence. b. Most cultures want to improve the plight of the poor. c. Dangerous environments are present in all cultures. d. Western cultures view poverty negatively.

ANS: D The meaning of poverty differs across cultures. Western cultures view poverty negatively, whereas other cultures often respect the poor.

A nurse is implementing a primary prevention strategy for migrant workers. Which of the following activities would the nurse most likely implement? a. Caring for persons afflicted with pesticide exposure b. Setting up clinics for persons with HIV c. Doing research in the field related to heat stress d. Educating about prevention of such illnesses as HIV and infectious diseases

ANS: D The primary level of prevention often includes education for the prevention of diseases, measures to reduce pesticide exposure, and immunizations. Caring for persons afflicted with pesticide exposure, setting up clinics for persons with HIV, and doing research in the field related to heat stress are examples of tertiary prevention.

A case manager is fulfilling the role of coordinator. Which of the following nursing interventions would most likely be completed? a. Providing information to all parties about the situations affecting the client b. Educating the client and providers in order to make informed decisions c. Supporting all parties to work toward mutual goals d. Arranging, regulating, and balancing needed health services for the client

ANS: D The role of coordinator is fulfilled when the nurse arranges, regulates, and balances needed health services for the client. Providing information to all parties about the client's situation is the role of the monitor/reporter. Educating the client and provider in order to make informed decisions is the role of educator, and supporting all parties to work toward mutual goals is the role of facilitator.

A nurse is investigating the structure of the community. Which of the following indicators would the nurse most likely collect data about? a. Infant mortality rate b. Effective communication c. Crime rate d. Emergency room utilization

ANS: D The structure of the community is defined in terms of services and resources. The subsystems of community structure consist of physical environment, health and social services, economy, transportation and safety, politics and government, communication, education, and recreation.

A nurse is collecting morbidity data. Which of the following would be the source that the nurse would use? a. Vital statistics reports b. Birth records c. Death certificates d. Specialized disease registries

ANS: D There are many sources of morbidity data (rate of disease incidence), including specialized disease registries. Vital statistics reports, birth records, and death certificates are part of mortality data.

A client comes into the clinic and tells the nurse he goes to an acupuncturist for pain. The nurse says he should take pain medication instead. Which of the following best describes the action taken by the nurse? a. Cultural conflict b. Cultural blindness c. Cultural relativism d. Cultural imposition

ANS: D This is an example of cultural imposition—the act of imposing one's cultural beliefs, values, and practices on individuals from another culture. Cultural conflict is a perceived threat that may arise from a misunderstanding of expectations when nurses are unable to respond appropriately to another individual's cultural practice because of unfamiliarity with the practice. Cultural blindness occurs when people state that everyone is treated the same, regardless of their cultural orientation. Cultural relativism recognizes that clients have different approaches to health, and that each culture should be judged on its own merit and not on the nurse's personal beliefs.

A nurse is investigating a potential bioterrorism attack. Which of the following evidence would a nurse find if a bioterrorism attack occurred? a. A large number of cases of influenza b. A case of cholera in a local politician following a trip to India c. Dengue fever in a group of students who just returned from a mission trip d. An unusual illness in a population

ANS: D Unusual illness in a population should trigger an investigation that may signal a covert bioterrorism attack. Cases of influenza, cholera, and dengue fever in a group would not be considered unexpected illnesses.

A nurse is working with a 17-year-old pregnant cocaine addict who is homeless. Which of the following best describes this client? a. At risk b. A special population c. A Healthy People 2020 target group d. A vulnerable individual

ANS: D Vulnerable individuals often have multiple risk factors. Vulnerable populations of concern to nurses are persons who are poor or homeless, have special needs, pregnant teens, migrant workers and immigrants, individuals with mental health problems, people who abuse addictive substances, persons who have been incarcerated, people with communicable diseases and those who are at risk, and persons who are HIV positive or have hepatitis B virus or STDs. Risk is an epidemiological term meaning that some people have a higher probability of illness than others.


Related study sets

CM III - Acute & Chronic Leukemia

View Set

Carbohydrates - Nutrition Counselling for the Dental Hygienist

View Set

Real Estate - Legal Aspects CHAPTER 8 SALE AND LEASE CONTRACTS

View Set

The Agricultural Revolution: Mastery Test

View Set

A+ Chapter 13: Operating System Basics

View Set

Chapter 14 Marketing Channels and Supply Chain Management

View Set